p03 - working capital finance

100
MANAGEMENT ADVISORY SERVICES WORKING CAPITAL FINANCE WORKING CAPITAL MANAGEMENT Net Working Capital 1 . During 1990, Mason Company’s current assets increased by $120, current liabilities decreased by $50, and net working capital (E) a. Increased by $70. c. Decreased by $170. b. Did not change. d . Increased by $170. CMA 1290 1-19 2 . During the year, Mason Company's current assets increased by $130, current liabilities decreased by $60, and net working capital (E) A. Increased by $70. C. Decreased by $190. B. Did not change. D . Increased by $190. Gleim 3 . A service enterprise's working capital at the beginning of January was $70,000. The following transactions occurred during January: Performed services on account $30,000 Purchased supplies on account 5,000 Consumed supplies 4,000 Purchased office equipment for cash 2,000 Paid short-term bank loan 6,500 Paid salaries 10,000 Accrued salaries 3,500 What is the amount of working capital at the end of January? (M) A . $80,500 C. $50,500 B. $78,500 D. $47,500 CIA 1193 IV-36 4 . The following are the January 1 and June 30 balance sheets of a company: Assets (in millions) Jan. 1 June 30 Cash $3 $4 Accounts receivable 5 4 Inventories 8 10 Fixed assets 10 11 Total assets $26 $29 Accounts payable $2 $3 Notes payable 4 3 Accrued wages 1 2 Long-term debt 9 11 Stockholder's equity 10 10 Total liabilities and stockholder's equity $26 $29 From January 1 to June 30, the net working capital (M) A. Decreased by $1 million. C . Increased by $1 million. B. Stayed the same. D. Increased by $2 million. CIA 1192 IV-52 CMA EXAMINATION QUESTIONS Page 1 of 100

Upload: elsa-mendoza

Post on 15-May-2017

1.003 views

Category:

Documents


126 download

TRANSCRIPT

Page 1: P03 - Working Capital Finance

MANAGEMENT ADVISORY SERVICES WORKING CAPITAL FINANCE

WORKING CAPITAL MANAGEMENTNet Working Capital1. During 1990, Mason Company’s current assets increased by $120, current liabilities

decreased by $50, and net working capital (E)a. Increased by $70. c. Decreased by $170.b. Did not change. d. Increased by $170. CMA 1290 1-19

2. During the year, Mason Company's current assets increased by $130, current liabilities decreased by $60, and net working capital (E)A. Increased by $70. C. Decreased by $190.B. Did not change. D. Increased by $190. Gleim

3. A service enterprise's working capital at the beginning of January was $70,000. The following transactions occurred during January:

Performed services on account $30,000Purchased supplies on account 5,000Consumed supplies 4,000Purchased office equipment for cash 2,000Paid short-term bank loan 6,500Paid salaries 10,000Accrued salaries 3,500

What is the amount of working capital at the end of January? (M)A. $80,500 C. $50,500B. $78,500 D. $47,500 CIA 1193 IV-36

4. The following are the January 1 and June 30 balance sheets of a company:Assets (in millions) Jan. 1 June 30Cash $3 $4Accounts receivable 5 4Inventories 8 10Fixed assets 10 11 Total assets $26 $29Accounts payable $2 $3Notes payable 4 3Accrued wages 1 2Long-term debt 9 11Stockholder's equity 10 10 Total liabilities and stockholder's equity $26 $29

From January 1 to June 30, the net working capital (M)A. Decreased by $1 million. C. Increased by $1 million.B. Stayed the same. D. Increased by $2 million. CIA 1192 IV-52

Covenant LimitationMaximum Loan Availment*. It is the policy of Franz Corp. that the current ratio cannot fall below 1.5 to 1.0. Its current

liabilities are P400,000 and the present current ratio is 2 to 1. How much is the maximum level of new short-term loans it can secure without violating the policy? (M)a. P400,000 c. P266,667b. P300,000 d. P800,000 RPCPA 1096

5. A firm's current ratio is currently 1.70 to 1. Management knows it cannot violate a working capital restriction contained in its bond indenture. If the firm's current ratio falls below 1.40 to 1, technically it will have defaulted. If current liabilities are $200 million, the maximum new commercial paper that can be issued to finance inventory expansion is (M)A. $80 million. C. $150 million.B. $370 million. D. $280 million. Gleim

6. A firm's current ratio is currently 1.75 to 1. Management knows it cannot violate a working capital restriction contained in its bond indenture. If the firm's current ratio falls below 1.5 to 1, technically it will have defaulted. If current liabilities are $250 million, the maximum new commercial paper that can be issued to finance inventory expansion is (M)A. $375.00 million. C. $562.50 million.B. $125.00 million. D. $437.50 million. CMA 0683 1-8, 1292 1-23

7. Management of a company does not want to violate a working capital restriction contained in its bond indenture. If the firm's current ratio falls below 2.0 to 1, technically it will have defaulted. The firm's current ratio is now 2.2 to 1. If current liabilities are $200 million, the maximum new commercial paper that can be issued to finance inventory expansion is (M)A. $20 million. C. $240 million.B. $40 million. D. $180 million. Gleim

8. Iken Berry Farms has $5 million in current assets, $3 million in current liabilities, and its initial inventory level is $1 million. The company plans to increase its inventory, and it will raise additional short-term debt (that will show up as notes payable on the balance sheet) to purchase the inventory. Assume that the value of the remaining current assets will not change. The company’s bond covenants require it to maintain a current ratio that is greater than or equal to 1.5. What is the maximum amount that the company can increase its inventory before it is restricted by these covenants?a. $0.50 million d. $1.66 millionb. $1.00 million e. $2.33 millionc. $1.33 million Brigham

CMA EXAMINATION QUESTIONS Page 1 of 61

Page 2: P03 - Working Capital Finance

MANAGEMENT ADVISORY SERVICES WORKING CAPITAL FINANCE

Maximum Cash Dividend9. MFC Corporation has 100,000 shares of stock outstanding. Below is part of MFC’s Statement

of Financial Position for the last fiscal year.MFC Corporation

Statement of Financial Position – Selected ItemsDecember 31, 1996

Cash $455,000Accounts receivable 900,000Inventory 650,000Prepaid assets 45,000

Accrued liabilities 285,000Accounts payable 550,000Current portion, long-term notes payable 65,000

What is the maximum amount MFC can pay in cash dividends per share and maintain a minimum current ratio of 2 to 1? Assume that all accounts other than cash remain unchanged. (M)a. $2.05 c. $3.35b. $2.50 d. $3.80 CMA 0697 1-16

Effect of Plant Expansion on Working Capital10. Shaw Corporation is considering a plant expansion that will increase its sales and net income.

The following data represent management’s estimate of the impact the proposal will have on the company:

Current ProposalCash $ 100,000 $ 120,000Accounts payable 350,000 430,000Accounts receivable 400,000 500,000Inventory 380,000 460,000Marketable securities 200,000 200,000Mortgage payable (current) 175,000 325,000Fixed assets 2,500,000 3,500,000Net income 500,000 650,000

The effect of the plant expansion of Shaw’s working capital will be a(n) (M)a. Decrease of $150,000. c. Increase of $30,000.b. Decrease of $30,000. d. Increase of $120,000. CMA 1292 1-22

11. Finan Corporation's management is considering a plant expansion that will increase its sales and have commensurate impact on its net working capital position. The following information presents management's estimate of the impact the proposal will have on Finan.

Current ProposalCash $ 100,000 $ 110,000Accounts payable 400,000 470,000Accounts receivable 560,000 690,000Inventory 350,000 380,000Marketable securities 200,000 200,000Fixed assets 2,500,000 3,500,000Net income 500,000 650,000

The impact of the plant expansion on Finan's working capital would be (M)A. A decrease of $100,000. C. An increase of $100,000.B. A decrease of $950,000. D. An increase of $950,000. CMA 1286 1-29

12. The Herb Salter Corporation is considering a plant expansion that will increase its sales and net income. The following data represent management's estimate of the impact the proposal will have on the company:

Current ProposedCash $ 120,000 $ 140,000Accounts payable 360,000 450,000Accounts receivable 400,000 550,000Inventory 360,000 420,000Marketable securities 180,000 180,000Mortgage payable (current) 160,000 310,000Fixed assets 2,300,000 3,200,000Net income 400,000 550,000

The effect of the plant expansion on Salter's working capital will be a(n) (M)A. Increase of $240,000 C. Increase of $230,000B. Decrease of $10,000 D. Increase of $10,000 Gleim

WORKING CAPITAL FINANCING POLICYModerate 13. Wildthing Amusement Company’s total assets fluctuate between $320,000 and $410,000,

while its fixed assets remain constant at $260,000. If the firm follows a maturity matching or moderate working capital financing policy, what is the likely level of its long-term financing? (E)a. $ 90,000 d. $410,000b. $260,000 e. $320,000c. $350,000 Brigham

Conservative23. Great Company has P8,000,000 in current assets, P3,500,000 of which are considered

permanent current assets. In addition, the firm has P6,000,000 invested in fixed assets. Great Company wishes to finance all fixed assets and permanent current assets plus half of its

CMA EXAMINATION QUESTIONS Page 2 of 61

Page 3: P03 - Working Capital Finance

MANAGEMENT ADVISORY SERVICES WORKING CAPITAL FINANCE

temporary current assets with long-term financing costing 15%. Short-term financing currently costs 10%. Great Company’s earnings before interest and taxes are P2,200,000. Income tax rate is 40%.How much would Real Company’s earnings after taxes be under this financing plan?A. P112,500 C. P225,000B. P127,500 D. P85,000 Pol Bobadilla

Aggressive49. Normal Company has total fixed assets of P100,000 and no current liabilities. The table below

displays its wide variation in current asset components:1st Qtr 2nd Qtr 3rd Qtr 4th Qtr

Cash P 20,000 P 10,000 P 15,000 P 20,000Accounts receivable 66,000 25,000 47,000 88,000Inventory 20,000 65,000 59,000 10,000 Total P106,000 P100,000 P121,000 P118,000

If Normal’s policy is to finance all fixed assets and half the permanent current assets with long-term financing and the rest with short-term financing, what is the level of long-term financing? (D)A. P68,000 C. P150,000B. P100,000 D. P155,625 Pol Bobadilla

Working Capital Policy Options14. Mason Company's board of directors has determined 4 options to increase working capital

next year. Option 1 is to increase current assets by $120 and decrease current liabilities by $50. Option 2 is to increase current assets by $180 and increase current liabilities by $30. Option 3 is to decrease current assets by $140 and increase current liabilities by $20. Option 4 is to decrease current assets by $100 and decrease current liabilities by $75. Which option should Mason choose to maximize net working capital? A. Option 1. C. Option 3.B. Option 2. D. Option 4. Gleim

15. Jarrett Enterprises is considering whether to pursue a restricted or relaxed current asset investment policy. The firm’s annual sales are $400,000; its fixed assets are $100,000; debt and equity are each 50 percent of total assets. EBIT is $36,000, the interest rate on the firm’s debt is 10 percent, and the firm’s tax rate is 40 percent. With a restricted policy, current assets will be 15 percent of sales. Under a relaxed policy, current assets will be 25 percent of sales. What is the difference in the projected ROEs between the restricted and relaxed policies? (M)a. 0.0% d. 1.6%b. 6.2% e. 3.8%c. 5.4% Brigham

ComprehensiveQuestions 56 thru 61 are based on the following information. GitmanIrish Air Services has determined several factors relative to its asset and financing mix. The firm earns 10 percent annually on its current assets. The firm earns 20 percent annually on its fixed assets. The firm pays 13 percent annually on current liabilities. The firm pays 17 percent annually on long-term funds. The firm's monthly current, fixed and total asset requirements for the previous year are

summarized in the table below:Month Current Assets Fixed Assets Total AssetsJanuary $45,000 $100,000 $145,000February 40,000 100,000 140,000March 50,000 100,000 150,000April 55,000 100,000 155,000May 60,000 100,000 160,000June 75,000 100,000 175,000July 75,000 100,000 175,000August 75,000 100,000 175,000September 60,000 100,000 160,000October 55,000 100,000 155,000November 50,000 100,000 150,000December 50,000 100,000 150,000

56. The firm's monthly average permanent funds requirement is (E)A. $100,000. C. $140,000.B. $57,500. D. $157,500.

57. The firm's monthly average seasonal funds requirement is (M)A. $17,500. C. $40,000.B. $57,500. D. $157,500.

58. The firm's annual financing costs of the aggressive financing strategy is (M)A. $21,175. C. $24,475.B. $26,075. D. $22,775.

59. The firm's annual financing costs of conservative financing strategy is (M)A. $22,775. C. $29,750.B. $26,075. D. $21,175.

60. The firm's annual profits on total assets for the previous year was (M)A. $20,000. C. $23,625.

CMA EXAMINATION QUESTIONS Page 3 of 61

Page 4: P03 - Working Capital Finance

MANAGEMENT ADVISORY SERVICES WORKING CAPITAL FINANCE

B. $21,500. D. $25,750.

61. If the firm's current liabilities in December were $40,000, the net working capital was (E)A. $140,000. C. $10,000.B. $60,000. D. -$10,000.

Questions 62 thru 68 are based on the following information. GitmanFlum Packages, Inc.

Assets Liabilities & EquityCurrent assets $10,000 Current Liabilities $ 5,000Fixed assets 20,000 Long-term debt 12,000

Equity 13,000Total $30,000 Total $30,000

The company earns 5 percent on current assets and 15 percent on fixed assets. The firm's current liabilities cost 7 percent to maintain and the average annual cost of long-term funds is 20 percent.

62. The firm's initial ratio of current to total asset is _____. A. 1:3 C. 2:3B. 3:1 D. 3:2

63. The firm's initial net working capital isA. -$ 5,000. C. $ 5,000.B. $13,000. D. $10,000.

64. The firm's initial annual profits on total assets is A. $2,500. C. $3,000.B. $3,500. D. $4,500.

65. If the firm was to shift $3,000 of current assets to fixed assets, the firm's net working capital would _____, the annual profits on total assets would _____, and the risk of technical insolvency would _____, respectively.A. increase; decrease; increase C. increase; decrease; decreaseB. decrease; increase; decrease D. decrease; increase; increase

66. If the firm was to shift $7,000 of fixed assets to current assets, the firm's net working capital would _____, the annual profits on total assets would _____, and the risk of not being able to meet current obligations would _____, respectively. A. increase; decrease; increase C. increase; decrease; decreaseB. decrease; increase; decrease D. decrease; increase; increase

67. If the firm was to shift $2,000 of current liabilities to long-term funds, the firm's net working capital would _____, the annual cost of financing would _____, and the risk of technical insolvency would _____, respectively. A. decrease; decrease; increase C. decrease; increase; decreaseB. increase; increase; decrease D. increase; decrease; decrease

68. The firm would like to increase its current ratio. This goal would be accomplished most profitably byA. increasing current liabilities. C. increasing current assets.B. decreasing current liabilities. D. decreasing current assets.

CASH MANAGEMENTCash Conversion Cycle29. A firm has an average age of inventory of 60 days, an average collection period of 45 days,

and an average payment period of 30 days. The firm's cash conversion cycle is ______ days. (E)A. 15 C. 75B. 45 D. 135 Gitman

34. A firm has an average age of inventory of 101 days, an average collection period of 49 days, and an average payment period of 60 days. The firm's cash conversion cycle is (E)A. 150 days. C. 112 days.B. 90 days. D. 8 days Gitman

37. A firm has an average age of inventory of 20 days, an average collection period of 30 days, and an average payment period of 60 days. The firm's cash conversion cycle is _____ days. (E)A. 70 C. -10B. 50 D. 110 Gitman

16. If the average age of inventory is 60 days, the average age of the accounts payable is 30 days, and the average age of accounts receivable is 45 days, the number of days in the cash flow cycle is (E)A. 135 days. C. 75 days.B. 90 days. D. 105 days. Gleim

17. If the average days of inventory is 90 days, the average age of accounts payable is 60 days, and the average age of accounts receivable is 65 days, the number of days in the cash flow cycle is (E)a. 215 days c. 95 daysb. 150 days d. 85 days CMA 1284 1-20, RPCPA 1096

CMA EXAMINATION QUESTIONS Page 4 of 61

Page 5: P03 - Working Capital Finance

MANAGEMENT ADVISORY SERVICES WORKING CAPITAL FINANCE

18. For the Cook County Company, the average age of accounts receivable is 60 days, the average age of accounts payable is 45 days, and the average age of inventory is 72 days. Assuming a 360-day year, what is the length of the firm’s cash conversion cycle? (E)a. 87 days d. 48 daysb. 90 days e. 66 daysc. 65 days Brigham

19. A growing company is assessing current working capital requirements. An average of 58 days is required to convert raw materials into finished goods and to sell them. Then an average of 32 days is required to collect on receivables. If the average time the company takes to pay for its raw materials is 15 days after they are received, then the total cash conversion cycle for this company is (E)A. 11 days. C. 75 days.B. 41 days. D. 90 days. CIA 0596 IV-53

20. Spartan Sporting Goods has $5 million in inventory and $2 million in accounts receivable. Its average daily sales are $100,000. The company’s payables deferral period (accounts payable divided by daily purchases) is 30 days. What is the length of the company’s cash conversion cycle? (E)a. 100 days d. 40 daysb. 60 days e. 33 daysc. 50 days Brigham

31. A firm purchased raw materials on account and paid for them within 30 days. The raw materials were used in manufacturing a finished good sold on account 100 days after the raw materials were purchased. The customer paid for the finished good 60 days later. The firm's cash conversion cycle is ______ days. (M)A. 10 C. 130B. 70 D. 190 Gitman

87. A firm with a cash conversion cycle of 175 days can stretch its average payment period from 30 days to 45 days. This will result in a(n) _____ in the cash conversion cycle of _____ days. (M)A. increase, 15 C. increase, 45B. decrease, 15 D. decrease, 45 Gitman

21. Porta Stadium Inc. has annual sales of $40,000,000 and keeps average inventory of $10,000,000. On average, the firm has accounts receivable of $8,000,000. The firm buys all raw materials on credit, its trade credit terms are net 30 days, and it pays on time. The firm’s managers are searching for ways to shorten the cash conversion cycle. If sales can be

maintained at existing levels but inventory can be lowered by $2,000,000 and accounts receivable lowered by $1,000,000, what will be the net change in the cash conversion cycle? Use a 360-day year. (M)a. +105 days d. -27 daysb. -105 days e. -3 daysc. +27 days Brigham

22. Bully Corporation purchases raw materials on July 1. It converts the raw materials into inventory by September 30. However, Bully pays for the materials on July 20. On October 31, it sells the finished goods inventory. Then, the firm collects cash from the sale 1 month later on November 30. If this sequence accurately represents the average working capital cycle, what is the firm's cash conversion cycle in days? (D)A. 92 days. C. 123 days.B. 133 days. D. 153 days. Gleim

23. You have recently been hired to improve the performance of Multiplex Corporation, which has been experiencing a severe cash shortage. As one part of your analysis, you want to determine the firm’s cash conversion cycle. Using the following information and a 360-day year, what is your estimate of the firm’s current cash conversion cycle? (M)Current inventory = $120,000.Annual sales = $600,000.Accounts receivable = $160,000.Accounts payable = $25,000.Total annual purchases = $360,000.Purchases credit terms: net 30 days.Receivables credit terms: net 50 days.a. 49 days d. 168 daysb. 193 days e. 143 daysc. 100 days Brigham

24. Gaston Piston Corp. has annual sales of $50,000,000 and maintains an average inventory level of $15,000,000. The average accounts receivable balance outstanding is $10,000,000. The company makes all purchases on credit and has always paid on the 30th day. The company is now going to take full advantage of trade credit and pay its suppliers on the 40th day. If sales can be maintained at existing levels but inventory can be lowered by $2,000,000 and accounts receivable lowered by $2,000,000, what will be the net change in the cash conversion cycle? (Assume there are 360 days in the year.) (M)a. -14.4 days d. -25.6 daysb. -18.8 days e. -38.8 daysc. -28.8 days Brigham

CMA EXAMINATION QUESTIONS Page 5 of 61

Page 6: P03 - Working Capital Finance

MANAGEMENT ADVISORY SERVICES WORKING CAPITAL FINANCE

25. Kolan Inc. has annual sales of $36,500,000 ($100,000 a day on a 365-day basis). On average, the company has $12,000,000 in inventory and $8,000,000 in accounts receivable. The company is looking for ways to shorten its cash conversion cycle, which is calculated on a 365-day basis. Its CFO has proposed new policies that would result in a 20 percent reduction in both average inventories and accounts receivables. The company anticipates that these policies will also reduce sales by 10 percent. Accounts payable will remain unchanged. What effect would these policies have on the company’s cash conversion cycle? (M)a. -40 days d. +22 daysb. -22 days e. +40 daysc. -13 days Brigham

26. Jordan Air Inc. has average inventory of $1,000,000. Its estimated annual sales are 12 million and the firm estimates its receivables conversion period to be twice as long as its inventory conversion period. The firm pays its trade credit on time; its terms are net 30. The firm wants to decrease its cash conversion cycle by 10 days. It believes that it can reduce its average inventory to $900,000. Assume a 360-day year and that sales will not change. By how much must the firm also reduce its accounts receivable to meet its goal of a 10-day reduction? (D)a. $ 101,900 d. $ 333,520b. $1,000,000 e. $ 0c. $ 233,333 Brigham

209.A firm has arranged for a lockbox system to reduce collection time of accounts receivable. Currently the firm has an average collection period of 43 days, an average age of inventory of 50 days, and an average payment period of 10 days. The lockbox system will reduce the average collection period by 3 days by reducing processing, mail, and clearing float. The firm's cash conversion cycle _____. (E)A. increases by 3 days C. will not changeB. decreases by 3 days D. is 93 days

Annual Savings81. A firm has annual operating outlays of $1,800,000 and a cash conversion cycle of 60 days. If

the firm currently pays 12 percent for negotiated financing and reduces its cash conversion cycle to 50 days, the annual savings is A. $ 50,000 C. $ 6,000.B. $200,000 D. $216,000. Gitman

82. A firm has a cash conversion cycle of 60 days. Annual outlays are $12 million and the cost of negotiated financing is 12 percent. If the firm reduces its average age of inventory by 10 days, the annual savings is ______.A. $104,000 C. $ 28,800B. $144,000 D. $40,000 Gitman

Cash Flow27. FLF Corporation had income before taxes of $50,000. Included in the calculation of this

amount was depreciation of $6,000, a charge of $7,000 for the amortization of bond discounts, and $5,000 for interest paid. The estimated pretax cash flow for the period is (M)A. $50,000 C. $37,000B. $57,000 D. $63,000 Gleim

28. RLF Corporation had income before taxes of $60,000 for the year 1991. Included in this amount was depreciation of $5,000, a charge of $6,000 for amortization of bond discounts, and $4,000 for interest expense. The estimated cash flow for the period is (M)a. $60,000. c. $49,000.b. $66,000. d. $71,000. CMA 0692 1-26

29. Shown below is a forecast of sales for Cooper Inc. for the first 4 months of the year (all amounts are in thousands of dollars).

January February March AprilCash sales $ 15 $ 24 $18 $14Sales on credit 100 120 90 70

On average, 50% of credit sales are paid for in the month of sale, 30% in the month following the sale, and the remainder is paid 2 months after the month of sale. Assuming there are no bad debts, the expected cash inflow for Cooper in March is (M)A. $138,000 C. $119,000B. $122,000 D. $108,000 CMA 1295 1-8

Baumol’s Model of Cash BalancesNumber of Conversions7. Suppose that the interest rate on Treasury bills is 6%, and every sale of bills costs $20. You

pay out cash at a rate of $400,000 a month. According to Baumol's model of cash balances, how many times a month should you sell bills? A. 30 C. 7B. 20 D. 4 B & M

6. Suppose that the interest rate on Treasury bills is 4%, and every sale of bills costs $40. You pay out cash at a rate of $1,000,000 a quarter. According to Baumol's model of cash balances, how many times a quarter should you sell bills? (Approximately.) A. 20 C. 12B. 22 D. 11 B & M

CMA EXAMINATION QUESTIONS Page 6 of 61

Page 7: P03 - Working Capital Finance

MANAGEMENT ADVISORY SERVICES WORKING CAPITAL FINANCE

4. Suppose that the interest rate on Treasury bills is 6%, and every sale of bills costs $60. You pay out cash at a rate of $800,000 a year. According to Baumol's model of cash balances, how many times a year should you sell bills? A. 20 C. 50B. 35 D. 15 B & M

Q5. Suppose that the interest rate on Treasury bills is 6%, and every sale of bills costs $60. You

pay out cash at a rate of $800,000 a year. According to Baumol's model of cash balances, what is Q? A. $17,376 D. $50,000B. $20,000 E. $40,000C. $10,000 B & M

8. Suppose that the interest rate on Treasury bills is 6%, and every sale of bills costs $20. You pay out cash at a rate of $400,000 a month. According to Baumol's model of cash balances, what is Q? A. $16,000 D. $43,000B. $24,000 E. $57,000 C. $31,000 B & M

Float Availability Float18. On an average day, a company writes checks totaling $1,500. These checks take 7 days to

clear. The company receives checks totaling $1,800. These checks take 4 days to clear. The cost of debt is 9%. What is the firm's availability float? A. $10,500 C. $1,800B. $7,200 D. None of the above B & M

Disbursement Float17. On an average day, a company writes checks totaling $1,500. These checks take 7 days to

clear. The company receives checks totaling $1,800. These checks take 4 days to clear. The cost of debt is 9%. What is the firm's disbursement float? A. $10,500 C. $1,800B. $1,500 D. None of the above B & M

20. H. Pottamus, Inc., has $2 million on deposit with the bank. It now writes checks for $100,000 and $200,000 and deposits a check for $80,000. Two weeks later it learns that the $200,000 check and $80,000 check have cleared. What is the company's disbursement float? A. $300,000 C. $100,000B. $220,000 D. -$100,000 B & M

Net Float19. On an average day, a company writes checks totaling $1,500. These checks take 7 days to

clear. The company receives checks totaling $1,800. These checks take 4 days to clear. The cost of debt is 9%. What is the firm's net float? A. $300 C. $2,100B. $3,300 D. $1,200 B & M

30. Assume that each day a company writes and receives checks totaling $10,000. If it takes 5 days for the checks to clear and be deducted from the company's account, and only 4 days for the deposits to clear, what is the float? (E)A. $10,000 C. $(10,000)B. $0 D. $50,000 CMA 0694 1-24

31. Average daily collection of checks for a firm is $40,000. The firm also writes on the average $35,000 of checks daily. If the collection period for checks is 5 days, calculate the net float. (E)A. $25,000 C. $175,000B. $40,000 D. $200,000 Gleim

32. Jumpdisk Company writes checks averaging $15,000 a day, and it takes five days for these checks to clear. The firm also receives checks in the amount of $17,000 per day, but the firm loses three days while its receipts are being deposited and cleared. What is the firm’s net float in dollars? (E)a. $126,000 d. $ 24,000b. $ 75,000 e. $ 16,000c. $ 32,000 Brigham

Annual Savings12. As part of a union negotiation agreement, the United Clerical Workers Union conceded to be

paid every two weeks instead of every week. A major firm employing hundreds of clerical workers had a weekly payroll of $1,000,000 and the cost of short-term funds was 12 percent. The effect of this concession was to delay clearing time by one week. Due to the concession, the firmA. realized an annual loss of $120,000. C. increased its cash cycle.B. realized an annual savings of $120,000. D. decreased its cash turnover. Gitman

Opportunity Cost33. What is the opportunity cost of keeping a cash balance of $2 million, if the daily interest rate is

0.02% and the average transaction cost of investing money overnight is $50? (E)A. $50 C. $400B. $350 D. $40,000 Gleim

CMA EXAMINATION QUESTIONS Page 7 of 61

Page 8: P03 - Working Capital Finance

MANAGEMENT ADVISORY SERVICES WORKING CAPITAL FINANCE

Questions 2 and 3 are based on the following information. CIA 0595 IV-45 & 46A company has a 10% cost of borrowing and incurs fixed costs of $500 for obtaining a loan. It has stable, predictable cash flows, and the estimated total amount of net new cash needed for transactions for the year is $175,000. The company does not hold safety stocks of cash.

34. When the average cash balance of the company is higher, the <List A> the cash balance is <List B>.

List A List BA. Opportunity cost of holding HigherB. Total transactions costs associated with obtaining HigherC. Opportunity cost of holding LowerD. Total costs of holding Lower

35. If the average cash balance for the company during the year is $20,916.50, the opportunity cost of holding cash for the year will be A. $2,091.65 C. $8,750.00B. $4,183.30 D. $17,500.00

Lock Box ServiceIncrease in Average Cash Balance36. CMR is a retail mail order firm that currently uses a central collection system that requires all

checks to be sent to its Boston headquarters. An average of 5 days is required for mailed checks to be received, 4 days for CMR to process them and 1½ days for the checks to clear through its bank. A proposed lockbox system would reduce the mail and process time to 3 days and the check clearing time to 1 day. CMR has an average daily collection of $100,000. If CMR should adopt the lockbox system, its average cash balance would increase by (E)a. $250,000. c. $650,000. b. $400,000. d. $800,000. CMA 1286 1-30

37. DLF is a retail mail order firm that currently uses a central collection system that requires all checks to be sent to its Boston headquarters. An average of 6 days is required for mailed checks to be received, 3 days for DLF to process them, and 2 days for the checks to clear through its bank. A proposed lockbox system would reduce the mailing and processing time to 2 days and the check clearing time to 1 day. DLF has an average daily collection of $150,000. If DLF adopts the lockbox system, its average cash balance will increase by (E)A. $1,200,000 C. $600,000B. $750,000 D. $450,000 Gleim

Daily Income (Loss)

24. Assume that the average number of daily payments to a lock-box is 200, the average size of the payment is $1,000, the rate of interest per day is 0.02% (i.e., 0.0002), the savings in mail time is 2 days, and the savings in processing time is 1 day. What is the daily return from operating the lock-box? A. $80 C. $120B. $100 D. $130 B & M

Increase in Annual Income (Loss)38. What are the expected annual savings from a lockbox system that collects 200 checks per day

averaging $500 each, and reduces mailing and processing times by 2.0 and 0.5 days, respectively, if the annual interest rate is 6%? (E)A. $250,000 C. $6,000B. $12,000 D. $15,000 Gleim

39. A firm has daily cash receipts of $300,000. A bank has offered to provide a lockbox service that will reduce the collection time by 3 days. The bank requires a monthly fee of $2,000 for providing this service. If money market rates are expected to average 6% during the year, the additional annual income (loss) of using the lockbox service is (E)A. ($24,000) C. $30,000B. $12,000 D. $54,000 Gleim

40. Foster Inc. is considering implementing a lock box collection system at a cost of $80,000 per year. Annual sales are $90 million, and the lockbox system will reduce collection time by 3 days. If Foster can invest funds at 8%, should it use the lockbox system? Assume a 360-day year. (E)a. Yes, producing savings of $140,000 per year.b. Yes, producing savings of $60,000 per year.c. No, producing a loss of $20,000 per year.d. No, producing a loss of $60,000 per year. CMA 1295 1-14

41. Cross Collectibles currently fills mail orders from all over the U. S. and receipts come in to headquarters in Little Rock, Arkansas. The firm’s average accounts receivable (A/R) is $2.5 million and is financed by a bank loan with 11 percent annual interest. Cross is considering a regional lockbox system to speed up collections that it believes will reduce A/R by 20 percent. The annual cost of the system is $15,000. What is the estimated net annual savings to the firm from implementing the lockbox system? (M)a. $500,000 d. $ 55,000b. $ 30,000 e. $ 40,000c. $ 60,000 Brigham

CMA EXAMINATION QUESTIONS Page 8 of 61

Page 9: P03 - Working Capital Finance

MANAGEMENT ADVISORY SERVICES WORKING CAPITAL FINANCE

42. A company has daily cash receipts of $150,000. The treasurer of the company has investigated a lock box service whereby the bank that offers this service will reduce the company’s collection time by four days at a monthly fee of $2,500. If money market rates average 4% during the year, the additional annual income (loss) from using the lock box service would be (E)a. $6,000. c. $12,000.b. $(6,000). d. $(12,000). CMA 0694 1-19

43. A banker has offered to set up and operate a lock box system for your company. Details are given below. Estimate the annual savings.

Average number of daily payments 325Average size of payments $1,250Daily interest rate 0.021%Saving in mailing time 1.3 daysSaving in processing time 0.9 daysBank charges $0.30

Assume 250 processing days per year. (M)A. $3,273 C. $23,500B. $22,675 D. $47,000 Gleim

44. Cleveland Masks and Costumes Inc. (CMC) has a majority of its customers located in the states of California and Nevada. Keystone National Bank, a major west coast bank, has agreed to provide a lockbox system to CMC at a fixed fee of $50,000 per year and a variable fee of $0.50 for each payment processed by the bank. On average, CMC receives 50 payments per day, each averaging $20,000. With the lockbox system, the company's collection float will decrease by 2 days. The annual interest rate on money market securities is 6%. If CMC makes use of the lockbox system, what would be the net benefit to the company? Use 365 days per year. (M)A. $59,125 C. $50,000B. $60,875 D. $120,000 CMA Samp Q1-6

Optimal Lock-Box Alternative45. Newman Products has received proposals from several banks to establish a lockbox system to

speed up receipts. Newman receives an average of 700 checks per day averaging $1,800 each, and its cost of short-term funds is 7% per year. Assuming that all proposals will produce equivalent processing results and using a 360-day year, which one of the following proposals is optimal for Newman? (M) CMA 0697 1-13a. A $0.50 fee per check. c. A fee of 0.03% of the amount collected.b. A flat fee of $125,000 per year. d. A compensating balance of $1,750,000

46. A firm has daily cash receipts of $300,000 and is interested in acquiring a lockbox service in order to reduce collection time. Bank 1's lockbox service costs $3,000 per month and will reduce collection time by 3

days. Bank 2's lockbox service costs $5,000 per month and will reduce collection time by 4

days. Bank 3's lockbox service costs $500 per month and will reduce collection time by 1 day. Bank 4's lockbox service costs $1,000 per month and will reduce collection time by 2

days.If money market rates are expected to average 6% during the year, and the firm wishes to maximize income, which bank should the firm choose? (M)A. Bank 1. C. Bank 3.B. Bank 2. D. Bank 4. Gleim

Other Cash Management SystemsChange in Profit (Loss)*. QRS makes large cash payments averaging P17,000 daily. The company changed from using

checks to sight drafts which will permit it to hold unto its cash for one extra day. If QRS can use the extra cash to earn 14% annually, what annual peso return will it earn? (E)a. P652.10 c. P6.52b. P6,521.00 d. P2,380 RPCPA 1097

47. What is the benefit for a firm with daily sales of $15,000 to be able to reduce the collection period by 2 days, given an 8% annual opportunity cost of funds? (M)A. $2,400 annual benefit. C. $600 annual benefit.B. $1,200 annual benefit. D. $7,500 annual benefit. Gleim

48. A firm has daily cash receipts of $100,000 and collection time of 2 days. A bank has offered to decrease the collection time on the firm’s deposits by two days for a monthly fee of $500. If money market rates are expected to average 6% during the year, the net annual benefit loss) from having this service is (M)a. $3,000 c. $0b. $12,000 d. $6,000 CMA 0696 1-12

49. A firm has daily cash receipts of $200,000. A commercial bank has offered to reduce the collection time by 3 days. The bank requires a monthly fee of $4,000 for providing this service. If money market rates will average 12% during the year, the additional annual income (loss) of having the service is (M)A. $(24,000). C. $66,240.B. $24,000. D. $68,000. CMA 0683 1-7

CMA EXAMINATION QUESTIONS Page 9 of 61

Page 10: P03 - Working Capital Finance

MANAGEMENT ADVISORY SERVICES WORKING CAPITAL FINANCE

50. A firm has daily cash receipts of $300,000. A commercial bank has offered to reduce the collection time by 2 days. The bank requires a monthly fee of $3,000 for providing this service. If the money market rates will average 11% during the year, the annual pretax income (loss) from using the service is (M)A. $(30,000) C. $66,000B. $30,000 D. $63,000 Gleim

Point of Indifference51. Average daily cash outflows are $3 million for Farms Inc. A new cash management system

can add 2 days to the disbursement schedule. Assuming Farms earns 10% on excess funds, how much should the firm be willing to pay per year for the cash management system. (E)a. $6,000,000. c. $1,500,000.b. $3,000,000. d. $600,000. CMA 1295 1-3

52. Troy Toys is a retailer operating in several cities. Its individual store managers deposit daily collections at a local bank in a noninterest-bearing checking account. Twice per week, the local bank issues a depository transfer check (DTC) to the central bank at headquarters. The controller of the company is considering using a wire transfer instead. The additional cost of each transfer would be $25; collections would be accelerated by two days; and an annual interest rate paid by the central bank is 7.2% (0.02% per day). At what amount of dollars transferred would it be economically feasible to use a wire transfer instead of the DTC? Assume a 360-day year. (M) CMA 1294 1-17a. It would never be economically feasible. c. Any amount greater than $173.b. $125,000 or above. d. Any amount greater than $62,500

53. Best Computers believes that its collection costs could be reduced through modification of collection procedures. This action is expected to result in a lengthening of the average collection period from 28 days to 34 days; however, there will be no change in uncollectible accounts. The company's budgeted credit sales for the coming year are $27,000,000, and short-term interest rates are expected to average 8%. To make the changes in collection procedures cost beneficial, the minimum savings in collection costs (using a 360-day year) for the coming year would have to be (M)A. $30,000. C. $180,000.B. $360,000. D. $36,000. CMA 1292 1-20

54. Best Computers believes that its collection costs could be reduced through modification of collection procedures. This action is expected to result in a lengthening of the average collection period from 30 to 35 days; however, there will be no change in uncollectible accounts, or in total credit sales. Furthermore, the variable cost ratio is 60%, the opportunity cost of a longer collection period is assumed to be negligible, the company's budgeted credit sales for the coming year are $45,000,000, and the required rate of return is 6%. To justify

changes in collection procedures, the minimum annual reduction of costs (using a 360-day year and ignoring taxes) must be (M)A. $375,000 C. $125,000B. $37,500 D. $22,500 Gleim

Checking Accounts55. Kemple is a newly established janitorial firm, and the owner is deciding what type of checking

account to open. Kemple is planning to keep a $500 minimum balance in the account for emergencies and plans to write roughly 80 checks per month. The bank charges $10 per month plus a $0.10 per check charge for a standard business checking account with no minimum balance. Kemple also has the option of a premium business checking account that requires a $2,500 minimum balance but has no monthly fees or per check charges. If Kemple’s cost of funds is 10%, which account should Kemple choose? a. Standard account, because the savings is $34 per year.b. Premium account, because the savings is $34 per year.c. Standard account, because the savings is $16 per year.d. Premium account, because the savings is $16 per year. CMA 0697 1-20

Economic Conversion Quantity (ECQ)Optimum Conversion Size47. Gear Inc.. has a total annual cash requirement of P9,075,000 which are to be paid uniformly.

Gear has the opportunity to invest the money at 24% per annum. The company spends, on the average, P40 for every cash conversion to marketable securities.What is the optimal cash conversion size?a. P60,000 c. P55,000b. P45,000 d. P72,500 Pol Bobadilla

Average Cash Balance56. A company uses the following formula in determining its optimal level of cash.

If b = fixed cost per transaction i = interest rate on marketable securities t = total demand for cash over a period of timeThis formula is a modification of the economic order quantity (EOQ) formula used for inventory management. Assume that the fixed cost of selling marketable securities is $10 per transaction and the interest rate of marketable securities is 6% per year. The company estimates that it will make cash payments of $12,000 over a one-month period. What is the average cash balance (rounded to the nearest dollar)? (E)a. $1,000 c. $3,464b. $2,000 d. $6,928 CMA 0696 1-10

CMA EXAMINATION QUESTIONS Page 10 of 61

Page 11: P03 - Working Capital Finance

MANAGEMENT ADVISORY SERVICES WORKING CAPITAL FINANCE

47. Mott Co. has a total annual cash requirement of P9,075,000 which are to be paid uniformly. Mott has the opportunity to invest the money at 24% per annum. The company spends, on the average, P40 for every cash conversion to marketable securities.What is the optimum average cash balance?a. P60,000 c. P45,000b. P55,000 d. P27,500 Pol Bobadilla

MARKETABLE SECURITIESCurrent Price57. Assuming a 360 day year, the current price of its $100 U.S. Treasury bill due in 180 days on a

6% discount basis is (E)a. $97.00 c. $100.00b. $94.00 d. $93.00 CMA 0694 1-26

58. Hendrix, Inc. is interested in purchasing a $100 U.S. Treasury bill and was presented with the following options:

Due Date Discount RateOption 1 180 days 6%Option 2 360 days 3.5%Option 3 120 days 8%Option 4 240 days 4.5%

If Hendrix wishes to buy the Treasury bill at the lowest purchasing price, which option should be chosen, assuming a 360-day year? (M)A. Option 1. C. Option 3.B. Option 2. D. Option 4. Gleim

Annually Compounded Rate of Return4. The discount on a 91-Treasury bill is 5.2%. What is the annually compounded rate of return?

A. 4.8% C. 5.4%B. 5.2% B & M

5. The discount on a 91-Treasury bill is 5.65%. What is the annually compounded rate of return? A. 5.2% C. 5.6%B. 5.9% D. 5.5% B & M

6. The discount on a 91-Treasury bill is 4.83%. What is the annually compounded rate of return? A. 4.83% D. 5.13%B. 4.78% E. 5.0%C. 1.22% B & M

Economic Conversion QuantityQuestions 98 and 99 are based on the following information. GleimSnobiz, Inc. has $2 million invested in Treasury bills yielding 8% per annum; this investment will satisfy the firm's need for funds during the coming year.

59. If it costs $50 to sell these bills, regardless of the amount, how much should be withdrawn at a time? (E)A. $50,000 C. $250,000B. $100,000 D. $500,000

60. If Snobiz, Inc. needs $167,000 a month, how frequently should the CFO sell off Treasury bills? (M)A. About every 3 days. C. About every 15 days.B. About every 9 days. D. About every 18 days.

Yield on Floating-rate Preferred Stock18. If the short-term commercial paper rate is 10% and the corporate tax rate is 35%, what yield

would a corporation require on an investment in floating-rate preferred stock? Assume the default risk is the same as for commercial paper. A. 15.2% C. 7.3%B. 10.0% D. 6.6% B & M

19. If the short-term commercial paper rate is 6% and the corporate tax rate is 35%, what yield would a corporation require on an investment in floating-rate preferred stock? Assume the default risk is the same as for commercial paper. A. 6.0% C. 9.2%B. 39% D. 4.4% B & M

20. If the short-term commercial paper rate is 8% and the corporate tax rate is 35%, what yield would a corporation require on an investment in floating-rate preferred stock? Assume the default risk is the same as for commercial paper. A. 5.8% C. 4.6%B. 5.3% D. 4.4% B & M

RECEIVABLES MANAGEMENTAccounts Receivable Balance*. JBJ Company’s account balance at June 30, 1987 for account receivables and related

allowances for doubtful accounts were P600,000 and P3,000 respectively. Aging of accounts receivable indicated that P48,000 of the June 30, 1987 receivable may be uncollectible. Net realizable value of accounts receivable were: (E)a. P597,000 c. P539,000

CMA EXAMINATION QUESTIONS Page 11 of 61

Page 12: P03 - Working Capital Finance

MANAGEMENT ADVISORY SERVICES WORKING CAPITAL FINANCE

b. P552,000 d. none of these RPCPA 1087

61. The Irwin Corporation has $3 million per year in credit sales. The company's average day's sales outstanding is 40 days. Assuming a 360-day year, what is Irwin's average amount of accounts receivable outstanding? (E)A. $500,000 C. $250,000B. $333,333 D. $75,000 Gleim

49. A company has sales of $1,800,000 (70% are credit), a gross profit ratio of 55%, an accounts receivable turnover of 12 times, and an inventory turnover of 4 times. The average accounts receivable balance isa. $202,500 c. $315,000b. $247,500 d. $450,000 ($105,000) L & H

*. Ten Q’s Inc. has an inventory conversion period of 60 days, a receivable conversion period of 35 days, and a payment cycle of 26 days. If its sales for the period just ended amounted to P972,000, what is the investment in accounts receivable? (Assume 360 days a year.) (E)a. P85,200 c. P94,500b. P72,450 d. P79,600 RPCPA 0595

62. For the Flesher Company, the average age of accounts receivable is 48 days, the average age of accounts payable is 32 days, and the average age of inventory is 59 days. Assume a 360-day year. If McIntyre's annual sales are $2,050,200, what is the firm's investment in accounts receivable? (E)A. $96,000 C. $182,240B. $336,005 D. $273,360 Gleim

63. For the Fratzie Company, the average age of accounts receivable is 48 days, the average age of accounts payable is 32 days, and the average age of inventory is 60 days. Assume a 360-day year. If Fratzie's annual sales are $2,870,280, what is the firm's investment in accounts receivable? (E)A. $127,567 C. $478,381B. $382,704 D. $637,839 Gleim

64. The company sells 10,000 units at a unit selling price of 66 annually. Assume that the average collection period is 25 days. After the credit policy is well established, what is the expected average accounts receivable balance for the company at any moment in time, assuming a 365-day year? (E)A. 684.93 C. 27,123.30B. 1,808.22 D. 45,205.48 CIA 0594 IV-36

65. Jackson Distributors sells to retail stores on credit terms of 2/10, net/30. Daily sales average 150 units at a price of $300 each. Assuming that all sales are on credit and 60% of customers take the discount and pay on day 10 while the rest of the customers pay on day 30. The amount of Jackson’s account receivable is (D)a. $1,350,000. c. $900,000. b. $990,000. d. $810,000. CMA 1295 1-6

95. Collectrite Company sells on terms 3/10, net 30. Total sales for the year are P900,000. Forty percent of the customers pay on the tenth day and take discounts, the other 60 percent pay, on average, 45 days after their purchases.What is the average amount of receivables?A. P70,000 C. P77,200B. P77,500 D. P67,500 Pol Bobadilla

66. A firm averages $4,000 in sales per day and is paid on an average, within 30 days of the sale. After they receive their invoice, 55% of the customers pay in cash while the remaining 45% pay by credit card. Approximately how much would the company have in accounts receivable on its balance sheet on a given date? (M)a. $4,000 c. $48,000 b. $120,000 d. $54,000 CMA 1294 1-22

*. Simba Corp., whose gross sales amounted to P1,200,000 sold on terms of 3/10, net 30. The collections manager estimated that 30% of the customers pay on the 10th day and take discounts; 40% on the 30th day; and the remaining 30% pay, on the average, 40 days after the purchase. If management would toughen on its collection policy and require that all non-discount customers pay on the 30th day, how much would be the receivables balance? (M)a. P60,000 c. P70,000b. P80,000 d. Zero RPCPA 0595

67. Ruth Company currently has $1,000,000 in accounts receivable. Its days sales outstanding (DSO) is 48 days. The company wants to reduce its DSO to the industry average of 32 days by pressuring more of its customers to pay their bills on time. The company’s CFO estimates that if this policy is adopted the company’s average sales will fall by 10 percent. Assuming that the company adopts this change and succeeds in reducing its DSO to 32 days and does lose 10 percent of its sales, what will be the level of accounts receivable following the change? Assume a 360-day year. (M)a. $600,000 d. $900,000b. $666,667 e. $966,667c. $750,000 Brigham

CMA EXAMINATION QUESTIONS Page 12 of 61

Page 13: P03 - Working Capital Finance

MANAGEMENT ADVISORY SERVICES WORKING CAPITAL FINANCE

Days Receivable68. An organization offers its customers credit terms of 5/10 net 20. One-third of the customers

take the cash discount and the remaining customers pay on day 20. On average, 20 units are sold per day, priced at $10,000 each. The rate of sales is uniform throughout the year. Using a 360-day year, the organization has days sales outstanding, to the nearest full day, of A. 13 days. C. 17 days.B. 15 days. D. 20 days. CIA 1195 IV-7

*. Hakuna Inc. sells on terms of 3/10, net 30 days. Gross sales for the year are P2,400,000 and the collections department estimates that 30% of the customers pay on the 10th day and take discounts; 40% pay on the 30th day; and the remaining 30% pay, on the average, 40 days after the purchase. Assuming 360 days per year, what is the average collection period. (M)a. 40 days. c. 20 daysb. 15 days. d. 27 days. RPCPA 0595

69. Clauson, Inc. grants credit terms of 1/15, net 30 and projects gross sales for next year of $2,000,000. The credit manager estimates that 40% of their customers pay on the discount date, 40% on the net due date, and 20% pay 15 days after the net due date. Assuming uniform sales and a 360-day year, what is the projected days’ sales outstanding rounded to the nearest whole day? (M)a. 20 days. c. 27 days.b. 24 days. d. 30 days. CMA 0697 1-9

Carrying Cost on Accounts Receivable94. The Tempo Company has an inventory conversion period of 60 days, a receivable conversion

period of 30 days, and a payable payment period of 45 days. The Tempo’s variable cost is 60% and annual fixed costs of P600,000. The current cost of capital for Tempo is 12%.If Tempo’s annual sales are P3,375,000 and all sales are on credit, what is the firm’s carrying cost on accounts receivable, using 360 days year?A. P281,250 C. P20,250B. P168,750 D. P56,250 Pol Bobadilla

Days Receivable & Average Accounts Receivable Balance70. Sixty percent of Baco's annual sales of $900,000 is on credit. If its year-end receivables

turnover is 4.5, what is the average collection period and the year-end receivables, respectively (assume a 365-day year)? (M)A. 81 days and $120,000. C. 73 days and $108,000.B. 73 days and $120,000. D. 81 days and $200,000. Gleim

Questions 48 and 49 are based on the following information. CIA 0594 IV-35 & 36

A company sells 10,000 skateboards a year at $66 each. All sales are on credit, with terms of 3/10, net 30, that is, a 3% discount if payment is made within 10 days; otherwise full payment is due at the end of 30 days. One half of the customers are expected to take advantage of the discount and pay on day 10. The other half are expected to pay on day 30. Sales are expected to be uniform throughout the year for both types of customers.

71. A company sells 10,000 skateboards a year at $66 each. All sales are on credit, with terms of 3/10, net 30, which means three percent discount if payment is made within 10 days; otherwise full payment is due at the end of 30 days. One half of the customers are expected to take advantage of the discount and pay on day 10. The other half are expected to pay on day 30. Sales are expected to be uniform throughout the year for both types of customers. What is the expected average collection period for the company? A. 10 days. C. 20 days.B. 15 days. D. 30 days.

72. Assume that the average collection period is 25 days. After the credit policy is well established, what is the expected average accounts receivable balance for the company at any point in time, assuming a 365-day year? (E)A. $684.93 C. $27,123.30B. $1,808.22 D. $45,205.48

Collection Efficiency28. A firm has annual sales of $365 million. Currently, customers take an average of 60 days to

pay. If the collection period can be permanently reduced by one day and the cost of capital is 10%, what is the increase in company value? A. Zero C. $500,000B. $100,000 D. $1 million B & M

Aging of Accounts ReceivableQuestions 160 & 161 are based on the following information. GitmanA breakdown of Teffan, Inc.'s outstanding accounts receivable dated June 30, 2003 on the basis of the month in which the credit sale was initially made follows. The firm extends 30-day credit terms.

Month of Credit Sale Accounts ReceivableJune, 2003 $ 410,000May, 2003 340,000April, 2003 270,000March, 2003 200,000February, 2003 or before 100,000 Total $1,320,000

160.Accounts receivable over 90 days total

CMA EXAMINATION QUESTIONS Page 13 of 61

Page 14: P03 - Working Capital Finance

MANAGEMENT ADVISORY SERVICES WORKING CAPITAL FINANCE

A. $200,000. C. $300,000.B. $470,000. D. $100,000.

161.An evaluation of the firm's collection efforts based on the aging schedule would suggestA. poor credit management. C. superior credit management.B. satisfactory credit management. D. overzealous collection efforts.

Customer Default29. The default rate of Demurrage Associates' new customers has been running at 10%. The

average sale for each new customer amounts to $800, generating a profit of $100 and a 40% chance of a repeat order next year. The default rate on repeat orders is only 2%. If the interest rate is 9%, what is the expected profit from each new customer? A. $88.70 D. $43.25B. $47.75 E. $50.83C. $101.00 B & M

30. The default rate of Don's new customers has been running at 20%. The average sale for each new customer amounts to $500, generating a profit of $200 and a 30% chance of a repeat order next year. The default rate on repeat orders is only 5%. If the interest rate is 6%, what is the expected profit from each new customer? A. $152.50 C. $275.00B. $149.53 D. $100.00 B & M

Seasonal DatingQuestions 10 and 11 are based on the following information. CIA 1194 IV-29 & 30Effective September 1, a company initiates seasonal dating as a component of its credit policy, allowing wholesale customers to make purchases early but not requiring payment until the retail selling season begins. Sales occur as follows:

Date of Sale Quantity SoldSeptember 1 300 units

October 1 100 unitsNovember 1 100 unitsDecember 1 150 unitsJanuary 1 50 units

Each unit has a selling price of $10, regardless of the date of sale. The terms of sale are 2/10 net 30, January 1 dating. All sales are on credit. All customers take the discount and abide by the terms of the discount policy. All customers take advantage of the new seasonal dating policy. The peak selling season for all customers is mid-November to late December.

73. For the selling firm, which of the following is not an expected advantage of initiating seasonal dating? CIA 1194 IV-29A. Reduced storage costs. C. Attractive credit terms for customers.B. Reduced credit costs. D. Reduced uncertainty about sales volume.

74. For sales after the initiation of the seasonal dating policy on September 1, total collections on or before January 11 will be A. $0 C. $6,860B. $6,370 D. $7,000 CIA 1194 IV-30

Change in Credit TermsPrime Rate75. The high cost of short-term financing has recently caused a company to reevaluate the terms

of credit it extends to its customers. The current policy is 1/10, net 60. If customers can borrow at the prime rate, at what prime rate must the company change its terms of credit in order to avoid an undesirable extension in its collection of receivables? (E)A. 2% C. 7%B. 5% D. 8% Gleim

Effect on Accounts Receivable Balance76. A firm sells on terms of 2/10 net 60. It sells 1,000 units per day at a unit price of $10. On 60%

of sales, customers take the cash discount. On the remaining 40% of sales, customers pay, on average, in 70 days. What would be the impact on the balance of accounts receivable if the firm initiates a more aggressive collection policy and is able to reduce the average payment period to 60 days for those customers not taking the cash discount? (Assume sales levels are unaffected by the change in policy.) (E)A. Decrease by $4,000. C. Decrease by $240,000.B. Decrease by $40,000. D. Decrease by $280,000. CIA 1196 IV-44

77. A company plans to tighten its credit policy. The new policy will decrease the average number of days for collection from 75 to 50 days and will reduce the ratio of credit sales to total revenue from 70% to 60%. The company estimates that projected sales will be 5% less if the proposed new credit policy is implemented. If projected sales for the coming year are $50 million, calculate the dollar amount of accounts receivable of this proposed change in credit policy. Assume a 360-day year. (M)a. $3,817,445 decrease. c. $3,333,334 decrease. b. $6,500,000 decrease. d. $18,749,778 increase. CMA 1294 1-24

*. Prest Corp. plans to tighten its credit policy. Below is the summary of changes:OLD NEW

Average number of days collection 75 50

CMA EXAMINATION QUESTIONS Page 14 of 61

Page 15: P03 - Working Capital Finance

MANAGEMENT ADVISORY SERVICES WORKING CAPITAL FINANCE

Ratio of credit sales to total sales 70% 60%Projected sales for the coming year is P100 million and it is estimated that the new policy will result in a 5% loss if the new policy is implemented. Assuming a 360-day year, what is the effect of the new policy on accounts receivable? (M)a. Decrease of P13 million. c. Decrease of P5 million.b. No change. d. Decrease of P 6.67 million. RPCPA 0596

78. Dartmoor Company’s budgeted sales for the coming year are $40,500,000, of which 80% are expected to be credit sales at terms of n/30. Dartmoor estimates that a proposed relaxation of credit standards will increase credit sales by 20% and increase the average collection period from 30 days to 40 days. Based on a 360 day year, the proposed relaxation of credit standards will result in an expected increase in the average accounts receivable balance of (M)a. $540,000 c. $900,000b. $2,700,000 d. $1,620,000 CMA 1292 1-21

48. Real Company’s budgeted sales for the coming year are P50,000,000 of which 75% are expected to be credit sales at terms of n/30. Real estimates that a proposed relaxation of credit standards will increase credit sales by 20% and increase the average collection period from 30 days to 40 days. Based on a 360-day year, the proposed relaxation of credit standards will increase average accounts receivable balance by: (E)a. P1,200,000 c. P1,875,000b. P3,125,000 d. P5,000,000 Pol Bobadilla

79. Flyn Company's budgeted sales for the coming year are expected to be $50,000,000, of which 75% are expected to be credit sales at terms of n/30. Flyn estimates that a proposed relaxation of credit standards will increase credit sales by 25% and increase the average collection period from 20 days to 30 days. Based on a 360-day year, the proposed relaxation of credit standards will result in an expected increase in the average accounts receivable balance of (M)A. $520,840 C. $2,083,340B. $1,822,930 D. $3,906,270 Gleim

*. Numero 1 Co.’s budgeted sales for the coming year are P96 million, of which 80% are expected to be credit sales at terms of n/30. The company estimates that a proposed relaxation of credit standards would increase credit sales by 30% and increase the average collection period form 30 days to 45 days. Based on a 360-day year, the proposed relaxation of credit standards would result to an increase in accounts receivable balance of (M)a. P6,880,000 c. P2,880,000b. P1,920,000 d. P6,080,000 RPCPA 0595

80. Cannon Company has enjoyed a rapid increase in sales in recent years, following a decision to sell on credit. However, the firm has noticed a recent increase in its collection period. Last year, total sales were $1 million, and $250,000 of these sales were on credit. During the year, the accounts receivable account averaged $41,096. It is expected that sales will increase in the forthcoming year by 50 percent, and, while credit sales should continue to be the same proportion of total sales, it is expected that the days sales outstanding will also increase by 50 percent. If the resulting increase in accounts receivable must be financed externally, how much external funding will Cannon need? Assume a 365-day year.a. $ 41,096 d. $106,471b. $ 51,370 e. $ 92,466c. $ 47,359 Brigham

New Accounts Receivable Balance81. Ruth Company currently has $1,000,000 in accounts receivable. Its days sales outstanding (DSO)

is 50 days. The company wants to reduce its DSO to the industry average of 32 days by pressuring more of its customers to pay their bills on time. The company’s CFO estimates that if this policy is adopted the company’s average sales will fall by 10 percent. Assuming that the company adopts this change and succeeds in reducing its DSO to 32 days and does lose 10 percent of its sales, what will be the level of accounts receivable following the change? Assume a 365-day year.a. $576,000 d. $900,000b. $633,333 e. $966,667c. $750,000 Brigham

Incremental Investment in Accounts Receivable*. Slippers Mart has sales of P3 million. Its credit period and average collection period are both

30 days and 1% of its sales end as bad debts. The general manager intends to extend the credit period to 45 days which will increase sales by P300,000. However, bad debts losses on the incremental sales would be 3%. Costs of products and related expenses amount to 40% exclusive of the cost of carrying receivables of 15% and bad debts expenses. Assuming 360 days a year, the change in policy would result to incremental investment in receivables of (M)a. P24,704. c. P701,573.b. P65,000. d. P9,750. RPCPA 1095

144.A firm is considering relaxing credit standards, which will result in annual sales increasing from $1.5 million to $1.75 million, the cost of annual sales increasing from $1,000,000 to $1,125,000, and the average collection period increasing from 40 to 55 days. The bad debt loss is expected to increase from 1 percent of sales to 1.5 percent of sales. The firm's required return on investments is 20 percent. The firm's cost of marginal investment in accounts receivable is (D)A. $5,556. C. $12,153.

CMA EXAMINATION QUESTIONS Page 15 of 61

Page 16: P03 - Working Capital Finance

MANAGEMENT ADVISORY SERVICES WORKING CAPITAL FINANCE

B. $9,943. D. $152,778. Gitman

Expected Discounts Taken*. The Liberal Sales Co. budgeted sales for the coming year are P30 million of which 80% are

expected to be on credit. The company wants to change its credit terms from n/30 to 2/10, n/30. If the new credit terms are adopted, the company estimates that cash discounts would be taken on 40% of the credit sales and the new uncollectible amount would be unchanged. The adoption of the new credit terms would result in expected discount availed of in the coming year of (E)a. P600,000 c. P480,000b. P288,000 d. P192,000 RPCPA 0596

82. Price Publishing is considering a change in its credit terms from n/30 to 2/10, n/30. The company’s budgeted sales for the coming year are $24,000,000, of which 90% are expected to be made on credit. If the new credit terms are adopted, Price estimates that discounts will be taken on 50% of the credit sales; however, uncollectible accounts will be unchanged. The new credit terms will result in expected discounts taken in the coming year of (M)a. $216,000 c. $240,000b. $432,000 d. $480,000 CMA 1292 1-19

83. Catfur Publishing is considering a change in its credit terms from n/20 to 3/10, n/20. The company's budgeted sales for the coming year are $20,000,000, of which 80% are expected to be made on credit. If the new credit terms are adopted, Catfur management estimates that discounts will be taken on 60% of the credit sales; however, uncollectible accounts will be unchanged. The new credit terms will result in expected discounts taken in the coming year of (M)A. $288,000 C. $360,000B. $480,000 D. $600,000 Gleim

Pretax Cost of Carrying Additional Investment in Receivables*. Mr. S. Mart assumed the presidency of Riches Corp. He instituted new policies and with

respect to credit policy, below is a summary of relevant information:Old Credit Policy New Credit Policy

Sales P1,800,000 P1,980,000Average collection period 30 days 36 days

The company requires a rate of return of 10% and a variable cost ratio of 60%. Using a 360-day year, the pre-tax cost of carrying the additional investment in receivables under the new policy would be (M)a. P4,800 c. P3,000b. P2,880 d. P4,080 RPCPA 1096

84. The following information regarding a change in credit policy was assembled by the Wilson Wax Company. The company has a required rate of return of 11% and a variable cost ratio of 50%. The opportunity cost of a longer collection period is assumed to be negligible.

Old Credit Policy New Credit PolicySales $4,600,000 $4,960,000Average collection period 30 days 35 days

The pretax cost of carrying the additional investment in receivables, assuming a 360-day year, is (M)A. $5,439 C. $13,778B. $10,878 D. $98,890 Gleim

85. The following information regarding a change in credit policy was assembled by the Wilson Wax Company. The company has a required rate of return of 10% and a variable cost ratio of 60%.

Old Credit Policy New Credit PolicySales $3,600,000 $3,960,000Average collection period 30 days 36 days

The pretax cost of carrying the additional investment in receivables, using a 360-day year, would be (M)A. $5,760. C. $8,160.B. $9,600. D. $960. CMA 1289 1-16

Effect on Before Tax Profit (Loss)138.A firm is analyzing a relaxation of credit standards that is expected to increase sales 10

percent. The firm is currently selling 400 units at an average sale price per unit of $575, and the variable cost per unit is $400 at the current sales volume. The average cost per unit is $425. What is the additional profit contribution from sales if credit standards are relaxed? (E)A. $23,000 C. $6,000B. $16,000 D. $7,000 Gitman

31. Terry's Place is currently experiencing a bad debt ratio of 4%. Terry is convinced that, with looser credit controls, this ratio will increase to 8%; however, she expects sales to increase by 10% as a result. The cost of goods sold is 80% of the selling price. Per $100 of current sales, what is Terry's expected profit under the proposed credit standards? A. $26.0 C. $13.2B. $15.4 D. $25.6 B & M

32. Tom's Toys is currently experiencing a bad debt ratio of 6%. Tom is convinced that, with tighter credit controls, he can reduce this ratio to 2%; however, he expects sales to drop by 8% as a result. The cost of goods sold is 75% of the selling price. Per $100 of current sales, what is Tom's expected profit under the proposed credit standards?

CMA EXAMINATION QUESTIONS Page 16 of 61

Page 17: P03 - Working Capital Finance

MANAGEMENT ADVISORY SERVICES WORKING CAPITAL FINANCE

A. $15.2 D. $27.0B. $23.0 E. $21.2 C. $19.0 B & M

33. Toni's Catering is currently experiencing a bad debt ratio of 5%. Toni is convinced that, with looser credit controls, this ratio will increase to 10%; however, she expects sales to increase by 20% as a result. The cost of goods sold is 90% of the selling price. Per $100 of current sales, what is Toni's expected profit under the proposed credit standards? A. $0 C. $10B. $12 D. $5 B & M

86. A company with $4.8 million in credit sales per year plans to relax its credit standards, projecting that this will increase credit sales by $720,000. The average collection period for new customers is expected to be 75 days, and the payment behavior of the existing customers is not expected to change. Variable costs are 80% of sales. The firm’s opportunity cost is 20% before taxes. Assuming a 360-day year, what is the company’s profit (loss) on the planned change in credit terms? (M)a. $0 c. $144,000b. $28,800 d. $120,000 CMA 0696 1-13

*. The Sales Director of Can Can Co. suggests that certain credit terms be modified. He estimates the following effects: Sales will increase by at least 20%. Accounts receivable turnover will be reduced to 8 times from the present turnover of 10

times. Bad debts, now at 1% of sales will increase to 1.5%. Sales before the proposed changes is at P900,000. Variable cost ratio is 55% and desired rate of return is 20%. Fixed expenses amount to P150,000.Should the company allow the revision of its credit terms? (M)a. Yes, because income will increase by P64,800 (P68,850).b. Yes, because losses will be reduced by P78,800.c. No, because income will be reduced by P13,000.d. No, because losses will increase by P28,000. RPCPA 0594

Effect on After Tax Profit (Loss)*. Wasting Resource Co. has annual credit sales of P4 million. Its average collection period is 40

days and bad debts are 5% of sales. The credit and collection manager is considering instituting a stricter collection policy, whereby bad debts would be reduced to 2% of total sales, and the average collection period would fall to 30 days. However, sales would also fall by an estimated P500,000 annually. Variable costs are 60% of sales and the cost of carrying

receivables is 12%. Assuming a tax rate of 35% and 360 days a year, the incremental change in the profitability of the company if stricter policy would be implemented would be (D)a. Zero as the positive and negative effects offset each other.b. A reduction in net income by P70,000.c. A reduction in net income by P38,350.d. A reduction in net income by P35,400. RPCPA 0597

87. Lawson Company has the opportunity to increase annual sales $100,000 by selling to a new group of customers. Based on sales, the uncollectible expense is expected to be 15% and collection costs will be 5%. The company’s manufacturing and selling expenses are 70% of sales, and the effective tax rate is 40%. If Lawson accepts this opportunity, the company’s after tax profit will increase by (M)a. $4,000. c. $10,000.b. $6,000. d. $9,000. CMA 1290 1-30

88. Parkison Company can increase annual sales by $150,000 if it sells to a new, riskier group of customers. The uncollectible accounts expense is expected to be 16% of sales, and collection costs will be 4%. The company's manufacturing and selling expenses are 75% of sales, and its effective tax rate is 38%. If Parkison accepts this opportunity, its after-tax income will increase by (M)A. $2,850 C. $7,500B. $4,650 D. $8,370 Gleim

*. Crest Co. has the opportunity to increase annual sales by P1 million by selling to new riskier customers. It has been estimated that uncollectible expenses would be 15% and collection costs 5%. The manufacturing and selling costs are 70% of sales and corporate tax is 35%. If it pursues this opportunity, the after tax profit will (M)a. Increase by P35,000. c. Increase by P65,000.b. Increase by P97,500. d. Remain the same. RPCPA 0596

Return on Equity89. Daggy Corporation has the following simplified balance sheet:

Cash $ 25,000 Current liabilities $200,000Inventories 190,000Accounts receivable 125,000 Long-term debt 300,000Net fixed assets 360,000 Common equity 200,000Total assets $700,000 Total claims $700,000

The company has been advised that their credit policy is too generous and that they should reduce their days sales outstanding to 36 days (assume a 365-day year). The increase in cash resulting from the decrease in accounts receivable will be used to reduce the company’s long-term debt. The interest rate on long-term debt is 10 percent and the company’s tax rate is 30

CMA EXAMINATION QUESTIONS Page 17 of 61

Page 18: P03 - Working Capital Finance

MANAGEMENT ADVISORY SERVICES WORKING CAPITAL FINANCE

percent. The tighter credit policy is expected to reduce the company’s sales to $730,000 and result in EBIT of $70,000. What is the company’s expected ROE after the change in credit policy? a. 14.88% d. 18.38%b. 16.63% e. 16.25%c. 15.86% ROE

Credit Policy Options90. A firm currently sells $500,000 annually with 3% bad debt losses. Two alternative policies are

available. Policy A would increase sales by $500,000, but bad debt losses on additional sales would be 8%. Policy B would increase sales by an additional $120,000 over Policy A and bad debt losses on the additional $120,000 of sales would be 15%. The average collection period will remain at 60 days (6 turns per year) no matter the decision made. The profit margin will be 20% of sales and no other expenses will increase. Assume an opportunity cost of 20%. What should the firm do? (M)A. Make no policy change. B. Change to only Policy A. C. Change to Policy B (means also taking Policy A first). D. All policies lead to the same total firm profit, thus all policies are equal. Gleim

ComprehensiveQuestions 96 & 97 are based on the following information. Pol BobadillaSmart, Inc. is considering changing its credit terms from 2/15, net 30, to 3/10, net 30 in order to speed up collections. At present, 40% of Smart’s customers take the 2% discount. Under the new terms, discount customers are expected to rise to 50%. Regardless of the credit terms, half of the customers who do not take the discount are expected to pay on time, whereas the remainder will pay 10 days late. The change does not involve a relaxation of credit standards; therefore bad debt losses are not expected to rise above their present 2% level. However, the more generous cash discount terms are expected to increase sales from P2 million to P2.6 million per year. Smart’s variable cost ratio is 75%, the interest rate on funds invested in accounts receivable is 9%, and the firm’s income tax rate is 40%.

96. The incremental carrying cost on receivable is (M)A. P843.75 C. P8,889B. P643.75 D. P6,667

97. The incremental after tax profit from the change in credit terms is (D)A. P68,493 C. P65,640B. 60,615 D. P57,615

Questions 140 thru 143 are based on the following information. Gitman

Caren's Canoes is considering relaxing its credit standards to encourage more sales. As a result, sales are expected to increase 15 percent from 300 canoes per year to 345 canoes per year. The average collection period is expected to increase to 40 days from 30 days and bad debts are expected to double the current 1 percent level. The price per canoe is $850, the variable cost per canoe is $650 and the average cost per unit at the 300 unit level is $700. The firm's required return on investment is 20 percent.

140.What is the firm's additional profit contribution from sales under the proposed relaxation of credit standards? (E)A. $2,250 C. $9,000B. $6,750 D. $69,000

141.What is the cost of marginal investments in accounts receivable under the proposed plan? (M)A. $1,817 C. $1,733B. $1,867 D. $1,617

142.What is the cost of marginal bad debts under the proposed plan? (E)A. $383 C. $3,315B. $765 D. $5,100

143.What is the net result of implementing the proposed plan? (M)A. + $3,952 C. + $2,083B. - $3,868 D. - $2,083

Questions 165 thru 170 are based on the following information. GitmanDizzy Animators, Inc. currently makes all sales on credit and offers no cash discount. The firm is considering a 3 percent cash discount for payment within 10 days. The firm's current average collection period is 90 days, sales are 400 films per year, selling price is $25,000 per film, variable cost per film is $18,750 per film, and the average cost per film is $21,000. The firm expects that the change in credit terms will result in a minor increase in sales of 10 films per year, that 75 percent of the sales will take the discount, and the average collection period will drop to 30 days. The firm's bad debt expense is expected to become negligible under the proposed plan. The bad debt expense is currently 0.5 percent of sales. The firm's required return on equal-risk investments is 20 percent.

165.What is the firm's marginal profit contribution from sales under the proposed plan of initiating the cash discount? A. $22,500 C. $62,500B. $40,000 D. $100,000

166.What is the marginal investment in accounts receivable under the proposed plan?

CMA EXAMINATION QUESTIONS Page 18 of 61

Page 19: P03 - Working Capital Finance

MANAGEMENT ADVISORY SERVICES WORKING CAPITAL FINANCE

A. $1,234,375 C. $1,567,300B. $1,382,500 D. $1,841,570

167.What is the cost of marginal investment in accounts receivable under the proposed plan? A. $313,460 C. $246,875B. $276,500 D. $368,314

168.What are the savings of marginal bad debts under the proposed plan?A. $500,000 C. $10,000B. $50,000 D. $5,000

169.What is the cost of the marginal cash discount? A. $768,750 C. $307,500B. $300,000 D. $230,625

170.What is the net result of increasing the cash discount?A. +$33,750 C. +$128,750B. -$33,750 D. -$58,750

FactoringAnnual Cost of Financing91. A company enters into an agreement with a firm that will factor the company’s accounts

receivable. The factor agrees to buy the company’s receivables, which average $100,000 per month and have an average collection period of 30 days. The factor will advance up to 80% of the face value of receivables at an annual rate of 10% and charge a fee of 2% on all receivables purchased. The controller of the company estimates that the company would save $18,000 in collection expenses over the year. Fees and interest are not deducted in advance. Assuming a 360-day year, what is the annual cost of financing? (M)a. 10.0% c. 14.0%b. 12.0% d. 17.5% CMA 0696 1-30

Proceeds92. A firm often factors its accounts receivable. Its finance company requires a 6% reserve and

charges a 1.4% commission on the amount of the receivables. The remaining amount to be advanced is further reduced by an annual interest charge of 15%. What proceeds (rounded to the nearest dollar) will the firm receive from the finance company at the time a $100,000 account due in 60 days is factored? (M)A. $92,600 C. $90,285B. $96,135 D. $85,000 Gleim

93. A firm that often factors its accounts receivable has an agreement with its finance company that requires the firm to maintain a 6% reserve and charges 1% commission on the amount of receivables. The net proceeds would be further reduced by an annual interest charge of 10% on the monies advanced. Assuming a 360-day year, what amount of cash (rounded to the nearest dollar) will the firm receive from the finance company at the time a $100,000 account that is due in 90 days is turned over to the finance company? (M)A. $93,000 C. $83,700B. $90,000 D. $90,675 CMA 0694 1-29

6 . Answer (B) is correct. If current liabilities are $250 million and the current ratio (current assets/current liabilities) is 1.75, current assets must be $437.5 million ($250 x 1.75). Hence, if X amount of commercial paper is issued, thereby increasing both current assets and current liabilities by X, the new current ratio (which cannot be less than 1.5) is $125 million.

($437.5 + X)/($250 + X) = 1.5 $437.5 + X = 1.5 ($250 + X)

$437.5 + X = $375 + 1.5X $437.5 = $375 + .5X

$62.5 = .5X X = $125

Answer (A) is incorrect because $375 million is the minimum amount of current assets required to keep the current ratio at the floor level of 1.5. Answer (C) is incorrect because $562.5 million is the sum of the existing current assets and $125 million. Answer (D) is incorrect because $437.5 million is the existing amount of current assets.

1 . REQUIRED: The effect on net working capital of an increase in current assets and a decrease in current liabilities.DISCUSSION: (D) Working capital is the excess of current assets over current liabilities. An increase in current assets or a decrease in current liabilities will increase working capital. Thus, net working capital increased by $170 ($120 + $50)

2 . Answer (D) is correct. Working capital is the excess of current assets over current liabilities. An increase in current assets or a decrease in current liabilities increases working capital. Thus, net working capital increased by $190 ($130 + $60). Answer (A) is incorrect because the decrease in current liabilities increases working capital. Answer (B) is incorrect because net working capital increased by $190. Answer (C) is incorrect because net working capital increased by $190.

3 . Answer (A) is correct. Working capital is the excess of total current assets (CA) over total current liabilities (CL). Thus, working capital at the end of January equals 80,500 computed as follows:

CMA EXAMINATION QUESTIONS Page 19 of 61

Page 20: P03 - Working Capital Finance

MANAGEMENT ADVISORY SERVICES WORKING CAPITAL FINANCE

94. A firm often factors its accounts receivable. The finance company requires an 8% reserve and charges a 1.5% commission on the amount of the receivable. The remaining amount to be advanced is further reduced by an annual interest charge of 16%. What proceeds (rounded to the nearest dollar) will the firm receive from the finance company at the time a $110,000 account that is due in 60 days is turned over to the finance company? (M)A. $81,950. C. $96,895.B. $83,630. D. $99,550. CMA 0683 1-13

CA*CL*Beginning working capital 70,000Performed services on account 30,000 INPurchased supplies on account -0- IIConsumed supplies (4,000) DNPurchased office equipment (2,000) DNPaid short-term bank loan-0- DDPaid salaries (10,000) DNAccrued salaries (3,500) NIWorking capital, end of January 80,500* N = no effect; I = increase; D = decreaseAnswer (B) is incorrect because 78,500 ignores the consumed supplies, the cash purchase of office equipment, and the accrued salaries. However, it incorrectly considers the supplies purchased on account and the repayment of the short-term bank loan. Answer (C) is incorrect because 50,500 does not include the services performed on account. Answer (D) is incorrect because 47,500 omits the services performed on account and accrued salaries but includes the repayment of short-term loan.

4 . Answer (C) is correct. Net working capital equals current assets (cash, accounts receivable, inventories for this company) minus current liabilities (accounts payable, notes payable, accrued wages). From January 1 to June 30, the net working capital increased by $1,000,000 {[($4 + $4 + $10) - ($3 + $3 + $2)] - [($3 + $5 + $8) - ($2 + $4 + $1)]}. Answer (A) is incorrect because a decrease of $1,000,000 results from omitting inventories. Answer (B) is incorrect because the difference between all assets and all liabilities stayed the same. Answer (D) is incorrect because an increase of $2,000,000 results from omitting accrued wages.

5 . Answer (C) is correct. Given that current liabilities are $200 million and the current ratio (current assets ÷ current liabilities) is 1.70, current assets must be $340 million ($200 million x 1.70). If X amount of commercial paper is issued, thereby increasing both current assets and current liabilities by X, and the new current ratio is the contractual minimum of 1.40 to 1, the maximum amount of commercial paper that can be issued is $150 million.

($340 + X) current assets ÷ ($200 + X) current liabilities = 1.4 $340 + X = 1.4 ($200 + X)

$340 + X = 280 + 1.4X $340 = $280 + .4X

$60 = .4X X = $150

95. Gatsby, Inc. is going to begin factoring its accounts receivable and has collected information on the following four finance companies:

Required Reserves Commissions Annual InterestChargeCompany A 6% 1.4% 15%Company B 7% 1.2% 12%Company C 5% 1.7% 20%Company D 8% 1.0% 5%

Which company will give Gatsby the highest proceeds from a $100,000 account due in 60 days? Assume a 360-day year. (D)A. Company A. C. Company C.B. Company B. D. Company D. Gleim

Discounting of Notes Receivable*. On September 15, 19x7, LTW Corporation accepted from a customer a P100,000, 90-day,

20% interest-bearing note dated the same day. On October 15, 19x7, LTW discounted the note at the Western Bank at 23% discount. The customer paid the note at maturity. Based on a 360-day year, what amount should LTW report as net interest revenue from the note transaction? 1 (D)

1 20P100,000 x 20% x 90/360) = P5,000P105,000 x 23% x 60/360) = 4,025 Net interest revenue 975

Answer (A) is incorrect because $80 million results in a current ratio of 1.50. Answer (B) is

8. Current ratio and inventory Answer: b Diff: E NWith the numbers provided, we can see that Iken Berry Farms has a current ratio of 1.67 (CA/CL = $5/$3 = 1.67). If notes payable are going to be raised to buy inventories, both the numerator and the denominator of the ratio will increase. We can increase current liabilities $1 million before the current ratio reaches 1.5.

CMA EXAMINATION QUESTIONS Page 20 of 61

Page 21: P03 - Working Capital Finance

MANAGEMENT ADVISORY SERVICES WORKING CAPITAL FINANCE

a. P975 c. P5,000.b. P20,000. d. P4,025. RPCPA 1097

ComprehensiveQuestions 204 through 206 are based on the following information. GleimFlesher, Inc.'s credit manager studied the bill-paying habits of its customers and found that 90% of them were prompt. She also discovered that 22% of the slow payers and 5% of the prompt ones subsequently defaulted. The company has 3,000 accounts on its books, none of which has yet defaulted.

is $320,000, and $260,000 of that balance is fixed assets, permanent current assets equal $60,000. The likely level of long-term financing is $320,000.Long-term debt financing = Permanent cash assets + Fixed assets.

Permanent cash assets = Low end of total assets - Fixed assets = $320,000 - $260,000 = $60,000.

Long-term debt financing = $60,000 + $260,000 = $320,000.

19 . Answer (C) is correct. The cash conversion cycle is the length of time between paying for purchases and receiving cash from the sale of finished goods. It equals the inventory conversion period, plus the receivables collection period, minus the payables deferral period, or 75 days (58 days + 32 days - 15 days). Answer (A) is incorrect because 11 days results from subtracting the receivables collection period. Answer (B) is incorrect because 41 days results from subtracting the receivables collection period and adding the payables deferral period. Answer (D) is incorrect because 90 days omits the payables deferral period.

22 . Answer (B) is correct. The cash conversion cycle is the length of time between the July 20 payment and the November 30 collection, or 133 days (11 days in July, 31 in August, 30 in September, 31 in October, and 30 in November). Answer (A) is incorrect because 92 days is the length of the raw material conversion cycle. Answer (C) is incorrect because 123 days excludes the customer credit period. Answer (D) is incorrect because 153 days includes the vendor's credit period.

25.

= + – .

For this problem we are only interested in the change in the CCC. We may therefore ignore the Payables Deferral Period since it is assumed to remain unchanged.

96. Calculate the total number of expected defaults, assuming no repeat business is on the horizon. (M)A. 795 C. 135B. 201 D. 66

97. Given average revenues from sales of $1,200 and the cost of sales of $1,100, what is the average expected profit or loss from extending credit to slow payers? (M)A. $100 profit. C. $220 loss.

Old CCC (ignore payables) = $12,000,000/$100,000 + $8,000,000/$100,000 = 120 + 80 = 200 days.

New CCC = $9,600,000/$90,000 + $6,400,000/$90,000 = 106.67 + 71.11 = 177.78 days.

Change in CCC = New CCC – Old CCC = 177.78 – 200 = -22.22 days. Round to 22 days shorter.

28 . REQUIRED: The estimated cash flow for the period.DISCUSSION: (D) To determine cash flow for the period, all noncash expenses should be added back to net income. Adding the $5,000 of depreciation and the $6,000 of discount amortization to the $60,000 of net income produces a cash flow of $71,000.Answer (A) is incorrect because the cash flow for the period is greater than net income given noncash expenses in the form of depreciation and bond discount amortization. Answer (B) is incorrect because $66,000 does not reflect noncash expenses for depreciation. Answer (C) is incorrect because the $5,000 of depreciation and the $6,000 for amortization should be added back to, not subtracted from, income.

34 . Answer (A) is correct. Opportunity cost is the forgone benefit, that is, the benefit from the next best alternative use of a resource. The opportunity cost of holding cash balances is calculated by multiplying the average cash balance by the opportunity cost rate (the return on the best alternative use of the cash). The opportunity cost of holding cash balances is higher if the average cash balance is higher. Answer (B) is incorrect because, if other factors are constant, a higher average cash balance requires fewer replenishing transactions. Hence, the total transactions costs associated with obtaining cash balances for the period are lower. Answer (C) is incorrect because the opportunity cost of holding a higher cash balance is higher. Answer (D) is incorrect because the total costs of holding cash balances equal the sum of the opportunity costs and transactions costs. Accordingly, the company should determine the optimal average cash

CMA EXAMINATION QUESTIONS Page 21 of 61

Page 22: P03 - Working Capital Finance

MANAGEMENT ADVISORY SERVICES WORKING CAPITAL FINANCE

B. $164 loss. D. $264 loss.

98. Given revenues from sales of $1,200 and the cost of sales of $1,100, what would the average level of revenues that makes it worthwhile to extend credit to slow payers? (M)A. $1,364.00 C. $1,410.26B. $1,389.74 D. $1,510.26

balance that minimizes total holding costs. Whether total costs are lower when the average cash balance is higher depends on whether the cash balance is below, at, or above the optimal average cash balance for the company.

7 . Answer (B) is correct. If current liabilities are $200 million and the current ratio (current assets ÷ current liabilities) is 2.2, current assets must be $440 million (2.2 x $200 million). If X amount of commercial paper is issued to finance inventory (current assets), thereby increasing both current assets and current liabilities by X, the level of current assets at which the new current ratio will be 2.0 is $480 million ($440 million + $40 million of commercial paper).

($440 + X) ÷ ($200 + X) = 2.0 $440 + X = 2($200 + X)

$440 + X = $400 + 2X $440= $400 + X

X = $40Answer (A) is incorrect because $20 million ignores the increase in current assets. This answer would be appropriate if noncurrent assets were being financed. Answer (C) is incorrect because $240 million is the amount of working capital both before and after the issuance of commercial paper. Answer (D) is incorrect because $180 million is a nonsense answer.

40 . REQUIRED: The decision regarding whether to use a lockbox system and the amount of savings or loss.DISCUSSION: (C) Dividing the $90 million of annual sales by 360 days produces daily sales of $250,000. If collection time is reduced by 3 days, then $750,000 (3 x $250,000) will be available for investment. At an 8% rate of return, the additional $750,000 will earn $60,000 annually. This is $20,000 less than the $80,000 cost of the system. The $20,000 loss means the company should not investment in the new system.Answer (A) is incorrect because $140,000 equals the $80,000 cost plus the investment earnings. Answer (B) is incorrect because the $80,000 cost should be considered. Answer (D) is incorrect because the earnings are $60,000.

45 . REQUIRED: The optimal fee structure for a lockbox system.

SHORT-TERM CREDITTrade CreditCash Discount*. On cash discounts, all of the following statements do not apply except (E)

a. If a firm buys P10,000 of goods on terms of 1/10, net 30 and pays within the discount period, the amount paid would be P9,000.

b. The cost of not taking a cash discount is always higher than the cost of a bank loan.c. With trade terms of 2/15, net 60, if the discount is taken the buyer receive 45 days of

credit. RPCPA 0597d. The cost of not taking the discount is higher for terms of 2/10, net 60 than for 2/10, net 30.

DISCUSSION: (D) Multiplying 700 checks times 360 days results in a total of 252,000 checks per year. Accordingly, under (A), total annual cost is $126,000 ($0.50 x 252,000), which is less desirable than the $125,000 flat fee in (B). Given that the annual collections equal $453,600,000 ($1,800 x 700 x 360), (C) is also less desirable because the annual fee would be $136,080 (.03% x $453,600,000). The best option is therefore to maintain a compensating balance of $1,750,000 when the cost of funds is 7%, resulting in a total cost of $122,500 (07 x $1,750,000). Answer (A) is incorrect because the annual cost is $126,000. Answer (B) is incorrect because the annual cost is $125,000. Answer (C) is incorrect because the annual cost is $136,080.

52 . REQUIRED: The amount at which wire transfers are preferable.DISCUSSION: (D) Given a $25 fee and an interest rate of 0.02% per day for 2 says, the breakeven amount is $62,500 [$25 transfer fee + (2 x .02% interest rate)]. Thus, the interest earned on a transfer of any amount greater than $62,500 would exceed the $25 fee.Answer (A) is incorrect because the $25 transfer fee is covered by the interest on $62,500 for 2 days. Answer (B) is incorrect because $125,000 is required if collections are accelerated by only one day. Answer (C) is incorrect because the interest on $173 for 2 days if less than $0.07.

58 . Answer (B) is correct. To determine the amount of interest the lender will earn, the 3.5% discount rate is multiplied by the face amount of the Treasury bill. The interest on this Treasury bill is $3.50 (3.5% x 1 year x $100); thus, the purchase price is $96.50 ($100 - $3.5). Answer (A) is incorrect because option 1 has a purchase price of $97.00. Answer (C) is incorrect because option 3 has a purchase price of $97.33. Answer (D) is incorrect because option 4 has a purchase price of $97.00.

64 . Answer (D) is correct. The expected average accounts receivable balance equals the average collection period times the credit sales per day. Thus, the average accounts receivable balance is 45,205.48 {[(10,000 units sold on credit x 66 price) ÷ 365 days] x 25 days}. The foregoing calculation assumes that receivables are recorded at their gross amounts.

CMA EXAMINATION QUESTIONS Page 22 of 61

Page 23: P03 - Working Capital Finance

MANAGEMENT ADVISORY SERVICES WORKING CAPITAL FINANCE

Free Trade Credit99. Phillips Glass Company buys on terms of 2/15, net 30. It does not take discounts, and it

typically pays 30 days after the invoice date. Net purchases amount to $720,000 per year. On average, how much “free” trade credit does Phillips receive during the year? (Assume a 360-day year.) (E)a. $30,000 d. $60,000b. $40,000 e. $70,000c. $50,000 Brigham

Answer (A) is incorrect because 684.93 is based on annual credit sales of 10,000. Answer (B) is incorrect because 1,808.22 equals the credit sales per day. Answer (C) is incorrect because 27,123.30 is based on a 15-day average collection period.

9 . REQUIRED: The maximum dividends consistent with a specified minimum current ratio.DISCUSSION: (B) Before the dividend, total current assets equal $2,050,000 ($455,000 cash + $900,000 receivables + $650,000 inventory + $45,000 prepaid assets). Current liabilities total $900,000 ($285,000 accrued liabilities+ $550,000 accounts payable + $65,000 current portion of long-term debt). The payment of the cash dividend will not change current liabilities, so a current ratio of 2 to 1 requires that current assets be maintained at a minimum of $1,800,000 (2 x $900,000). Thus, cash can decrease by $250,000 ($2,050,000 – $1,800,000). The maximum per share rate is $2.50 ($250,000 ÷ 100,000 shares).Answer (A) is incorrect because $2.05 fails to include prepaid assets (prepaid expenses) as current assets. Answer (C) is incorrect because $3.35 fails to include the current portion of long-term debt as a current liability. Answer (D) is incorrect because $3.80 fails to consider prepaid assets as a current asset and the current portion of long-term debt as a current liability.

10 . REQUIRED: The effect of plant expansion on working capital.DISCUSSION: (B) Working capital is defined as current assets minus current liabilities. Working capital is calculated as follows:CurrentProposalCash$ 100,000$ 120,000Accounts receivable400,000500,000Inventory380,000480,000Marketable securities 200,000 200,000Total current assets$1,080,000$1,280,000Accounts payable$ 350,000$ 430,000Mortgage payable – current 175,000 325,000Total current liabilities$ 525,000$ 755,000Working capital$ 555,000$ 525,000Working capital would decrease from the current $555,000 to $525,000 under the proposal, a decline of $30,000.Answer (A) is incorrect because working capital would decrease by $30,000. Answers (C) and (D) are incorrect because working capital would decrease

11 . Answer (C) is correct. Working capital is defined as current assets minus current liabilities. The working capital for each possibility is calculated as follows:

Non-free Trade Credit*. Bachoy & Co. buys on terms 2/10, net 30, but generally does not pay until 40 days after the

invoice date. Its purchases total P2,160,000 per year. Assuming 360 days a year, the amount of “non-free” trade credit used by the company on the average each year is (M)a. P180,000 c. P60,000b. P240,000 d. P120,000 RPCPA 0595

CurrentProposalCash$100,000$110,000Accounts receivable560,000690,000Inventories 350,000380,000Marketable securities 200,000 200,000Total current assets$1,210,000$1380,000Less account payable400,000470,000Working capital$810,000$910,000Thus, the impact on working capital would be an increase of $100,000 (from $810,000 to $910,000).Answer (A) is incorrect because working capital increases by $100,000. Answer (B) is incorrect because $950,000 equals the sum of the increases in working capital and fixed assets, minus the increase in net income. Answer (D) is incorrect because $950,000 equals the sum of the increases in working capital and fixed assets, minus the increase in net income.

12 . Answer (B) is correct. Working capital is defined as current assets minus current liabilities. Working capital is calculated as follows: CurrentProposedCash$ 120,000 $ 140,000Accounts receivable400,000 550,000Inventory 360,000 420,000Marketable securities 180,000 180,000 Total current assets$1,060,000 $1,290,000Accounts payable (360,000) (450,000)Mortgage payable - current (160,000) (310,000)Total current liabilities$ 520,000 $ 760,000Working capital $ 540,000 $ 530,000Working capital decreases by $10,000 from the current $540,000 to $530,000 under the proposal. Answer (A) is incorrect because $240,000 is the increase in current liabilities. Answer (C) is incorrect because $230,000 is the proposed increase in current assets. Answer (D) is incorrect because $10,000 is the decrease in working capital.

14 . Answer (A) is correct. Working capital is the excess of current assets over current liabilities. An increase in current assets or a decrease in current liabilities will increase working capital. Option 1 maximizes Mason Company's net working capital, increasing it by $170 ($120 + $50). Answer (B) is incorrect because option 2 increases net working capital by $150. Answer (C) is incorrect because option 3 decreases net working capital by $160. Answer (D) is incorrect because option 4 decreases net working capital by $25.

15.Construct simplified comparative balance sheets and income statements for the restricted and relaxed policies:(In thousands)

CMA EXAMINATION QUESTIONS Page 23 of 61

Page 24: P03 - Working Capital Finance

MANAGEMENT ADVISORY SERVICES WORKING CAPITAL FINANCE

100. Phranklin Pharms Inc. purchases merchandise from a company that gives sales terms of 2/15, net 40. Phranklin Pharms has gross purchases of $800,000 per year. What is the maximum amount of costly trade credit Phranklin could get, assuming they abide by the suppliers credit terms? (Assume a 360-day year.) (M)a. $87,111.20 c. $54,444.50b. $32,666.70 d. $52,266.67 Brigham

Accounts Payable Balance101. Your firm buys on credit terms of 2/10, net 45, and it always pays on Day 45. If you calculate

that this policy effectively costs your firm $157,500 each year, what is the firm’s average accounts payable balance? (M)

15% of Sales Restricted25% of Sales RelaxedCurrent assets$ 60$100Fixed assets100100 Total assets$160$200Debt80100Equity and retained earnings80100 Total liabilities

and equity$160$200EBIT3636Less: Interest (10%)(8)(10)EBT2826Less: Taxes (40%)( 11.2)( 10.4) Net income$ 16.8$ 15.6

ROE = NI/Equity; $16.8/$80 = 0.21; $15.6/$100 = 0.156.ROE (restricted policy) = 21.0%.ROE (relaxed policy) = 15.6%.Difference in ROEs = 0.21 - 0.156 = 0.054 = 5.4%.

16 . Answer (C) is correct. The cash flow cycle begins when the firm pays for merchandise it has purchased and ends when the firm receives cash from the sale of the merchandise. Inventory is held for an average of 60 days prior to sale, but the average age of accounts payable is 30 days. Consequently, the average time between outlay and sale is 30 days. Receivables are collected an average of 45 days after sale, so the length of the cash flow cycle is 75 days (30 + 45). Answer (A) is incorrect because the age of payables should be deducted from the sum of the other items. Answer (B) is incorrect because the payables are not added to the inventory period. They are deducted. Answer (D) is incorrect because 105 days equals the sum of the inventory cycle and the receivables cycle.

17 . REQUIRED: The length of the cash flow cycle.DISCUSSION: (C) The cash flow cycle begins when the firm pays for merchandise it has purchased and ends when it receives cash from the sale of the merchandise. Answer (A) is incorrect because the age of accounts payable was added instead of deducted in the computation of the cash flow cycle. Answer (B) is incorrect because the age of accounts payable was not deducted in the computation of the cash flow cycle. Answer (D) is incorrect because the age of accounts receivable was deducted instead of added and the age of accounts payable was added instead of deducted in the computation.

18.

a. $1,234,000 d. $625,000b. $75,000 e. $750,000c. $157,500 Brigham

Annual Nominal Rate19. The cost of giving up a cash discount under the terms of sale 1/10 net 60 (assume a 360-day

year) is (E)A. 7.2 percent. C. 14.7 percent.B. 6.1 percent. D. 12.2 percent. Gitman

= + – .

= 72 + 60 - 45 = 87 days.

70 . Answer (A) is correct. Credit sales amounted to $540,000 (60% x $900,000). Dividing $540,000 by 4.5 produces an average receivable balance of $120,000. Dividing 365 days by the turnover of 4.5 results in an average collection period of 81 days. Answer (B) is incorrect because the collection period is 81 days. Answer (C) is incorrect because the collection period is 81 days and the average receivables is too low. Answer (D) is incorrect because it includes cash sales as well as credit sales in receivables.

20. Facts given: Payables deferral period = 30 days; Inv = $5,000,000; Rec. = $2,000,000; ADS = $100,000.

= + – .

Step 1: Determine the inventory conversion period:Inventory conversion period = Inventory/Daily sales

= $5,000,000/$100,000= 50 days.

Step 2: Determine the receivables collection period:Receivables collection period = Receivables/Daily sales

= $2,000,000/$100,000= 20 days.

Step 3: Given data and information calculated above, determine the firm’s cash conversion cycle:Cash conversion cycle = 50 + 20 - 30

= 40 days.

21. Old With Change

CMA EXAMINATION QUESTIONS Page 24 of 61

Page 25: P03 - Working Capital Finance

MANAGEMENT ADVISORY SERVICES WORKING CAPITAL FINANCE

20. The cost of giving up a cash discount under the terms of sale 5/20 net 120 (assume a 360-day year) is (E)A. 15 percent. C. 15.8 percent.B. 18.9 percent. D. 20 percent. Gitman

102. Your company has been offered credit terms on its purchases of 4/30, net 90. What will be the nominal cost of trade credit if your company pays on the 35th day after receiving the invoice? (E)

ICP = = = 90 = = 72

+ +

DSO = = = 72 = 63

DP = 30 days - 30 DP - 30 CCC = 132 days New CCC = 105 days

Change in CCC = 105 – 132 = -27 days.Net change is –27 days (CCC is 27 days shorter).

77 . REQUIRED: The dollar impact on accounts receivable of a change in credit policy.DISCUSSION: (C) If sales are $50 million, 70% of which are on credit, total credit sales will be $35 million. The receivables turnover equals 4.8 times per year (360 days ÷ 75-day collection period). Receivables turnover equals net credit sales divided by average receivables. Accordingly, average receivables equal $7,291,667 ($35,0000 ÷ 4.8). Under the new policy, sales will be $47.5 million (95% x $50,000,000), and credit sales will be $28.5 million (60% x $47,500,000). The collection period will be reduced to 50 days, resulting in a turnover of 7.2 times per year (360 ÷ 50). The average receivables balance will therefore be $3,958,333 ($28,500,000 ÷ 7.2), a reduction of $3,333,334 ($7291,667 - $3,958,333).Answers (A) and (B) are incorrect because the decrease will be $3,333,334. Answer (D) is incorrect because receivables will decrease.

23.Calculate each of the three main components of the cash conversion cycle:Inventory Conversion period (ICP):

ICP = = = 72 days.

a. 30% d. 87%b. 300% e. 156%c. 3% Brigham

22. If a company purchases merchandise on terms of 2/10, n/30, the cash discount available is equivalent to what annual nominal rate of interest (assuming a 360-day year)?a. 2% c. 36%b. 24% d. 72% K, W & W

Days sales outstanding (DSO):

DSO = = = 96 days.

Payables deferral period (PDP):

PDP = = = 25 days.

Cash conversion cycle (CCC):CCC = ICP + DSO – PDP = 72 + 96 – 25 = 143 days.

24 .First, calculate Sales/Day = $50,000,000/360 = $138,888.89.Then, calculate the old inventory conversion period:Inventory/Sales per day = $15,000,000/$138,888.89 = 108 days.Then, find the new inventory conversion period:$13,000,000/$138,888.89 = 93.6 days.We have cut the inventory conversion period by 108 – 93.6 = 14.4 days.

Then, calculate the old DSO:Accts. Rec./Sales per day = $10,000,000/$138,888.89 = 72 days.Then, find the new DSO = $8,000,000/$138,888.89 = 57.6 days.We have cut the DSO by 72 – 57.6 = 14.4 days.

Finally, find the total net change = -14.4 + (-14.4) – 10 = -38.8 days.

83 . Answer (A) is correct. If 80% of the $20,000,000 of sales are on credit, $16,000,000 of sales will be subject to the discount. If discounts are taken on 60% of credit sales, the expected discount will be $288,000 [3% x (60% x $16,000,000)]. Answer (B) is incorrect because $480,000 is the potential discount on all credit sales. Answer (C) is incorrect because $360,000 assumes all sales are on credit and that 60% of discounts are taken. Answer (D) is incorrect because $600,000 assumes all sales are on credit and that 100% of the discounts are taken.

CMA EXAMINATION QUESTIONS Page 25 of 61

Page 26: P03 - Working Capital Finance

MANAGEMENT ADVISORY SERVICES WORKING CAPITAL FINANCE

103. When a company offers credit terms of 2/10, net 30, the annual interest cost, based on a 360-day year, is (E)A. 24.0%. C. 36.0%.B. 35.3%. D. 36.7%. CMA 0691 1-6, 0697 1-8

104. When a company offers credit terms of 3/10, net 30, the annual interest cost based on a 360-day year is (E)A. 36.7% C. 37.1%B. 24.5% D. 55.6% Gleim

89. Credit policy and ROE Answer: c Diff: M RUse the DSO formula to calculate accounts receivable under the new policy as 36 = AR/($730,000/365) or AR = $72,000. Thus, $125,000 - $72,000 = $53,000 is the cash freed up by reducing DSO to 36 days. Retiring $53,000 of long-term debt leaves $247,000 in long-term debt. Given a 10% interest rate, interest expense is now $247,000(0.1) = $24,700. Thus, EBT = EBIT - Interest = $70,000 - $24,700 = $45,300. Net income is $45,300(1 - 0.3) = $31,710. Thus, ROE = $31,710/$200,000 = 15.86%.

26 .ICP = 360 days/($12 million/$1 million) = 30 days.DSO = 2.0 ICP = 60 days.

Solve for accounts receivable:DSO = 60 = Receivable accounts/Sales per day = (A/R)/($12/360) = $2 million.

Calculate new ICP, change in CCC, and new DSO required to meet goal:New ICP = 360/($12/0.9) = 360/13.333 = 27 days.Net change in ICP = -3 days.Total change in CCC required = 10 days.Reduction in DSO needed = 10 – 3 = 7 days.New DSO required = 60 – 7 = 53 days.

Solve for new receivables level:DSO = 53 = [(A/R)/($12,000,000/360)]A/R = 53 $33,333.33 = $1,766,667.Old A/R = $2,000,000. New A/R = $1,766,667.Reduction required in A/R = $2,000,000 - $1,766,667 = $233,333.

27 . Answer (D) is correct. To determine cash flow for the period, all noncash expenses should be added to income. Adding the $6,000 of depreciation and the $7,000 of discount

105. If a firm's credit terms require payment within 45 days but allow a discount of 2% if paid within 15 days (using a 360-day year), the approximate cost or benefit of the trade credit terms is (E)A. 2%. C. 48%.B. 16%. D. 24%. CMA 0692 1-23

*. If a firm purchases raw materials from its suppliers on a 2/10, n/60 cash discount basis, the equivalent annual interest rate (using a 360-day year) of foregoing the cash discount and making payment on the 60th day is (E)

amortization to $50,000 of income produces a pretax cash flow of $63,000. Interest is not added back to income because it requires a cash payment. Answer (A) is incorrect because the cash flow for the period is greater than income given noncash expenses in the form of depreciation and bond discount amortization. Answer (B) is incorrect because $57,000 does not reflect the noncash expense for depreciation. Answer (C) is incorrect because the $6,000 of depreciation and the $7,000 for amortization should be added to, not subtracted from, income.

29 . Answer (C) is correct. Cash inflows for March would consist of 50% of March credit sales (50% x $90 = $45), plus 30% of February credit sales (30% x $120 = $36), plus 20% of January credit sales (20% x $100 = $20), plus cash sales for March of $18. Consequently, total collections equal $119,000. Answer (A) is incorrect because $138,000 equals the sum of February credit sales and March cash sales. Answer (B) is incorrect because $122,000 equals 50% of January credit sales, 30% of February credit sales, 20% of March credit sales, and 100% of March cash sales. Answer (D) is incorrect because $108,000 is the total sales for March, not the total cash collections for March.

30 . Answer (A) is correct. The float period is the time between when a check is written and when it clears the payor's checking account. Check float results in an interest-free loan to the payor because of the delay between payment by check and its deduction from the bank account. If checks written require one more day to clear than checks received, the net float equals one day's receipts. The company will have free use of the money for one day. In this case, the amount is $10,000. Answer (B) is incorrect because the company enjoys one day's net float because its checks clear more slowly than its deposits. Answer (C) is incorrect because the net float is positive. The company can write checks (up to $10,000) even when it has no money because the checks do not clear until a day after deposits clear. Answer (D) is incorrect because the net float represents the difference between when deposits clear and when disbursements clear.

31 . Answer (A) is correct. The difference between collections and payables is $5,000 daily. Five days' worth amounts to $25,000 of float.

CMA EXAMINATION QUESTIONS Page 26 of 61

Page 27: P03 - Working Capital Finance

MANAGEMENT ADVISORY SERVICES WORKING CAPITAL FINANCE

a. 36.7% c. 73.5%b. 14.7% d. 12.2% RPCPA 0596

106. If a firm purchases raw materials from its supplier on a 2/10, net 40, cash discount basis, the equivalent annual interest rate (using a 360-day year) of forgoing the cash discount and making payment on the 40th day is (E)A. 2% C. 24.49%B. 18.36% D. 36.72% Gleim

107. Using a 360-day year, what is the opportunity cost to a buyer of not accepting terms 3/10, net 45? (E)A. 55.67% C. 22.27%B. 31.81% D. 101.73% CMA 0694 1-20, 1294 1-18

*. Mamimili, Inc. purchased an item on credit with terms of 3/10, n/45. Based on a 360-day year, the company’s annual interest cost of foregoing the cash discount and making payment on the last day of the credit period is (E)a. 24.00% c. 24.74%b. 31.81% d. 30.86% RPCPA 1095, 1096

Answer (B) is incorrect because $40,000 ignores the impact of payables and the five-day float period. Answer (C) is incorrect because $175,000 ignores the impact of receivables. Answer (D) is incorrect because $200,000 ignores the impact of payables.

32 . Positive disbursement float = $15,000(5) = $75,000.Negative collections float = $17,000(3) = $51,000.Net float = $75,000 - $51,000 = $24,000.

33 . Answer (B) is correct. The opportunity cost of keeping a cash balance is calculated by finding the amount of income that could be earned minus any expenses necessary to receive the income. In this example, the amount of daily income that could be earned is $400 ($2,000,000 x .0002). The cost of investing the money, $50, is then subtracted from the $400 to calculate an opportunity cost of $350. Answer (A) is incorrect because $50 is the average transaction cost of investing money overnight. Answer (C) is incorrect because the cost of investing the money has to be subtracted from the amount of daily income that can be earned to get the opportunity cost. Answer (D) is incorrect because $40,000 is the result of multiplying $2 million by 0.02 (not 0.02%, which is 0.0002).

108. Richardson Supply has a $100 invoice with payment terms of 2/10, net 60. Richardson can either take the discount or place the funds in a money market account paying 6% interest. Using a 360-day year, Richardson's cost of not taking the cash discount is (M)A. 12.2%. C. 6.4%.B. 8.7%. D. 6.2%. CMA 0687 1-28

109. What effective interest rate is charged to a purchaser receiving terms of 5/10, net 60 if the purchaser fails to take the discount and pays in 60 days? (E)A. 30.64% C. 38.40%B. 36.60% D. 45.39% Gleim

110. Dixie Tours Inc. buys on terms of 2/15, net 30. It does not take discounts, and it typically pays 35 days after the invoice date. Net purchases amount to $720,000 per year. What is the nominal cost of its non-free trade credit? (E)a. 17.2% d. 36.7%b. 23.6% e. 50.6%c. 26.1% Brigham

111. A firm is offered trade credit terms of 3/15, net 45. The firm does not take the discount, and it pays after 67 days. What is the nominal annual cost of not taking the discount? (E)a. 21.41% d. 23.48%b. 22.07% e. 24.52%c. 22.95% Brigham

*. The official terms of purchases of U Tang & Co. are 2/10, net 30 but generally the company does not pay until 40 days after the invoice date. Its purchases total P3,600,000 per year. Assuming 360 days a year, the approximate cost of the “non-free trade credit amounts to (M)a. 18.36% c. 21.90%b. 24.50% d. 19.40% RPCPA 1095

46. If a firm purchases raw materials from its supplier on a 3/10, n/50 term, the approximate annual interest rate (using 360-day year) of giving up a cash discount and making payment on the 60th day isa. 22.27. percent c. 18.37 percentb. 27.84 percent d. 14.69 percent Pol Bobadilla

*. Three suppliers of Ma Corp. offer different credit terms. Core Co. offers terms of 1½ /15, net 30. Doug Corp. offers terms of 1/10, net 30. Ernst Inc. offers terms of 2/10, net 60. Ma Corp. would have to borrow from a bank at an annual rate of 12% in order to take any cash discounts. Which one of the following would be the most attractive for Ma Corp. (Assume 360 days a year.) (M)

CMA EXAMINATION QUESTIONS Page 27 of 61

Page 28: P03 - Working Capital Finance

MANAGEMENT ADVISORY SERVICES WORKING CAPITAL FINANCE

a. Purchase from Core Co., pay in 15 days and borrow any money needed from the bank.b. Purchase from Core Co. pay in 30 days and borrow any money needed from the bank.c. Purchase from Ernst Inc., pay in 60 days and borrow any money needed from the bank.d. Purchase from Doug Corp. and pay in 30 days. RPCPA 0595

*. Software Center, Inc.’s new controller is reviewing the company’s cash management. Below are relevant information regarding trade credits from the suppliers of the company:

Suppliers Average Monthly Purchases Credit TermsTech Co. P 100,000 Net 30

Computech 300,000 2/10, n/30Compuworks

1,000,000 5/10, n/120

So-wares 600,000 3/10, n/45The company uses a 360-day year. Assume that all of the suppliers can supply any and all of the requirements of Software and can provide unlimited credit line to the company and that the company can have only one supplier. With a cost of bank borrowing of 18% per annum, which supplier should Software choose? (D)a. Compuworks due to the longest credit term of 120 days.b. Computech due to cost of trade credit of 36.7%.c. Compuworks due to the highest trade discount at 5%.d. Tech Co. due to no discount policy. RPCPA 1096

Effective Annual Rate7. Suppose you purchase goods on terms of 2/10, net 50. Taking compounding into account,

what annual rate of interest is implied by the cash discount? (Assume a year has 365 days.) A. 2% C. 102%B. 20% D. 18% B & M

6. Suppose you purchase goods on terms of 5/10, net 30. Taking compounding into account, what annual rate of interest is implied by the cash discount? (Assume a year has 365 days.) A. 155% C. 91%B. 5% D. 255% B & M

5. Supposing you purchase goods on terms of 10/20, net 60. Taking compounding into account, what annual rate of interest is implied by the cash discount? (Assume a year has 365 days.) A. 91% C. 250%B. 139% D. 162% B & M

112. Suppose the credit terms offered to your firm by your suppliers are 2/10, net 30 days. Out of convenience, your firm is not taking discounts, but is paying after 20 days, instead of waiting until Day 30. You point out that the nominal cost of not taking the discount and paying on Day 30 is around 37 percent. But since your firm is not taking discounts and is paying on Day 20, what is the effective annual cost of your firm’s current practice, using a 360-day year? (M)a. 36.7% d. 43.6%b. 105.4% e. 106.9%c. 73.4% Brigham

47. If a firm purchases raw materials from its suppliers on a 2/10, n/50 term, the equivalent annual effective interest (using 360-day year) of continuously giving up a cash discount and making payment on the 50th day is

CMA EXAMINATION QUESTIONS Page 28 of 61

Page 29: P03 - Working Capital Finance

MANAGEMENT ADVISORY SERVICES WORKING CAPITAL FINANCE

A. 14%. C. 12.29%B. 19.94% D. 14.69% Pol Bobadilla

113. Hayes Hypermarket purchases $5 million in goods over a 1-year period from its sole supplier. The supplier offers trade credit under the following terms: 2/15 net 45. If Hayes chooses to pay on time but not to take the discount, what is the average level of the company’s accounts payable, and what is the effective cost of its trade credit? (Assume a 360-day year.) (M)a. $208,333; 17.81% d. $625,000; 17.54%b. $416,667; 17.54% e. $625,000; 27.43%c. $416,667; 27.43% Brigham

Weighted Average Annual Nominal Interest RateQuestions 112 and 113 are based on the following situation. CMA 1296 1-10 & 11CyberAge Outlet, a relatively new store, is a café that offers customers the opportunity to browse the internet or play computer games at their tables while they drink and dine. The customer pays a fee based on the amount of time spent signed on to the computer. The store also sells books, tee-shirts, and computer accessories. CyberAge paying all of is bills on the last day of the payment period, thus forfeiting all supplier discounts. Shown below are data on CyberAge’s two major vendors, including average monthly purchases and credit terms.

Vendor Average Monthly Purchases Credit TermsWebmaster $25,000 2/10, net 30Softidee 50,000 5/10, net 90

114. Assuming a 360-day year and the CyberAge continues paying on the last day of the credit period, the company’s weighted-average annual interest rate on trade credit (ignoring the effects of compounding) on these two vendors is (D)a. 27.0% c. 28.0%b. 25.2% d. 30.2%

115. Should CyberAge use trade credit and continue paying at the end of the credit period? (D)a. Yes, if the cost of alternative short-term financing is less.b. Yes, if the firm’s weighted-average cost of capital is equal to its weighted-average cost of

trade credit.c. No, if the cost of alternative long-term financing is greater.d. Yes, if the cost of alternative short-term financing is greater.

BANK LOANSPrincipal AmountDiscounted Loan116. Picard Orchards requires a $100,000 annual loan in order to pay laborers to tend and harvest

its fruit crop. Picard borrows on a discount interest basis at a nominal annual rate of 11

percent. If Picard must actually receive $100,000 net proceeds to finance its crop, then what must be the face value of the note? (E)a. $111,000 d. $ 89,000b. $100,000 e. $108,840c. $112,360 Brigham

117. Viking Farms harvests crops in roughly 90-day cycles based on a 360-day year. The firm receives payment from its harvests sometime after shipment. Due in part to the firm’s rapid growth, it has been borrowing to finance its harvests using 90-day bank notes on which the firm pays 12 percent discount interest. If the firm requires $60,000 in proceeds from each note, what must be the face value of each note? (E)a. $61,856 d. $68,182b. $67,531 e. $67,423c. $60,000 Brigham

Interest on Loan with Compensating Balance*. The Manila Commercial Bank and Bank Rap Corp. signed a loan agreement subject to the

following terms:a. Stated interest rate of 18% on a one-year loan; andb. 15% compensating non-interest bearing checking account balance to be maintained

by Bank Rap with Manila Commercial Bank.The net proceeds of the loan was P1 million. The principal amount of the loans was (M)a. P1,176,471 c. P1,492,537b. P1,000,000 d. P1,219,512. RPCPA 0595 adapted

Discounted Loan with Compensating Balance*. The Manila Commercial Bank and Bank Rap Corp. signed a loan agreement subject to the

following terms:a. Stated interest rate of 18% on a one-year discounted loan; andb. 15% compensating non-interest bearing checking account balance to be maintained

by Bank Rap with Manila Commercial Bank.The net proceeds of the loan was P1 million. The principal amount of the loans was (M)a. P1,176,471 c. P1,492,537b. P1,000,000 d. P1,219,512. RPCPA 0595

Annual Interest Rate of One-year LoanLIBOR-based Interest Rate29. The interest rate on a loan is set at "1% over LIBOR." If the LIBOR rate is 5% then the interest

rate on the loan is: (E)A. 5% C. 6%B. 4% D. 7% B & M

CMA EXAMINATION QUESTIONS Page 29 of 61

Page 30: P03 - Working Capital Finance

MANAGEMENT ADVISORY SERVICES WORKING CAPITAL FINANCE

Simple Interest118 A one year, $20,000 loan with a 10% nominal interest rate provides the borrower with the use

of <List A> if interest is charged on a <List B> basis. (E)CIA 0595 IV-50 A. B. C. D.List A $18,000 $20,000 $20,000 $22,000List B Simple Simple Discount Discount

Annual Nominal Rate of Add-on Loan119. Coverall Carpets Inc. is planning to borrow $12,000 from the bank. The bank offers the choice

of a 12 percent discount interest loan or a 10.19 percent add-on, 1-year installment loan, payable in 4 equal quarterly payments. What is the approximate (nominal) rate of interest on the 10.19 percent add-on loan? (E)a. 5.10% d. 20.38%b. 10.19% e. 30.57%c. 12.00% Brigham

Questions 1 & 2 are based on the following information. BrighamYou have just taken out a loan for $75,000. The stated (simple) interest rate on this loan is 10 percent, and the bank requires you to maintain a compensating balance equal to 15 percent of the initial face amount of the loan. You currently have $20,000 in your checking account, and you plan to maintain this balance. The loan is an add-on installment loan that you will repay in 12 equal monthly installments, beginning at the end of the first month.

120. How large are your monthly payments? (E)a. $6,250 d. $5,250b. $7,000 e. $6,875c. $7,500

121. What is the nominal annual add-on interest rate on this loan?a. 10.00% d. 20.00% (E)b. 16.47% e. 24.00%c. 18.83%

Effective Interest Rate of a Simple Interest with Compensating Balance 122. If a firm borrows $500,000 at 10% and is required to maintain $50,000 as a minimum

compensating balance at the bank, what is the effective interest rate on the loan? (E)A. 10.0% C. 9.1%B. 11.1% D. 12.2% CMA 1294 1-23

27. The bank offers you a term loan at 10% on condition that you maintain a 10% compensating balance. What is the effective rate of interest? A. 9.0% C. 13.7%B. 10.0% D. 11.1% B & M

36. A bank lends a firm $500,000 for one year at 8 percent and requires compensating balances of 10 percent of the face value of the loan. The effective annual interest rate associated with this loan isA. 8.9 percent. C. 7.2 percent.B. 8 percent. D. 7.0 percent. Gitman

123. FDR Corporation needs $5 million, and the company's bank has offered to make a loan at 10% annually, provided the company maintains a 15% compensating balance. What is the effective cost of the loan? (E)A. 10% C. 16.67%B. 11.76% D. 17.65% Gleim

124. On January 1, Scott Corporation received a $300,000 line of credit at an interest rate of 12% from Main Street Bank and drew down the entire amount on February 1. The line of credit agreement requires that an amount equal to 15% of the loan be deposited into a compensating balance account. What is the effective annual cost of credit for this loan arrangement? (E)A. 11.00% C. 12.94%B. 12.00% D. 14.12% CMA Samp Q1-7

125. Hagar Company’s bank requires a compensating balance of 20% on a $100,000 loan. If the stated interest on the loan is 7%, what is the effective cost of the loan?(E)a. 5.83% c. 8.40%b. 7.00% d. 8.75% CMA 0697 1-19

26. The bank offers you a term loan at 8% on condition that you maintain a 20% compensating balance. What is the effective rate of interest? A. 6.4% C. 18.0%B. 10.0% D. 8.0% B & M

28. The bank offers you a term loan at 11% on condition that you maintain a 20% compensating balance. What is the effective rate of interest? A. 13.8% C. 9.7%B. 11.0% D. 8.8% B & M

126. The Dixon Corporation has an outstanding 1 year bank loan of $300,000 at a total interest rate of 8%. In addition, Dixon is required to maintain a 20% compensating balance in its checking

CMA EXAMINATION QUESTIONS Page 30 of 61

Page 31: P03 - Working Capital Finance

MANAGEMENT ADVISORY SERVICES WORKING CAPITAL FINANCE

account, Assuming the company would normally maintain zero balance in its checking account, the effective interest rate on the loan is (E)a. 6.4% c. 20.0%b. 8.0% d. 10.0% CMA 1295 1-10

*. The HIJ Company has an outstanding bank loan of P400,000 at an interest rate of 12%. The company is required to maintain a 20% compensating balance in its checking account. What is the effective interest cost of the loan? Assume that the company would not normally maintain this average amount? (E)A. 8%. C. 12%B. 15%. D. 10% RPCPA 1001

Effective Interest Rate of a Simple Interest with Interest-earning Compensating Balance21. On January 1, 2001, Olin Company borrows $2,000,000 from National Bank at 12% annual

interest. In addition, Olin is required to keep a compensatory balance of $200,000 on deposit at National Bank which will earn interest at 4%. The effective interest that Olin pays on its $2,000,000 loan isa. 10.0%. c. 12.0%.b. 11.6%. d. 12.8%. K, W & W

127. A company obtained a short-term bank loan of $250,000 at an annual interest rate of 6%. As a condition of the loan, the company is required to maintain a compensating balance of $50,000 in its checking account. The company’s checking account earns interest at an annual rate of 2%. Ordinarily, the company maintains a balance of $25,000 in its checking account for transaction purposes. What is the effective interest rate of the loan? (D)a. 6.44% c. 5.80%b. 7.00% d. 6.66% CMA 0696 1-11, 0697 1-19

128. A company obtained a short-term bank loan of $500,000 at an annual interest rate of 8%. As a condition of the loan, the company is required to maintain a compensating balance of $100,000 in its checking account. The checking account earns interest at an annual rate of 3%. Ordinarily, the company maintains a balance of $50,000 in its account for transaction purposes. What is the effective interest rate of the loan? (D)a. 7.77% c. 9.25%b. 8.22% d. 8.56% CMA 0694 1-21

*. Bal and Subas obtained a short-term bank loan for P1 million at an annual interest of 12%. As a condition of the loan, the company is required to maintain a compensating balance of P200,000 in its savings account which earns interest at an annual rate of 6%. The company would otherwise maintain only P100,000 in the savings account for transactional purposes. The effective cost of the loan is (D)

a. 13.20% c. 12%b. 12.67% d. 13.5% RPCPA 1095

Effective Interest Rate of Discount129. Elan Corporation is considering borrowing $100,000 from a bank for 1 year at a stated interest

rate of 9%. What is the effective interest rate to Elan if this borrowing is in the form of a discounted note? (E)A. 8.10% C. 9.81%B. 9.00% D. 9.89% CMA 1295 1-11

130. A company has just borrowed $2 million from a bank. The stated rate of interest is 10%. If the loan is discounted and is repayable in one year, the effective rate on the loan is approximately (E)A. 8.89% C. 10.00%B. 9.09% D. 11.11% Gleim

131. Cincy Corp. is borrowing $1 million at 10% for a year on a discount basis with a bank. How much in funds are available for use and what is the effective cost of interest? (E)A. $1,000,000; 10.0% C. $900,000; 10.0%B. $1,000,000; 11.1% D. $900,000; 11.1% Gleim

132. If a company borrows $30,000 from a bank for one year at a stated annual interest rate of 11%, but interest is prepaid (a discounted loan), then what is the effective annual interest rate? (E)A. 9.64% C. 12.36%B. 10.00% D. 11.00% Gleim

42. A firm arranges a discount loan at a 12 percent interest rate, and borrows $100,000 for one year. The stated interest rate is _____ and the effective interest rate is _____.A. 12.00 %; 12.00% C. 12.00%; 13.64%B. 13.64%; 12.00% D. 12.00%; 10.71% Gitman

Effective Annual Percentage Rate on a Discounted Rate with Compensating Balance133. An enterprise borrows funds from its bank for a one-year period. The bank charges interest at

a nominal rate of 15% per annum, on a discount basis, and requires a 10% compensating balance. The effective annual interest rate on the loan is A. 16.67% C. 20.00%B. 17.65% D. 25.00% CIA 1195 IV-52

CMA EXAMINATION QUESTIONS Page 31 of 61

Page 32: P03 - Working Capital Finance

MANAGEMENT ADVISORY SERVICES WORKING CAPITAL FINANCE

134. The Altmane Corporation was recently quoted terms on a commercial bank loan of 7% discounted interest with a 20% compensating balance. The term of the loan is 1 year. The effective cost of borrowing is (rounded to the nearest hundredth) (M)A. 8.75%. C. 7.53%.B. 9.41%. D. 9.59%. CMA 1289 1-21

*. Cashy Co. got a recent quote on a commercial bank loan of 16% discounted rate with a 20% compensating balance. The term of the loan is one year. The effective cost of borrowing is (M)

35 . Answer (A) is correct. The opportunity cost of holding cash balances for the year equals the average cash balance multiplied by the opportunity cost rate, or $2,091.65 ($20,916.50 x 10%). Answer (B) is incorrect because $4,183.30 results if twice the average cash balance is used in calculating the opportunity cost. Answer (C) is incorrect because $8,750.00 results if half the total cash requirement for the year is used in calculating the opportunity cost. Answer (D) is incorrect because $17,500.00 results if the total cash required for the year is used in calculating the opportunity cost.

36 . Answer (C) is correct. Checks are currently tied up for 10½ days (5 for mailing, 4 for processing, and 1½ for clearing). If that were reduced to 4 days, CMR's cash balance would increase by $650,000 (6½ days x $100,000 per day). Answer (A) is incorrect because CMR's cash balance would increase by $650,000 [(10½ days - 4 days) x $100,000 per day]. Answer (B) is incorrect because $400,000 is arrived at by multiplying $100,000 per day collection by the 4-day processing and clearing time with the lockbox system. The $100,000 per day collection should have been multiplied by the 6½ day decrease (10½ days - 4 days) in check processing and clearing. Answer (D) is incorrect because CMR's cash balance would increase by $650,000 [(10½ days - 4 days) x $100,000 per day].

37 . Answer (A) is correct. Checks are currently tied up for 11 days (6 for mailing, 3 for processing, and 2 for clearing). If that period were reduced to 3 days, DLF's cash balance would increase by $1,200,000 (8 days x $150,000 per day). Answer (B) is incorrect because the decrease is 8 days, not 5. Answer (C) is incorrect because $600,000 represents only a 4-day savings. Answer (D) is incorrect because the lockbox system will result in an additional 8 days of savings, not 3.

38 . Answer (D) is correct. The first step in finding the expected annual savings is multiplying the average number of checks collected daily by the average amount of each check. This is $100,000 (200 x $500). The $100,000 is multiplied by the number of days it saves for mailing, 2.0, and processing, 0.5. These numbers are added together to get $250,000 [($100,000 x 2) + ($100,000 x 0.5)]. This number multiplied by the annual interest rate will give expected

a. 19.05% c. 22.85%b. 20.00% d. 25.00% RPCPA 0596

135. The Flesher Corporation was recently quoted terms on a commercial bank loan of 6% discounted interest with a 22% compensating balance. The term of the loan is 1 year. The effective cost of borrowing is (rounded to the nearest hundredth) (M)A. 6.00% C. 7.69%B. 6.38% D. 8.33% Gleim

34. A bank lends a firm $1,000,000 for one year at 12 percent on a discounted basis and requires compensating balances of 10 percent of the face value of the loan. The effective annual

annual savings of $15,000 ($250,000 x 0.06). Answer (A) is incorrect because $250,000 has not been multiplied by the annual interest rate of 6%. Answer (B) is incorrect because $12,000 is the result of leaving out the processing portion of the savings, calculated as $100,000 x 0.5 (and then later multiplied by 6%). Answer (C) is incorrect because $6,000 is the result of multiplying 200 checks per day by their average amount, and then multiplying this total of $100,000 by the annual interest rate. Mailing and processing time saved are thus unaccounted for in this case.

39 . Answer (C) is correct. Because collections will be accelerated by 3 days at a rate of $300,000 per day, the company will have an additional $900,000 to invest. At a rate of 6%, the interest earned will be $54,000 per year. However, the bank will charge $24,000 (12 months x $2,000 per month) for its services. Thus, the firm will increase its income by $30,000 ($54,000 - $24,000). Answer (A) is incorrect because ($24,000) ignores the additional interest revenue from investing the increased funds. Answer (B) is incorrect because $12,000 is based on 2 days of accelerated inflows rather than 3. Answer (D) is incorrect because $54,000 ignores the $24,000 bank service charge.

41

?.Calculate the net reduction in A/R:Current A/R = $2,500,000. New A/R with 20% reduction:$2,500,000 - 0.20($2,500,000) = $2,000,000.Net reduction in A/R = $500,000.Calculate the interest savings and net savings:Interest savings = $500,000(0.11) = $55,000.Net savings = Interest savings - Annual lockbox cost = $55,000 - $15,000 = $40,000.

42 . REQUIRED: The additional annual income or loss from using a lockbox service.CMA EXAMINATION QUESTIONS Page 32 of 61

Page 33: P03 - Working Capital Finance

MANAGEMENT ADVISORY SERVICES WORKING CAPITAL FINANCE

interest rate associated with this loan isA. 12 percent. C. 13.6 percent.B. 13.3 percent. D. 15.4 percent. Gitman

Annual Interest Rate of Less Than One Year LoanEffective Annual Interest Rate on Monthly Payment136. What is the effective annual interest rate on a 9% APR automobile loan that has monthly

payments? (M)A. 9% C. 9.81%B. 9.38% D. 10.94% Gleim

137. A credit card account that charges interest at the rate of 1.25% per month would have an annually compounded rate of and an APR of (M)A. 16.08%; 15% C. 12.68%; 15%B. 14.55%; 16.08% D. 15%; 14.55% Gleim

DISCUSSION: (B) If daily cash receipts are $150,000 and the lockbox service will speed collections by four days, the company will have available an additional $600,000 (4 days x $150,000). The result is increased interest revenue of $24,000 (4% interest rate x $600,000). However, the $2,500 monthly service charges total $30,000 for the year, which is a $6,000 net loss ($24,000 interest revenue – $30,000 monthly service charges).Answers (A), and (C) are incorrect because a loss would result since the service charges exceed the additional interest revenue. Answer (D) is incorrect because the loss is $6,000, the difference between the $24,000 of additional interest revenue and the $30,000 total service charges.

43 . Answer (B) is correct. The firm will begin earning interest 2.2 days (1.3 + .9) earlier with the lockbox plan. Multiplying 325 daily accounts by $1,250 results in average daily collections of $406,250. Thus, the gross savings is $188 ($406,250 x 2.2 x .00021). The cost is $97.50 (325 accounts x $.30). The net savings is $90.50 per day ($188 - $97.50). Multiplying 250 days by $90.50 = $22,675. Answer (A) is incorrect because $3,273 results from ignoring the savings in processing time. Answer (C) is incorrect because $23,500 is half the gross savings. Answer (D) is incorrect because $47,000 results from overlooking the bank charges of $.30 per account.

44 . Answer (B) is correct. If payments are collected 2 days earlier, the company can earn $120,000 ($20,000 x 50 payments per day x 2 days x .06) at a cost of $59,125 [$50,000 + (50 payments x 365 days x $.50)], a gain of $60,875. Answer (A) is incorrect because $59,125 is the annual lockbox cost. Answer (C) is incorrect because $50,000 is the annual fixed fee. Answer (D) is incorrect because $120,000 is the annual savings without regard to costs.

Effective Interest Rate of a Simple Interest with Interest-earning Compensating Balance138. On July 1, 2001, Dichter Company obtained a $2,000,000 180-day bank loan at an annual rate

of 12%. The loan agreement requires Dichter to maintain a $400,000 compensating balance in its checking account at the lending bank. Dichter would otherwise maintain a balance of only $200,000 in this account. The checking account earns interest at an annual rate of 6%. Based on a 360-day year, the effective interest rate on the borrowing is (D)a. 12%. c. 13.33%.b. 12.67% d. 13.5% AICPA 0587 I-32

Effective Interest Rate of a Discount44. A firm has a line of credit and borrows $25,000 at 9 percent interest for 180 days or half a

year. What is the effective rate of interest on this loan if the interest is paid in advance?A. 4.7 percent C. 9.9 percentB. 9.4 percent D. 10.3 percent Gitman

Effective Interest Rate of a Discounted Rate with Transaction Cost139. Corbin, Inc. can issue 3-month commercial paper with a face value of $1,000,000 for

$980,000. Transaction costs will be $1,200. The effective annualized percentage cost of the financing, based on a 360-day year, will be (D)A. 8.48%. C. 8.00%.B. 8.66%. D. 2.00%. CMA 1290 1-28

140. Randy, Inc. can issue 3-month commercial paper with a face value of $1,500,000 for $1,450,000. Transaction costs will be $1,500. The effective annualized percentage cost of the financing, based on a 360-day year, will be (D)A. 3.45% C. 14.22%B. 3.56% D. 13.79% Gleim

Effective Interest Rate of a Discount with Compensating Balance124.Bye Company borrows from a bank a certain loan at a stated discount rate of 12% per annum.

The bank requires 10% of loan as compensating balance in its new checking account. The loan is payable at the end of 6 months. The effective interest rate of this loan isA. 28.21% C. 27.27%B. 14.29% D. 15.38% Pol Bobadilla

Revolving CreditEffective Rate of Simple Interest with Interest and Fee141. Inland Oil arranged a $10,000,000 revolving credit agreement with a group of small banks. The

firm paid an annual commitment fee of one-half of one percent of the unused balance of the loan commitment. On the used portion of the loan, Inland paid 1.5 percent above prime for the

CMA EXAMINATION QUESTIONS Page 33 of 61

Page 34: P03 - Working Capital Finance

MANAGEMENT ADVISORY SERVICES WORKING CAPITAL FINANCE

funds actually borrowed on an annual, simple interest basis. The prime rate was at 9 percent for the year. If Inland borrowed $6,000,000 immediately after the agreement was signed and repaid the loan at the end of one year, what was the total dollar cost of the loan agreement for one year? (E)a. $560,000 d. $900,000b. $650,000 e. $675,000c. $540,000 Brigham

Effective Rate of Simple Interest with Compensating Balance Requirement142. JWDavis.com has a revolving credit agreement with a bank. JWDavis.com can borrow up to

$2 million at 10% interest and is required to keep a 15% compensating balance on all borrowed funds. If the firm borrows $1.2 million for the entire year, what is the effective cost of borrowing? (M)A. 10.00% C. 11.76%B. 11.11% D. 13.33% Gleim

Interest Payment23. If the interest rate on a line of credit is 12% per year, what is the quarterly interest payment on

a loan of $10 million? A. $1,200,000 C. $400,000B. $300,000 D. None of the above B & M

143. The treasury analyst for Garth Manufacturing has estimated the cash flows for the first half of next year (ignoring any short-term borrowings) as follows:

Cash (millions)Inflows Outflows

January $2 $1February 2 4March 2 5April 2 3May 4 2June 5 3

Garth has a line of credit of up to $4 million on which it pays interest monthly at a rate of 1% of the amount loaned. Garth is expected to have a cash balance of $2 million on January 1 and no amount utilized on its line of credit, approximately how much will Garth pay in interest during the first half of the year? (M)a. Zero c. $80,000b. $61,000 d. $132,000 CMA 0697 1-15

Interest Payment43. XYZ Corporation borrowed $100,000 for six months from the bank. The rate is prime plus 2

percent. The prime rate was 8.5 percent at the beginning of the loan and changed to 9 percent after two months. This was the only change. How much interest must XYZ corporation pay? (M)A. $2,476 C. $18,212B. $5,417 D. $21,500 Gitman

45. A firm arranged for a 120-day bank loan at an annual rate of interest of 10 percent. If the loan is for $100,000, how much interest in dollars will the firm pay? (Assume a 360-day year.) (E)A. $10,000 C. $3,333B. $30,000 D. $1,000 Gitman

Commercial PaperAnnual Interest Rate54. A firm issued $2 million worth of commercial paper that has a 90-day maturity and sells for

$1,900,000. The annual interest rate on the issue of commercial paper is (E)A. 5 percent. C. 17 percent.B. 10 percent. D. 21 percent. Gitman

Maturity Value55. A firm has directly placed an issue of commercial paper that has a maturity of 60 days. The

issue sold for $980,000 and has an annual interest rate of 12.24 percent. The value of the commercial paper at maturity is (E)A. $19,992. C. $999,992.B. $980,000. D. $960,008. Gitman

FINANCING ALTERNATIVESDifferent Trade Credit Options144. Merkle, Inc. has a temporary need for funds. Management is trying to decide between not

taking discounts from one of their three biggest suppliers, or a 14.75% per annum renewable discount loan from its bank for 3 months. The suppliers' terms are as follows:

Fort Co. 1/10, net 30Riley Manufacturing Co. 2/15, net 60Shad, Inc. 3/15, net 90

Using a 360-day year, the cheapest source of short-term financing in this situation is (D)A. The bank. C. Riley Manufacturing Co.B. Fort Co. D. Shad, Inc. CMA 1283 1-25

Trade Credit vs. Bank LoanPoint of Indifference

CMA EXAMINATION QUESTIONS Page 34 of 61

Page 35: P03 - Working Capital Finance

MANAGEMENT ADVISORY SERVICES WORKING CAPITAL FINANCE

. ABC Company finances all of its seasonal inventory needs from the local bank at an effective interest cost of 9%. The firm’s supplier promises to extend trade credit on terms that will match the 9% bank credit rate. What terms would the supplier have to offer (approximately)?a. 2/10, n/60. c. 2/10, n/90.b. 2/10, n/100. d. 3/10, n/60. RPCPA 1001

Credit Term vs. Simple Interest145. A company has accounts payable of $5 million with terms of 2% discount within 15 days, net

30 days (2/15 net 30). It can borrow funds from a bank at an annual rate of 12%, or it can wait until the 30th day when it will receive revenues to cover the payment. If it borrows funds on the last day of the discount period in order to obtain the discount, its total cost will be (M)A. $51,000 less. C. $100,000 less.B. $75,500 less. D. $24,500 more. CIA 1192 IV-54

146. A corporation is currently experiencing cash-flow problems and has determined that it is in need of short-term credit. It can either use its trade credit on $100,000 of accounts payable with terms of 1/10, net 30 or a 30-day note with a 20% annual simple interest rate. Which is the best alternative, and what is its effective rate of interest (rounded to a whole percentage and using a 360-day year)? (D) CIA 1188 IV-54A. The trade credit. Its effective rate is 10%. C. The note. Its effective rate is 17%.B. The trade credit. Its effective rate is 20%. D. The note. Its effective rate is 20%.

147. Every 15 days a company receives $10,000 worth of raw materials from its suppliers. The credit terms for these purchases are 2/10, net 30, and payment is made on the 30th day after each delivery. Thus, the company is considering a 1-year bank loan for $9,800 (98% of the invoice amount). If the effective annual interest rate on this loan is 12%, what will be the net dollar savings over the year by borrowing and then taking the discount on the materials? (D)A. $3,624 C. $4,800B. $1,176 D. $1,224 Gleim

148. Quickbow Company currently uses maximum trade credit by not taking discounts on its purchases. Quickbow is considering borrowing from its bank, using notes payable, in order to take trade discounts. The firm wants to determine the effect of this policy change on its net income. The standard industry credit terms offered by all its suppliers are 2/10, net 30 days, and Quickbow pays in 30 days. Its net purchases are $11,760 per day, using a 360-day year. The interest rate on the notes payable is 10 percent and the firm’s tax rate is 40 percent. If the firm implements the plan, what is the expected change in Quickbow’s net income? (VD)a. -$23,520 d. +$37,728b. -$32,160 e. +$62,880c. +$23,520 Brigham

Delayed Payment to Supplier or Discounted Interest*. Budi Corp. intends to acquire a new equipment to increase its capacity. It is estimated to cost

P2.4 million. A bank loan can finance the acquisition at ten (10%) percent discounted interest. Alternatively, the company may just delay payment to its suppliers. Presently, the company buys under terms of 2/10, net 40, but management believes payment could be delayed 30 additional days, without penalty; that is payment could be made in 70 days. Assuming 360 days a year, the company should (M)a. Borrow since it is cheaper by 1.13% than delaying payment to suppliers.b. Borrow since it is cheaper by 2.5% than delaying payment to suppliers.c. Delay payments to suppliers since it would cost 12% as against bank loan of 10%.d. Delay payment to suppliers since it does not cost anything. RPCPA 0597

46 . Answer (D) is correct. Because collections made using Bank 4's lockbox service will be accelerated by 2 days at a rate of $300,000 per day, the firm will have an additional $600,000 to invest. At a rate of 6%, the interest earned will be $36,000 per year. However, the bank will charge $12,000 (12 months x $1,000 per month) for its services. Thus, the firm will gain $24,000 ($36,000 - $12,000). Answer (A) is incorrect because Bank 1 will increase the firm's income by only $18,000. Answer (B) is incorrect because Bank 2 will increase the firm's income by only $12,000. Answer (C) is incorrect because Bank 3 will increase the firm's income by only $12,000.

47 . Answer (A) is correct. The first step in calculating the benefit received by a firm for speeding up collections is to multiply the amount of daily sales by the number of days saved. In this example, that amount would be $30,000 ($15,000 x 2). This number is then multiplied by the annual opportunity cost of funds, 8%, to get a benefit of $2,400. Answer (B) is incorrect because $1,200 is $15,000 x 8%. Answer (C) is incorrect because $600 is ($15,000 ÷ 2) x 8%. Answer (D) is incorrect because $7,500 is daily sales, $15,000, divided by two.

48 . REQUIRED: The net annual benefit or loss.DISCUSSION: (D) If collection time is 2 days, and average daily receipts are $100,000, the average cash balance will increase by $200,000 if the bank’s system is adopted. At a 6% interest rate, $200,000 will generate $12,000 of interest revenue annually. The $500 monthly charge by the bank will result in an annual expense of $6,000. Thus, the net annual benefit is $6,000 ($12,000 - $6,000).Answer (A) is incorrect because $3,000 miscalculates the annual service charge. Answer (B) is incorrect because $12,000 fails to deduct the annual service charge from the interest earned. Answer (C) is incorrect because $0 results from figuring the interest earned for only one day, not two.

CMA EXAMINATION QUESTIONS Page 35 of 61

Page 36: P03 - Working Capital Finance

MANAGEMENT ADVISORY SERVICES WORKING CAPITAL FINANCE

*. Meals Etc. has been very successful. It is the newest fastfood outlet at the Greenbelt area of Makati featuring ordinary Filipino food packed with banana leaves. After six months of operations, it needs to expand. The owner, Mr. K. Eng estimates that P2.4 million will be required to put up another outlet in the Ortigas area.Financing was offered by a friendly banker at 10 percent discounted interest. Alternatively, Mr. Eng is thinking of just delaying payment to its suppliers. All his sales are on cash basis. The company purchases under terms of 2/10, net 40 but Mr. Eng believes that he could delay payments by another 30 days without any problem. This means payment could be made in 70 days. Assuming 360 days a year, Meals Etc. should opt for (M)

49 . Answer (B) is correct. Since collections will be speeded up by three days, at the rate of $200,000 per day, the company will have an additional $600,000 to invest. At 12%, the interest earned would be $72,000 per year. However, the bank will charge $48,000 (12 months x $4,000 per month) for its services. Thus, the firm will increase its income by $24,000 ($72,000 - $48,000). Answer (A) is incorrect because the service will generate $24,000 in additional annual income, not losses. Answer (C) is incorrect because $66,240 is a nonsense number. Answer (D) is incorrect because $68,000 assumes an annual bank charge of $4,000.

50 . Answer (B) is correct. Given that collections will be accelerated by 2 days, at the rate of $300,000 per day, the company will have an additional $600,000 to invest. At 11%, the interest earned will be $66,000 per year. However, the bank will charge $36,000 (12 months x $3,000 per month) for its services. Thus, the firm will increase its pretax income by $30,000 ($66,000 - $36,000). Answer (A) is incorrect because profits will increase by $30,000. Interest revenue will exceed the incremental costs. Answer (C) is incorrect because $66,000 fails to subtract the $36,000 cost of the lockbox system. Answer (D) is incorrect because $63,000 assumes the annual fee is $3,000.

51 . REQUIRED: The amount a firm should be willing to pay annually for a new cash management system.DISCUSSION: (D) If cash outflows are $3 million per day, holding cash 2 extra days means that average balances should increase by $6 million. At a 10% interest rate, the additional $6 million would generate interest revenue of $600,000 per year. Thus, if the system can be acquired for $600,000 or less, it would be beneficial to do so.Answer (A) is incorrect because $6 million is the increase in cash, not the interest earned on that additional cash. Answer (B) is incorrect because $3 million is the amount of daily payments, not the savings. Answer (C) is incorrect because the daily payments should be multiplied by two, not divided by two.

53 . Answer (D) is correct. Given sales of $27,000,000, the average amount of daily sales must be $75,000 ($27,000,000/360 days). The increased accounts receivable balance is

a. Bank loan since its cost of 11.11% is cheaper than the cost of delaying payments of 12.24% RPCPA 0597

b. Delaying payments since it has no cost compared to the 10% discounted bank interest.c. Bank loan since its cost of 10% is cheaper than the cost of delaying payments of 12%.d. Delaying payments since its cost is only 2% compared to 10% discounted bank interest.

Credit Term vs. Loan with Compensation Balance RequirementQuestions 137 and 138 are based on the following information. CMA 0687 1-29 & 30

therefore $450,000 (6 days x $75,000). With an additional $450,000 of capital invested in receivables, the company's interest cost will increase by $36,000 per year (8% x $450,000). Thus, the company must save at least $36,000 per year to justify the change in procedures. Answer (A) is incorrect because $36,000 is the minimum savings required. Answer (B) is incorrect because $36,000 is the minimum savings required. Answer (C) is incorrect because $36,000 is the minimum savings required.

54 . Answer (D) is correct. The cost of carrying receivables equals average receivables times the variable cost ratio times the cost of money. Given sales of $45,000,000, regardless of which collection procedures are applied, the average daily sales must be $125,000 ($45,000,000 ÷ 360 days). The increase in the average receivables balance is therefore $625,000 (5 days x $125,000). Given an additional $625,000 of receivables, the company's additional required investment in receivables is $375,000 ($625,000 x 60% variable cost ratio), and the incremental pretax cost of this investment is $22,500 (6% x $375,000). Accordingly, the collection costs must be reduced by a pretax minimum of $22,500 to offset the cost of the increased investment in receivables. Answer (A) is incorrect because $375,000 is the increased investment in receivables. Answer (B) is incorrect because $37,500 results from omitting the variable cost ratio from the calculation. Answer (C) is incorrect because $125,000 is the average daily sales.

55 . REQUIRED: The checking account that should be chosen.DISCUSSION: (D) The standard account will cost $10 per month plus $8 in check charges ($0.10 x 80 checks), for a total of $18 per month or $216 per year. The premium account has no check charges, but it will require the depositor to maintain a balance of $2,000 more than desired. At a 10% cost of capital, the incremental $2,000 minimum deposit will cost $200 per year. Thus, the premium account should be selected because it is cheaper by $16 per year.Answer (A) is incorrect because the relevant cost of the minimum premium account deposit is based on the $2,000 incremental deposit, not the full $2,500. Answers (B) and (C) are incorrect because the savings on the premium account is $16.

56 . REQUIRED: The average cash balance.CMA EXAMINATION QUESTIONS Page 36 of 61

Page 37: P03 - Working Capital Finance

MANAGEMENT ADVISORY SERVICES WORKING CAPITAL FINANCE

Morton Company needs to pay a supplier's invoice of $50,000 and wants to take a cash discount of 2/10, net 40. The firm can borrow the money for 30 days at 12% per annum plus a 10% compensating balance.

149. The amount Morton Company must borrow to pay the supplier within the discount period and cover the compensating balance is (M)A. $55,000. C. $55,556.B. $55,056. D. $54,444.

DISCUSSION: The EOQ for inventory is a function of ordering cost per order, inventory demand, and carrying cost. In the cash model, the fixed cost per sale of securities is equivalent to the ordering cost, the demand for cash is similar to the demand for inventory, and the interest rate is effectively the cost of carrying a dollar of cash for the period. Substituting in the formula yields an optimal cash balance of about $6,928.Answer (A) is incorrect because $1,000 results from using 24% in the denominator. Answer (B) is incorrect because $2,000 results from using 6% in the denominator. Answer (C) is incorrect because $3,464 results from using 2% in the denominator.

57 . REQUIRED: The current price of a Treasury bill issued at a discount.DISCUSSION: (A) The 6% discount rate is multiplied times the face amount of the Treasury bill to determine the amount of interest the lender will earn. The interest on this Treasury bill is $3 (6% x 0.5 years x $100); thus, the purchase price is $97 ($100 - $3)Answer (B) is incorrect because the interest is for 180 days, not a full year. Answer (C) is incorrect because the purchase price will always be less than the face value when a Trasury bill is sold at a discount. Answer (D) is incorrect because the interest rate is 6% per year.

59 . Answer (A) is correct. It is important to recognize that this is nothing more than an economic-order-quantity (EOQ) problem. The annual demand is $2,000,000, the ordering cost is $50 per order, and carrying cost is $.08 per dollar. Plugging these amounts into the standard EOQ model gives the following result.

= $50,000

Thus, the firm can minimize its cost by withdrawing in $50,000 lots, or 40 times per year. Answer (B) is incorrect because the firm will minimize its costs with withdrawals of $50,000 each time. Answer (C) is incorrect because the firm will minimize its costs with withdrawals of $50,000 each time. Answer (D) is incorrect because the firm will minimize its costs with withdrawals of $50,000 each time.

60 . Answer (B) is correct. It is important to recognize that this is nothing more than an economic-order-quantity (EOQ) problem. The annual demand is $2,000,000, the ordering cost

150. Assuming Morton Company borrows the money on the last day of the discount period and repays it 30 days later, the effective interest rate on the loan is (M)A. 12.00%. C. 13.20%.B. 13.33%. D. 13.48%.

151. A company will receive cash from sales in 1 year that can be used to pay for materials. The supplier will allow payment in 1 year. If the company pays the supplier immediately, it will receive a 20% discount off the $100,000 purchase price, but it must borrow the full amount. A bank has offered the company three alternatives:

is $50 per order, and carrying cost is $.08 per dollar. Plugging these amounts into the standard EOQ model gives the following result.

= $50,000

Dividing $167,000 by $50,000 indicates the firm should make 3.34 withdrawals per month, or about every 9 days. Answer (A) is incorrect because only about $16,000 would be needed in each 3-day period, but $50,000 is a more economical withdrawal. Answer (C) is incorrect because the $50,000 would run out before the next withdrawal. Answer (D) is incorrect because the $50,000 would run out before the next withdrawal.

61 . Answer (B) is correct. Dividing $3 million of sales by 360 days results in an average of $8,333.33 per day. Multiplying the average daily sales by the 40 days outstanding results in $333,333. Answer (A) is incorrect because $500,000 is based on 60 days outstanding. Answer (C) is incorrect because $250,000 is based on 30 days outstanding. Answer (D) is incorrect because $75,000 is based on dividing sales by 40 days, which results in a meaningless solution.

62 . Answer (D) is correct. Dividing the $2,050,200 annual sales by 360 days produces average daily sales of $5,695. Multiplying $5,695 by the 48 days in receivables produces a total of $273,360. Answer (A) is incorrect because $96,000 results from assuming sales of $2,000 per day. Answer (B) is incorrect because $336,005 is based on the days of inventory rather than receivables. Answer (C) is incorrect because $182,240 nets the days of payables against the days of receivables.

63 . Answer (B) is correct. If the average age of receivables is 48 days, it means that 48/360 of the year's sales is tied up in receivables. Thus, 48/360 x $2,870,280 = $382,704. Answer (A) is incorrect because $127,567 is based on the 16 days difference between receivables and payables rather than 48 days. Answer (C) is incorrect because $478,381 is based on the 60 days of inventory. Answer (D) is incorrect because $637,839 is based on the

CMA EXAMINATION QUESTIONS Page 37 of 61

Page 38: P03 - Working Capital Finance

MANAGEMENT ADVISORY SERVICES WORKING CAPITAL FINANCE

1. A 1-year loan at 18% with no other fees,2. A 1-year loan at 15% with the provision that it maintains 20% of whatever amount it

borrows as noninterest-bearing compensating balances over the life of the loan, or 3. A guaranteed line of credit of $100,000 at 17% with the provision that the bank will collect

a 1% fee on the average amount of unused funds. The company expects to borrow no other funds. The company would achieve the lowest cost of financing by (M)

A. Allowing the supplier to finance the materials and making payment at the end of 1 year. B. Accepting the 1-year loan at 18% with no other provisions. C. Accepting the 1-year loan at 15% with the compensating balance provisions.

80 days of receivables added to payables rather than just 60 days.

65 . REQUIRED: The amount of accounts receivable for a firm with credit terms of 2/10, net 30.DISCUSSION: (D) The firm has daily sales of $45,000 consisting of 150 units at $300 each. For 30 days, sales total $1,350,000. Forty percent of these sales, or $540,000, will be uncollected because customers do not take their discounts. The remaining 60%, or $810,000, will be paid within the discount period. However, by the end of 30 days, only 2/3 of the $810,000 will be collected because the sales from days 21 through 30 are still within the discount period. Therefore, an additional $270,000 ($810,000 - $540,000) will still be uncollected after the 30th day, but will be subject to a discount. In total, the average receivable balance is $810,000, consisting of $540,000 on which no discount will be taken and $270,000 that will be paid within the discount period.Answer (A) is incorrect because 60% of the sales will be paid for within the 10-day discount period. Answers (B) and (C) are incorrect because these are based on a sales total of $1,500,000 for 30 days rather than $1,350,000.

66 . REQUIRED: The average balance in accounts receivable given the average payment period and sales per day.DISCUSSION: (B) If sals are $4,000 per day, and customers pay in 30 days, 30 days of sales are outstanding, or $120,000. Whether customers pay by credit card or cash, collection requires 30 days.Answer (A) is incorrect because $4,000 is only one day’s seales. Answer (C) is incorrect because invoices are outstanding for 30 days, not 12 days. Answer (D) is incorrect because $54,000 is based on the 45% of collections via credit card.

67. Accounts receivable Answer: a Diff: MFirst solve for current annual sales using the DSO equation as follows: 48 = $1,000,000/(Sales/360) to find annual sales equal to $7,500,000. If sales fall by 10%, the new sales level will be $7,500,000(0.9) = $6,750,000. Again, using the DSO equation, solve for the new accounts receivable figure as follows: 32 = AR/($6,750,000/360) or AR = $600,000.

D. Accepting the guaranteed line of credit at 17% with the fee required on the average amount of unused funds. CIA 1186 IV-43

Questions 158 through 160 are based on the following information. GleimMorton Company needs to pay a supplier's invoice of $60,000 and wants to take a cash discount of 2/10, net 40. The firm can borrow the money for 30 days at 11% per annum with a 9% compensating balance. Assume a 360-day year.

68 . Answer (C) is correct. Days' sales outstanding is the average collection period. Assuming that all sales are on credit, that one-third of the customers take advantage of the 5% cash discount and pay on day 10, and that the remaining two-thirds of the customers pay on day 20, the days' sales outstanding is 17 days [(.33 x 10 days) + (.67 x 20 days)]. Answer (A) is incorrect because 13 days includes only the days' sales outstanding for customers that do not take the cash discount. Answer (B) is incorrect because 15 days uses five days until payment for the customers that take the cash discount. Answer (D) is incorrect because 20 days equals the 20-day collection period for customers not taking the cash discount.

69 . REQUIRED: The projected days’ sales outstanding.DISCUSSION: (C) Given than 40% of sales will be collected on the 15th day, 40% on the 30th day, and 20% on the 45th day, the days’ sales outstanding can be determined by weighting the collection period for each group of receivables by its collection percentage. Hence, the projected days’ sales outstanding equal 27 days [(40% x 15) + (40% x 30) + 20% x 45)]. Answer (A) is incorrect because average receivables are outstanding for much more than 20 days. Answer (B) is incorrect because 24 days assumes 40% of receivables are collected after 15 days and 60% after 30 days. Answer (D) is incorrect because more receivables are collected on the 15th day than on the 45th day; thus, the average must be less than 30 days.

71 . Answer (C) is correct. The average collection period is the average time it takes to receive payment from customers. because one-half of the customers will pay on day 10 and half will pay on day 30, the average collection period is 20 days [.5(10 days) + .5(30 days)]. Answer (A) is incorrect because 10 days assumes all customers take the discount. Answer (B) is incorrect because 15 days assumes half of the customers pay on day 30 but ignores the remaining half of the customers who pay on day 10. Answer (D) is incorrect because 30 days assumes all customers pay on day 30.

72 . Answer (D) is correct. The expected average accounts receivable balance equals the average collection period times the credit sales per day. Thus, the average accounts

CMA EXAMINATION QUESTIONS Page 38 of 61

Page 39: P03 - Working Capital Finance

MANAGEMENT ADVISORY SERVICES WORKING CAPITAL FINANCE

152. The amount Morton Company must borrow to pay the supplier within the discount period and cover the compensating balance is (M)A. $60,000 C. $64,615B. $65,934 D. $58,800

153. Assuming Morton Company borrows the money on the last day of the discount period and repays it 30 days later, the effective interest rate on the loan is (M)A. 11% C. 12.09%B. 10% D. 9.90%

receivable balance is 45,205.48 {[(10,000 units sold on credit x 66 price) ÷ 365 days] x 25 days}. The foregoing calculation assumes that receivables are recorded at their gross amounts.Answer (A) is incorrect because 684.93 is based on annual credit sales of 10,000. Answer (B) is incorrect because 1,808.22 equals the credit sales per day. Answer (C) is incorrect because 27,123.30 is based on a 15-day average collection period.

73 . Answer (B) is correct. Seasonal dating is a credit policy under which a customer may make purchases early but payment is not due until the customer's retail selling season begins. When seasonal dating is used, the selling company incurs higher credit costs because customers have longer to pay. Thus, reduced credit costs are not an advantage of seasonal dating. Answer (A) is incorrect because reduced storage costs result when customers buy earlier. Answer (C) is incorrect because attractive credit terms result when customers can buy earlier and not pay until their selling season begins. Answer (D) is incorrect because, when customers buy earlier, uncertainty about sales volume is reduced.

74 . Answer (C) is correct. Assuming that all of the customers take advantage of seasonal dating and the discount, the collections on or before January 11 will be the number of units sold, times the unit selling price, times 1 minus the discount percentage, or $6,860 [(700 units x $10)(1 - .02)]. Answer (A) is incorrect because all of the customers are taking the discount and will pay on or before January 11. Answer (B) is incorrect because $6,370 does not include the collection of the revenue from units sold on January 1. Answer (D) is incorrect because $7,000 does not consider the 2% discount.

75 . Answer (D) is correct. Terms of 1/10, net 60 mean that a buyer can save 1% of the purchase price by paying 50 days early. In essence, not taking the discount results in the buyer's borrowing 99% of the invoice price for 50 days at a total interest charge of 1% of the invoice price. Because a year has 7.3 50-day periods (365 ÷ 50), the credit terms 1/10, net 60 yield an effective annualized interest charge of approximately 7.37% [(1% ÷ 99%) x 7.3]. If the prime rate were higher than 7.37%, the buyer would prefer to borrow from the vendor (i.e., not

154. If Morton fails to take the discount and pays on the 40th day, what effective rate of annual interest is it paying the vendor? (M)A. 2% C. 24.49%B. 24% D. 36.73%

Different Bank Loan OptionsSimple Interest vs. Discount19. To raise money for working capital, The Bigger Donut is considering a one-year loan from the

Philtrust Bank. Two alternatives are available;

pay within the discount period) rather than from a bank. Consequently, an 8% prime rate could cause the vendor's receivables to increase. Answer (A) is incorrect because the prime rate must be greater than 7.37% to make the company's terms preferable to those of a bank. Answer (B) is incorrect because the prime rate must be greater than 7.37% to make the company's terms preferable to those of a bank. Answer (C) is incorrect because the prime rate must be greater than 7.37% to make the company's terms preferable to those of a bank.

76 . Answer (B) is correct. This firm's daily credit sales equal $4,000 (40% x 1,000 units x $10 price). Hence, reducing the accounts receivable collection period by 10 days will decrease the balance by $40,000 (10 days x $4,000). Answer (A) is incorrect because a $4,000 decrease assumes a 1-day reduction. Answer (C) is incorrect because $240,000 is the total balance of accounts receivable under the new policy. Answer (D) is incorrect because $280,000 is the total balance of accounts receivable under the old policy.

78 . REQUIRED: The expected increase in the average accounts receivable balance as a result of relaxed credit standards.DISCUSSION: (D) Of the $40,500,000 of sales, 80% are expected to be on credit, a total of $32,400,000. Average daily credit sales are therefore $90,000 ($32,400,000 ÷ 360). If 30 days of sales are outstanding at any one time, the average balance in accounts receivable is $2,700,000. If credit sales increase by 20%, the aforementioned $90,000 average of daily sales will increase to $108,000 (120% x $90,000). For an average collection period of 40 days, the average accounts receivable balance will be $4,320,000 (40 x $108,000). Hence, the expected increase in the balance is $1,620,000 ($4,320,000 - $2,700,000).Answer (A) is incorrect because $540,000 would have been the increase if the average balance outstanding remains at 30 days of sales. Answer (B) is incorrect because $2,700,000 is the average amount in accounts receivable before any change in credit terms. Answer (C) is incorrect because $900,000 would have been the increase based solely on the increased number of days outstanding, assuming no increase in sales.

CMA EXAMINATION QUESTIONS Page 39 of 61

Page 40: P03 - Working Capital Finance

MANAGEMENT ADVISORY SERVICES WORKING CAPITAL FINANCE

Alternative 1: a P70,000, 15% note issued at face value.Alternative 2: a P70,000, non-interest bearing note discounted at 15%.The company plans to borrow on November 1, 1993 and the accounting period follows the calendar.If you are asked by the Chief Executive Officer on the preferable alternative to take, assuming whatever difference is material in amount and based on your knowledge of accounting for notes, what advise should you give the CEO of The Bigger Donut? (M)a. alternative 1 c. alternative 2b. either alternative d. Both alternatives

79 . Answer (B) is correct. Of the $50,000,000 of sales, 75% are expected to be on credit, a total of $37,500,000. Average daily credit sales are therefore $104,167 ($37,500,000 ÷ 360 days). If the average collection period is 20 days, the average balance in accounts receivable is $2,083,340 (20 x $104,167). If credit sales increase by 25%, average daily sales will increase to $130,209 (125% x $104,167). For an average collection period of 30 days, the average accounts receivable balance will be $3,906,270 (30 x $130,209). Hence, the expected increase in the balance is $1,822,930 ($3,906,270 - $2,083,340). Answer (A) is incorrect because $520,840 is based on a 20-day collection period and a 25% increase in credit sales. Answer (C) is incorrect because $2,083,340 is the average accounts receivable balance before any change in credit terms. Answer (D) is incorrect because $3,906,270 is the average accounts receivable balance given the change in credit terms.

80. Accounts receivable increase Answer: b Diff: M RDSO = $41,096/($250,000/365) = 60 days.New A/R = [($250,000)(1.5)/(365)](60)(1.5) = $92,466.Hence, increase in receivables = $92,466 - $41,096 = $51,370.

81. Accounts receivable Answer: a Diff: M RFirst solve for current annual sales using the DSO equation as follows: 50 = $1,000,000/(Sales/365) to find annual sales equal to $7,300,000. If sales fall by 10%, the new sales level will be $7,300,000(0.9) = $6,570,000. Again, using the DSO equation, solve for the new accounts receivable figure as follows: 32 = AR/($6,570,000/365) or AR = $576,000.

82 . REQUIRED: The expected discounts taken in the coming year if a 2% discount is allowed for early payment.DISCUSSION: (A) If 90% of the $24,000,000 of sales are on credit, $21,600,000 of sales will be subject to the discount. If 50% of the credit customers take the discount, discounts will be taken on sales of $10,800,000. The expected discount will be $216,000 (2$ of $10,800,000).Answer (B) is incorrect because $432,000 would have been the discount on all of the credit sales. Answer (C) is incorrect because only 90% of the sales are on credit; $240,000 would be correct only if a cash discount were allowed on cash sales as well as credit sales. Answer

Discount155. Fonze is considering borrowing $100,000 from a bank for 1 year. This borrowing is in the form

of a discounted note, and he wants the effective interest rate to be between 11% and 12%. Which of the following bank(s) could Fonze choose, if Bank 1 has a stated interest rate of 9%, Bank 2 has a stated interest rate of 9.5%, Bank 3 has a stated interest rate of 10%, and Bank 4 has a stated interest rate of 10.5%? (M)A. Bank 2 only. C. Either Bank 1 or Bank 3.B. Bank 3 only. D. Either Bank 3 or Bank 4. Gleim

(D) is incorrect because $480,00 is based on the assumption that all sales will be discounted.

84 . Answer (A) is correct. The cost of carrying receivables equals average receivables times the variable cost ratio times the cost of money. Under the old policy, average daily sales are $12,778 ($4,600,000 ÷ 360 days). Given a 30-day average collection period, the average receivables balance is $383,340 ($12,778 x 30 days). Under the new policy, average daily sales are $13,778 ($4,960,000 ÷ 360 days), and the average receivables balance is $482,230 ($13,778 x 35 days). Hence, the average balance is $98,890 higher under the new policy. Because the company's incremental (variable) costs are 50% of sales, the additional investment is $49,445 (50% x $98,890). The interest rate, or required rate of return, is 11%. Thus, the incremental pretax carrying cost is $5,439 (11% x $49,445). Answer (B) is incorrect because $10,878 fails to adjust for the proportion of incremental costs included in the additional receivables. Answer (C) is incorrect because $13,778 is the average daily sales under the new policy. Answer (D) is incorrect because $98,890 is the amount of the additional receivables.

85 . Answer (A) is correct. The first step is to determine the average investment in receivables under each policy. Under the old policy, average daily sales are $10,000 ($3,600,000/360 days). Given a 30-day average collection period, the average receivables balance is $300,000 ($10,000 x 30 days). Under the new policy, average daily sales are $11,000 ($3,960,000/360 days), and the average receivables balance is $396,000 ($11,000 x 36 days). Hence, the average balance is $96,000 higher under the new policy. Because the company's incremental (variable) costs are 60% of sales, the extra investment is only $57,600 (60% x $96,000). The interest rate, or required rate of return, is 10%. Thus, the incremental carrying cost is $5,760 (10% x $57,600). Answer (B) is incorrect because $9,600 equals the 10% required rate of return times the $96,000 differential between the average receivables balances. Answer (C) is incorrect because the differential between the average receivables balances is $96,000, not $136,000. Answer (D) is incorrect because $960 assumes a 10% variable cost ratio.

86 . REQUIRED: The annual benefit or loss resulting from a change in credit terms.CMA EXAMINATION QUESTIONS Page 40 of 61

Page 41: P03 - Working Capital Finance

MANAGEMENT ADVISORY SERVICES WORKING CAPITAL FINANCE

Simple Interest with Compensation Balance156. Hagar Company is planning on purchasing a larger warehouse next year. Hagar has contacted

four banks and received information about loans of $100,000. Bank 1 requires a compensating balance of 20% and has a stated interest rate of 7%. Bank 2 requires a compensating balance of 15% and has a stated interest rate of 8%. Bank 3 requires a compensating balance of 18% and has a stated interest rate of 7.5%. Bank 4 requires a compensating balance of 0% and has a stated interest rate of 9%. The bank with the lowest effective loan cost is (M)A. Bank 1. C. Bank 3.B. Bank 2. D. Bank 4. Gleim

DISCUSSION: The incremental sales will produce an increased contribution margin of $144,000 (20% x $720,000). However, that amount must be offset by the cost of funds invested in receivables. The variable costs associated with the incremental sales are $576,000 (80% of $720,000). Given a 75-day credit period, the average investment in receivables equals $120,000 ($576,000 x (75 ÷ 360). Accordingly, the cost of the investment in additional receivables is $24,000 (20% opportunity cost x $120,000), and the net benefit of the planned change in credit terms is $120,000 ($144,000 - $24,000).Answer (A) is incorrect because the company benefits from the change in credit terms. Answer (B) is incorrect because $28,800 results from multiplying the contribution margin by the 20% interest rate. Answer (C) is incorrect because $144,000 overlooks the costs created by having funds invested in receivables for 75 days,

87 . REQUIRED: The increase in after-tax profit as a result of an increase in sales.DISCUSSION: (B) The company’s manufacturing and selling costs exclusive of bad debts equal 70% of bad debts. Hence, the gross profit on the $100,000 increase in sales will be $30,000 (30% x $100,000). Assuming $15,000 of bad debts and $5,000 of collection expense, the increase in pretax income will be $10,000 ($30,000 - $20,000). Consequently, after0tax income will increase by $6,000 [$10,000 – (40% x $10,000).Answers (A), (C) and (D) are incorrect because after-tax income will increase by $6,000.

88 . Answer (B) is correct. The company's manufacturing and selling costs exclusive of bad debts equal 75% of sales. Hence, the gross profit on the $150,000 increase in sales will be $37,500 (25% x $150,000). Assuming $24,000 (16% x $150,000) of bad debts and $6,000 (4% x $150,000) of collection expense, the increase in pretax income will be $7,500 ($37,500 - $30,000). Consequently, after-tax income will increase by $4,650 [$7,500 - (38% x $7,500)]. Answer (A) is incorrect because $2,850 is based on a 62% tax rate, the complement of the actual tax rate. Answer (C) is incorrect because $7,500 is the pretax income. Answer (D) is incorrect because $8,370 omits collection costs from the calculation.

90 . Answer (C) is correct. This problem can be solved using incremental analysis. Since incremental profits will be 20% of sales, and bad debt expense (the only incremental expense) will be less than 20%, then both options will increase profits.

Questions 87 and 88 are based on the following information. GleimBurgers & Burgers' bank (Bank 1) has offered to lend the firm $5 million at an annual rate of 12%, provided the company maintains a compensating balance of 25% is maintained. However, a competitor bank (Bank 2) offered a $5 million loan at 15% annually with a 10% compensating balance. 157. Which bank offers the better deal, expressed on an annual basis? (M)

A. Bank 1 is better by .7%. C. Bank 2 is better by .7%.B. Bank 1 is better by 3.0%. D. Bank 2 is better by 2.0%.

Answer (A) is incorrect because, under both new policies, the incremental costs will be less than incremental revenues. Thus, profits will be increased under either Policy A or B. Answer (B) is incorrect because the 15% increase in cost will be less than the 20% profit margin. Answer (D) is incorrect because all policies will result in different profit levels.

91 . REQUIRED: The annual percentage cost of financing under a factoring agreement.DISCUSSION: (D) In an average month, the company will receive $80,000 at the time the receivables are sent to the factor. Over a year’s time, the interest on this average advance of $80,000 would be $8,000 at 10% interest. In addition, the factor will charge a 2% factor fee, or $24,000 ($100,000 x 12 x 0.02) over the course of a year. However, this $24,000 fee is offset by the $18,000 savings in collection expenses, producing a net outlay of only $6,000. Adding the $6,000 to the $8,000 of interest produces an annual net cost of $14,000. Dividing the $14,000 cost by the $80,000 of advanced funds results in a cost of 17.5%.Answer (A) is incorrect because 10% overlooks the factor fee. Answer (B) is incorrect because 12% overlooks the fact that the 2% fee recurs every month. Answer (C) is incorrect because 14% miscalculates the factor fee and the savings from reduced collection costs.

92 . Answer (C) is correct. The factor will withhold $6,000 (6% x $100,000) as a reserve against returns and allowances and $1,400 (1.4% x $100,000) as a commission. The remaining $92,600 will be reduced by interest at the rate of 15% annually. The interest charge should be $2,315, assuming a 360-day year [($92,600 x .15) ÷ (60-day payment period ÷ 360 days)]. The proceeds to be received by the seller equal $90,285 ($92,600 - $2,315). Answer (A) is incorrect because $92,600 ignores interest. Answer (B) is incorrect because $96,135 fails to deduct the 6% reserve. Answer (D) is incorrect because $85,000 assumes that the only amount withheld is a full year's interest on $100,000.

93 . Answer (D) is correct. The calculation of the cash that the company will initially receive from factoring $100,000 in receivables is as follows:Amount of receivables$100,000Minus 6% reserve(6,000)Minus 1% factor fee (1,000)Amount accruing to the company$93,000Minus: 10% x(90/360) x $93,000(2,325)Cash to be received$90,675Answer (A) is incorrect because the amount received is also reduced by the interest charged. Answer (B) is incorrect because $90,000 assumes the only deduction is for

CMA EXAMINATION QUESTIONS Page 41 of 61

Page 42: P03 - Working Capital Finance

MANAGEMENT ADVISORY SERVICES WORKING CAPITAL FINANCE

158. Which would be the better deal if the company usually keeps an average of $500,000 on deposit? (M)A. Bank 1 by 0.9%. C. Bank 2 by 1.0%.B. Bank 1 by 3.0%. D. Bank 2 by 0.9%.

Lump Sum Payment vs. Installment Payment159. Roger Retailer sells $20,000 of merchandise to Bob Buyer. Bob offers Roger two payment

options for the merchandise. The first option is Bob will pay Roger $20,000 in a lump sum 1 year from the date of purchase. The second option is Bob will pay Roger $750 every 2 weeks for 1 year, starting 2 weeks after the date of purchase (a total of 26 bi-weekly payments). If Roger's relevant discount rate is 0.5% per bi-weekly period, which option should Roger accept? (M)A. Bi-weekly payments, as they have a higher present value by $675. B. Either one, since they have the same present value. C. The lump sum payment, as it has a higher present value by $500. D. The lump sum payment, as it has a higher present value by $1,750. Gleim

Factoring vs. Different Bank Loan TermsQuestions 139 through 142 are based on the following information. CMA 1296 1-14 to 17The Frame Supply Company has just acquired a large account and needs to increase its working capital by $100,000. The controller of the company has identified the four sources of funds given below.A. Pay a factor to buy the company’s receivables, which average $125,000 per month and have

an average collection period of 30 days. The factor will advance up to 80% of the face value of receivables at 10% and charge a fee of 2% on all receivables purchased. The controller estimates that the firm would save $24,000 in collection expenses over the year. Assume the fee and interest are not deductible in advance.

B. Borrow $110,000 from a bank at 12% interest. A 9% compensating balance would be required.

C. Issue $110,000 of 6-month commercial paper to net $100,000. (New paper would be issued every 6 months.)

D. Borrow $125,000 from a bank on a discount basis at 20%. No compensating balance would be required.

Assume a 360-day year in all of your calculations.

160. The cost of Alternative A is (M)a. 10.0% c. 13.2%

10% interest for 360 days. Answer (C) is incorrect because the interest charged will be for 90 days, not a full year.

b. 12.0% d. 16.0%

161. The cost of Alternative B is (M)a. 9.0% c. 13.2%b. 12.0% d. 21.0%

162. The cost of Alternative C is (M)a. 9.1% c. 16.2%b. 10.0% d. 20.0%

163. The cost of Alternative D is (M)a. 20.0% c. 40.0%b. 25.0% d. 50.0%

ANSWER EXPLANATIONS

CMA EXAMINATION QUESTIONS Page 42 of 61

Page 43: P03 - Working Capital Finance

MANAGEMENT ADVISORY SERVICES WORKING CAPITAL FINANCE

94 . Answer (C) is correct. The factor will hold out $8,800 (8% x $110,000) as a reserve against returns and allowances and $1,650 (1.5% x 110,000) as a commission. That leaves $99,550 to be advanced to the seller. However, interest at the rate of 16% annually is also to be withheld. For 60 days that interest would amount to approximately $2,655 (assuming a 360-day year). The proceeds to be given to the seller equal $96,895 ($99,550 - $2,655). Answer (A) is incorrect because $81,950 equals $99,550 minus 16% of $110,000. Answer (B) is incorrect because $83,630 is a nonsense answer. Answer (D) is incorrect because $99,550 equals the amount received by the firm plus the interest charge.

95 . Answer (A) is correct. Company A will withhold $6,000 (6% x $100,000) as a reserve against returns and allowances and $1,400 (1.4% x $100,000) as a commission. The remaining $92,600 will be reduced by interest at the rate of 15% annually. The interest charge will be $2,315, assuming a 360-day year [($92,600 x .15) ÷ (60-day payment period ÷ 360 days)]. The proceeds to be received by Gatsby equal $90,285 ($92,600 - $2,315). Answer (B) is incorrect because Company B will produce proceeds of only $89,964. Answer (C) is incorrect because Company C will produce proceeds of only $90,190. Answer (D) is incorrect because Company D will produce proceeds of only $90,241.

96 . Answer (B) is correct. With 3,000 accounts, 2,700 (90%) would be categorized as prompt paying and 300 as slow paying. Multiplying 22% times the 300 slow-paying accounts results in 66 defaults. Multiplying 5% times the 2,700 prompt-paying accounts results in an additional 135 defaults, for a total of 201. Answer (A) is incorrect because 795 uses 22% on all accounts, not just 300 of them. Answer (C) is incorrect because 135 is only the defaults on the prompt-paying accounts. Answer (D) is incorrect because 66 is only the defaults on the slow-paying accounts.

97 . Answer (B) is correct. The firm will earn $100 of gross profit ($1,200 - $1,100) from each customer who pays but will lose $1,100 on each default. The income statement for the 300 slow-pay customers would appear as follows:Sales (300 x 1,200)$360,000CGS (300 x 1,100) 330,000Gross profit30,000- Bad debt expense (66 x 1,200) (79,200)Net loss $(49,200)Dividing $49,200 by the 300 accounts produces an average loss of $164. Answer (A) is incorrect because $100 is the potential profit if there are no defaults. Answer (C) is incorrect because $220 is the actual average loss per bad account but does not consider the $100 profit from the 78% who do pay. Answer (D) is incorrect because $264 is the write-off amount for each bad debt.

98 . Answer (C) is correct. This can be solved algebraically with x being average revenue: 300x - $330,000 - 66x = 0234x = $330,000x = 1410.26

If average revenues were at least $1,410.26, the income statement would appear as follows:Sales (300 @ $1,410.26) $423,078.00CGS 330,000.00Gross profit93,078.00Bad debt expense (66 x $1,410.26) 93,077.16Net income$ .84Alternatively, another way of addressing the problem is to determine the amount of gross profit that will have to be generated by the 234 paying customers to cover the $1,100 loss on each of 66 defaulting customers. Multiplying 66 x $1,100 produces a loss of $72,600. Dividing the $72,600 loss by the 234 paying customers results in $310.26 of gross profit that must be generated from every paying customer just to break even. Adding the $1,100 cost to the

CMA EXAMINATION QUESTIONS Page 43 of 61

Page 44: P03 - Working Capital Finance

MANAGEMENT ADVISORY SERVICES WORKING CAPITAL FINANCE

$310.26 produces a revenue of $1,410.26. Answer (A) is incorrect because increasing the revenue by the amount of the average loss per account is insufficient because the additional $164 will be collected from only 234 customers, not all 300. Answer (B) is incorrect because a price of $1,389.74 will still result in an overall loss. Answer (D) is incorrect because the $310.26 gross profit is added to cost, not revenue.

99 . Daily purchases = = $2,000.

Free trade credit = $2,000 15 = $30,000.

100

?. Phranklin’s net purchases are $800,000 x (1 - 0.02) = $784,000. Purchases per day are $784,000/360 = $2,177.78. Total trade credit is 40 x $2,177.78 = $87,111.20. Free trade credit is 15 x $2,177.78 = $32,666.70. Thus, costly trade credit, assuming discounts are taken, is $87,111.20 - $32,666.70 = $54,444.50. If discounts are not taken, then the maximum amount of costly trade credit is $87,111.20.

101

?. Approximate percentage cost = = 0.2099 = 21%.

Accounts payable = = $750,000.

102 . Nominal percentage cost = = 3.00 = 300%.

CMA EXAMINATION QUESTIONS Page 44 of 61

Page 45: P03 - Working Capital Finance

MANAGEMENT ADVISORY SERVICES WORKING CAPITAL FINANCE

103 . Answer (D) is correct. Assume that the gross amount of an invoice is $1,000. With a 2% discount, the buyer will pay only $980 on the tenth day. Thus, the seller is forgoing $20 to receive payment 20 days sooner than would otherwise be required. The 20-day interest rate is .0204 ($20/$980). The number of 20-day periods in a year is 18 (360/20). If the interest rate is 2.04% for each 20-day period, the annual interest rate (rounded to the nearest tenth) is 36.7% (18 x 2.04%).

CMA EXAMINATION QUESTIONS Page 45 of 61

Page 46: P03 - Working Capital Finance

MANAGEMENT ADVISORY SERVICES WORKING CAPITAL FINANCE

106 . Answer (C) is correct. The buyer could satisfy the $100 obligation by paying $98 on the 10th day. By choosing to wait until the 40th day, the buyer is effectively paying a $2 interest charge for the use of $98 for 30 days (40-day credit period - 10-day discount period). The interest rate on what is essentially a 30-day loan is 2.04081% ($2 ÷ $98). Extrapolating this 30-day rate to a yearly rate involves multiplying by the number of periods in a year. Thus, the effective annual rate is about 24.49% [2.04081% x (360 ÷ 30 days)].

112

Answer (A) is incorrect because the annual interest cost is 36.7%. Answer (B) is incorrect because the annual interest cost is 36.7%. Answer (C) is incorrect because the annual interest cost is 36.7%.

104 . Answer (D) is correct. Assume that the gross amount of an invoice is $1,000. Given a 3% discount, the buyer will pay $970 on the tenth day. Thus, the seller is forgoing $30 to receive payment 20 days sooner than would otherwise be required. The 20-day interest rate (rounded) is 3.09% ($30 ÷ $970). The number of 20-day periods in a year is 18 (360 ÷ 20). If the interest rate is 3.09% for each 20-day period, the annual interest rate (rounded) is 55.6% (18 x 3.09). Answer (A) is incorrect because 36.7% assumes terms of 2/10, net 30. Answer (B) is incorrect because 24.5% assumes terms of 2/10, net 40. Answer (C) is incorrect because 37.1% assumes terms of 3/10, net 40.

105 . Answer (D) is correct. Assume that an invoice is due in 45 days. However, the seller allows a 2% cash discount if the invoice is paid within 15 days. Given early payment, the seller will receive their money at least 30 days (45 - 15) sooner than the contract requires. However, the seller is effectively paying 2% of the invoice price to receive the money 30 days early. The approximate annual interest cost is 24% because a 360-day year contains 12 periods of 30 days each. Answer (A) is incorrect because the 2% savings is for 30 days only; the annualized interest rate paid for receiving the money early is about 24%. Answer (B) is incorrect because 16% assumes eight discount periods of 45 days each in a year rather than 12. Answer (C) is incorrect because 48% assumes 15-day discount periods.

CMA EXAMINATION QUESTIONS Page 46 of 61

Page 47: P03 - Working Capital Finance

MANAGEMENT ADVISORY SERVICES WORKING CAPITAL FINANCE

Answer (A) is incorrect because 2% is the discount rate. Answer (B) is incorrect because 18.36% is based on the 40-day credit period. Answer (D) is incorrect because 36.72% is based on a 20-day credit period.

107 . Answer (B) is correct. Payments should be made within discount periods if the return is more than the firm's cost of capital. With terms of 3/10, net 45, the buyer is earning a 3% savings for paying on the tenth day, or 35 days earlier than would otherwise be required. For example, on a $1,000 invoice, the payment would be only $970. The $30 savings is comparable to interest earned on a $970 loan to the vendor (the payment is not due for another 35 days). The interest rate on this hypothetical loan is 3.09278% ($30/$970). That return is for a 35-day period. Annualizing the return requires determining the number of 35-day periods in a year. Multiplying the return for 35 days times the periods in a year results in an annual rate of return of about 31.81% [3.09278% x (360 days/35 days)]. Answer (A) is incorrect because 55.67% is based on terms of 3/10, net 30. Answer (C) is incorrect because 22.27% is based on an earning period of 50 days. Answer (D) is incorrect because the discount is 3%, not 9%.

108 . Answer (B) is correct. The company will initially lose $2 by not taking the discount. This amount is partially offset by interest earned on $98 for 50 days of $.817. Thus, the net cost is $1.183 ($2.00 - $.817). Since a 360-day year has 7.2 fifty-day periods, the total annualized cost is $8.52 (7.2 x $1.183). The loss rate is about 8.7% ($8.52/$98). Answer (A) is incorrect because the cost of not taking the cash discount is 8.7%. The company will initially lose $2 by not taking the discount. This amount is partially offset by interest earned on $98 for 50 days of $.817. Thus, the net cost is $1.183 ($2.00 - $.817). Because a 360-day year has 7.2 periods of 50 days each, the total annualized cost is $8.52 (7.2 x $1.183). The loss rate is about 8.7% ($8.52/$98). Answer (C) is incorrect because the cost of not taking the cash discount is 8.7%. The company will initially lose $2 by not taking the discount. This amount is partially offset by interest earned on $98 for 50 days of $.817. Thus, the net cost is $1.183 ($2.00 - $.817). Because a 360-day year has 7.2 periods of 50 days each, the total annualized cost is $8.52 (7.2 x $1.183). The loss rate is about 8.7% ($8.52/$98). Answer (D) is incorrect because the cost of not taking the cash discount is 8.7%. The company will initially lose $2 by not taking the discount. This amount is partially offset by interest earned on $98 for 50 days of $.817. Thus, the net cost is $1.183 ($2.00 - $.817). Because a 360-day year has 7.2 periods of 50 days each, the total annualized cost is $8.52 (7.2 x $1.183). The loss rate is about 8.7% ($8.52/$98).

109 . Answer (C) is correct. Assume a $1,000 invoice. The discount would be $50, and the customer would have to pay $950. Thus, the customer is paying $50 for the extended use of $950 -- a rate of 5.26% for 50 days. Extrapolating this to a full year involves multiplying by the number of 50-day periods in a year, which is 7.3 (365 ÷ 50). Thus, 5.26% x 7.3 periods gives an annual rate of 38.4%. Answer (A) is incorrect because 30.64% is the result of assuming the use of $1,000 for 60 days. Answer (B) is incorrect because 36.60% is the result of assuming the use of $1,000 instead of $950. Answer (D) is incorrect because 45.39% is the result of using the wrong number of days.

110 . Nominal percentage cost = x = 36.7%.

111 . Nominal percentage cost = x = 21.41%.

CMA EXAMINATION QUESTIONS Page 47 of 61

Page 48: P03 - Working Capital Finance

MANAGEMENT ADVISORY SERVICES WORKING CAPITAL FINANCE

?. Calculate the nominal percentage, which is the nominal annual cost:

Nominal cost = = 0.0204 36 = 0.7344 = 73.44%.

Calculate the effective annual rate (EAR):Numerical solution:EAR = (1.0204)36 - 1.0 = 2.0689 - 1.0 = 106.89% 106.9%.

Financial calculator solution: (EAR)Inputs: P/YR = 36; NOM% = 73.44. Output: EFF% = 106.89% 106.9%.

113

?. The company pays every 45 days or 360/45 = 8 times per year. Thus, the average accounts payable are $5,000,000/8 = $625,000. The effective cost of trade credit can be found as follows:EAR = (1 + 2/98)360/30 - 1 = 1.2743 - 1 = 0.2743 = 27.43%.

114 . REQUIRED: The weighted-average annual interest rate.DISCUSSION: If the company pays Web Master within 10 days, it will save $500 (2% x $25,000). Thus, the company is effectively paying $500 to retain $24,500 ($25,000 - $500) for 20 days (30 – 10). The annualized interest rate on this borrowing is 36.7346% [($500 ÷ $24,500) x (360 days ÷ 20 days)]. Similarly, the company is, in effect, paying Softidee $2,500 (5% x $50,000) to hold $47,500 ($50,000 - $2,500) for 80 days (90 – 10). The annualized rate on this borrowing is 23.6842% [($2,500 ÷ $47,500) x (360 days ÷ 80 days)]. The weighted-average of these two rates based on average monthly purchases is 28.0%. {[36.7346% x ($25,000 ÷ $75,000)] + [23.6842% x ($50,000 ÷ $75,000)]}. This calculation, however, understates the true cost of not taking the discount because it does not consider the effects of compounding.Answer (A) is incorrect because 27.0% is based on gross purchases. Answer (C) is incorrect because 29.3% is an unweighted (simple) average based on gross purchases. Answer (D) is incorrect because 30.2% is an unweighted average of the two interest rates.

115 . REQUIRED: The true statement about the decision to use trade credit and pay at the end of the credit period.DISCUSSION: (D) The company is currently paying an annual rate of 28% (see previous question) to obtain trade credit and pay at the end of the credit period. This policy should be continued if trade credit is the only source of financing, or if other sources are available only at a higher rate.Answer (A) is incorrect because the company should continue the current practice unless alternative short-term financing is available at a lower rate. Answer (B) is incorrect because the weighted-average cost of capital is usually a concern in capital budgeting and is not as important in the decision process as the marginal cost of capital. Answer (C) is incorrect because the company should maintain its current practice if the cost of alternative long-term financing is higher.

116 . Face value =

CMA EXAMINATION QUESTIONS Page 48 of 61

Page 49: P03 - Working Capital Finance

MANAGEMENT ADVISORY SERVICES WORKING CAPITAL FINANCE

119 . Total to be repaid = $12,000(1.1019) = $13,222.80. Interest = $13,222.80 - $12,000 = $1,222.80.

Approximate rateAdd-on = = 0.2038 = 20.38%.

120 . The monthly payments would be:

Monthly payment = = $6,875.121

?. Approximate rate = = 20%.

122 . Answer (B) is correct. At 10%, the interest on a $500,000 loan is $50,000 per year. However, the $500,000 loan is effectively reduced to $450,000 of usable funds by the compensating balance requirement. Thus, the borrower pays $50,000 of interest for a $450,000 loan, an effective rate of 11.1% ($50,000/$450,000). Answer (A) is incorrect because 10% is the nominal rate. Answer (C) is incorrect because 9.1% equals $50,000 divided by $550,000. Answer (D) is incorrect because 12.2% equals $55,000 divided by $450,000.

123 . Answer (B) is correct. The compensating balance is $750,000, which comes from the product of $5,000,000 and 0.15. The interest charged is $5,000,000 x 0.1 = $500,000. Thus, the amount of funds available for use is $4,250,000. The cost of the loan is the interest cost of $500,000 divided by the effective amount of money available for use: Effective Cost = $500,000 ÷ $4,250,000 = 0.1176 = 11.76%. Answer (A) is incorrect because the effective cost of the loan is not equal to the annual rate of interest. Answer (C) is incorrect because 16.67% is the result of dividing the compensating balance by $4,500,000 (amount of interest charged minus the interest). Answer (D) is incorrect because 17.65% is the result of dividing the compensating balance by $4,250,000 (which is the interest minus the compensating balance).

124 . Answer (D) is correct. Annual interest is $36,000 ($300,000 x 12%), and the amount available is $255,000 [$300,000 - (15% x $300,000)]. Thus, the effective interest rate is 14.12% ($36,000 ÷ $255,000). Answer (A) is incorrect because 11.00% is the nominal rate for 11 months. Answer (B) is incorrect because 12.00% is the nominal rate of interest. Answer (C) is incorrect because 12.94% equals $33,000 (11 months of interest) divided by $255,000.

CMA EXAMINATION QUESTIONS Page 49 of 61

Page 50: P03 - Working Capital Finance

MANAGEMENT ADVISORY SERVICES WORKING CAPITAL FINANCE

125 . REQUIRED: The effective cost of a loan when a compensating balance is required.DISCUSSION: Interest on the loan is $7,000 (7% x $100,000). Given that the borrower has to maintain a 20% compensating balance, only $80,000 [$100,000 – (20% x $100,000)] is available for use. Thus, the company is paying $7,000 for the use of $80,000 in funds at an effective cost of 8.75% ($7,000 ÷ $80,000). Answer (A) is incorrect because the borrower has access to less, not more, than the face amount of the loan. Answer (B) is incorrect because the effective rate is higher than the contract rate as a result of the compensating balance requirement. Answer (C) is incorrect because 8.40% is 120% of the contract rate.

126 . REQUIRED: The effective interest rate given a compensating balance requirement.DISCUSSION: (D) The requirement to maintain a compensating balance of 20% of the $300,000 loan means that the borrower has effective use of only 80% of the loan or $240,000. The 8% interest rate applied to a $300,000 lon requires an annual interest expenditure of $24,000. In turn, paying $24,000 for the use of $240,000 indicates an effective interest rate of 10%.Answer (A) is incorrect because having only 80% of the borrowed funds available means the effective rate will be greater than the 8% contract rate. Answer (B) is incorrect because 8% is the nominal rate. Answer (C) is incorrect because 20% is the percentage of the required compensating balance.

127 . REQUIRED: The effective interest rate on a loan that requires a compensating balance of $25,000 above the company’s normal working balance. DISCUSSION: The $50,000 compensating balance requirement is partially satisfied by the company’s practice of maintaining a $25,000 balance for transaction purposes. Thus, only $25,000 of the loan will not be available for current use, leaving $225,000 of the loan usable. At 6% interest, the $250,000 loan would require an interest payment of $15,000 per year. This is partially offset by the 2% interest earned on the $25,000 incremental balance, or $500. Subtracting the $500 interest earned from the $15,000 of expense results in net interest expense of $14,500 for the use of $225,000 in funds. Dividing $14,500 by $225,000 produces an effective interest rate of 6.44%. Answer (B) is incorrect because 7.00% fails to consider that the $25,000 currently being maintained counts toward the compensating balance requirement. Answer (C) is incorrect because 5.8% fails to consider the compensating balance requirement. Answer (D) is incorrect because 6.66% fails to consider the interest earned on the incremental balance being carried.

128 . REQUIRED: The effective interest rate on a loan given a compensating balance requirement.DISCUSSION: (D) Of the $500,000 borrowed, the debtor has the use of only $450,000. The compensating balance provision requires a minimum balnace that is $50,000 greater than the balance the company usually maintains. At 8% on $500,000 loan, the annual interest expense is $40,000. However, this amount is reduced by the interest earned on the extra $50,000 in the checking account. At 3%, the extra $50,000 earns $1,500 per year. Thus, the net expense is $38,500. The effective interest rate is 8.555% ($38,500 ÷ $450,000).Answer (A) is incorrect because the effective interest rate must exceed the 8% contract rate because not all of the borrowed funds are available for the debtor’s use. Answer (B) is incorrect because 8.22% assumes incremental earnings on the checking account of $3,000. Answer (C) is incorrect because 9.25% is based on the assumption that the company ordinarily maintains a zero balance.

129 . Answer (D) is correct. Applying the 9% interest rate to a $100,000 loan results in interest expense of $9,000. If the loan is processed in the form of a discounted note, the interest will be deducted from the proceeds of the loan. Thus, the $9,000 of interest will be deducted from the $100,000 note, resulting in loan proceeds of $91,000. The borrower is paying $9,000 for a loan of $91,000, resulting in an effective interest rate of 9.89%. Answer (A) is incorrect because the lesser amount of funds available on a discounted note means the effective rate will be higher than the contract rate. Answer (B) is incorrect because 9% is the nominal rate (discount rate). Answer (C) is incorrect because 9.81% equals the nominal rate multiplied by 9%.

CMA EXAMINATION QUESTIONS Page 50 of 61

Page 51: P03 - Working Capital Finance

MANAGEMENT ADVISORY SERVICES WORKING CAPITAL FINANCE

= = = $112,359.55 $112,360.

130 . Answer (D) is correct. If the loan is discounted, the borrower receives the face amount minus the prepaid interest. Thus, the borrower will receive proceeds of $1,800,000 [$2,000,000 - (10% x $2,000,000)]. The effective interest rate is 11.11% ($200,000 ÷ $1,800,000).

131 . Answer (D) is correct. When a note is discounted, the interest is deducted from the face of the note and is never received by the borrower. At a 10% interest rate, the interest will be $100,000, meaning the borrower will receive only $900,000. A cost of $100,000 on $900,000 of available funds results in an effective rate of 11.1%. Answer (A) is incorrect because only $900,000 is available for use. Answer (B) is incorrect because only $900,000 is available for use. Answer (C) is incorrect because the effective rate is 11.1%.

132 . Answer (C) is correct. Under a discounted loan, the interest is subtracted from the face of the note to determine the proceeds. Thus, on a $30,000 note at 11%, the proceeds would be only $26,700. Dividing the $3,300 of interest by the $26,700 proceeds results in an effective rate of 12.36%. Answer (A) is incorrect because the effective rate on a discounted note will be higher than the contract rate. Answer (B) is incorrect because 10% results from adding the interest to the face of the note instead of deducting it. Answer (D) is incorrect because 11% is the contract rate, not the effective rate.

133 . Answer (C) is correct. Discount interest is subtracted before the loan proceeds are paid to the borrower. A compensating balance is an amount that the borrower must keep on deposit with the lender. The effective annual interest rate is increased by both the discount interest arrangement and by the compensating balance requirement. The effective rate equals the nominal rate dividend by one minus the sum of the nominal rate and the compensating balance percentage, or 20% [.15 ÷ (1.0 - .15 - .10)]. Answer (A) is incorrect because 16.67% does not adjust for the discount interest arrangement. Answer (B) is incorrect because 17.65% does not adjust for the compensating balance requirement. Answer (D) is incorrect because 25.00% is the sum of the nominal rate and the compensating balance percentage.

134 . Answer (D) is correct. Assuming a $1,000 loan, the interest at 7% for 1 year is $70. Hence, the proceeds of the loan are $930 ($1,000 - $70). Also, 20% of the note, or $200, cannot be used by the borrower because of the compensating balance requirement. Consequently, only $730 is available for use by the borrower. Paying $70 interest for the use of $730 gives an interest rate of $9.59% ($70/$730). Answer (A) is incorrect because 8.75% equals $70 divided by $800. Answer (B) is incorrect because 9.41% equals $70 divided by $744. Answer (C) is incorrect because 7.53% equals $70 divided by $930.

135 . Answer (D) is correct. Assuming a $1,000 loan, the interest at 6% for 1 year is $60. Hence, the proceeds of the loan with discounted (paid-in-advance) interest are $940 ($1,000 - $60). Also, 22% of the note, or $220, cannot be used by the borrower because of the compensating balance requirement. Consequently, only $720 is available for use. Paying $60 interest for the use of $720 results in an effective cost of borrowing of 8.33% ($60 ÷ $720). Answer (A) is incorrect because 6% is the contract rate. Answer (B) is incorrect because 6.38% does not consider the compensating balance requirement. Answer (C) is incorrect because 7.69% assumes the interest is not paid in advance.

CMA EXAMINATION QUESTIONS Page 51 of 61

Page 52: P03 - Working Capital Finance

MANAGEMENT ADVISORY SERVICES WORKING CAPITAL FINANCE

Answer (A) is incorrect because the prepayment of interest reduces the funds available, resulting in an effective interest rate greater than the contract rate. Answer (B) is incorrect because the prepayment of interest reduces the funds available, resulting in an effective interest rate greater than the contract rate. Answer (C) is incorrect because 10% is the contract rate. The effective rate is higher because the full $2 million face amount of the note will not be available to the borrower.

117

?. Convert the annual rate to a periodic rate (quarterly) in the denominator of the face value formula:

Face value =

= = = $61,855.67 $61,856.

118 . Answer (B) is correct. Simple interest is charged on the amount actually paid to the borrower. If interest is charged on a simple basis, the full $20,000 face value of the loan is made available to the borrower.Answer (A) is incorrect because interest is charged on a discount basis when it is deducted from the face value borrowed. Answer (C) is incorrect because interest charged on a discount basis results in a deduction from the face value. The borrower does not receive the full face value of the loan. Answer (D) is incorrect because interest is charged on an add-on basis when the face value of the loan initially equals the borrowed amount plus the nominal interest charge.

CMA EXAMINATION QUESTIONS Page 52 of 61

Page 53: P03 - Working Capital Finance

MANAGEMENT ADVISORY SERVICES WORKING CAPITAL FINANCE

136 . Answer (B) is correct. An amount-of-one table can be used to find the effective rate when the APR is known. An annual rate of 9% is equivalent to .75% per month. Using the .75% column, the accumulated interest factor is .093806, or 9.38% for the year. Answer (A) is incorrect because 9% is the APR. Answer (C) is incorrect because 9.81% is too high. Answer (D) is incorrect because 10.94% is a nonsense answer.

137 . Answer (A) is correct. The annual percentage rate is easy to compute: multiply the 1.25% monthly rate times 12 months to arrive at 15%. The compounded rate would be slightly higher. Thus, given the alternatives, answer (A) is the only acceptable alternative where the compounded rate is greater than the 15% APR. To check your solution, look at an "Amount of 1" table for 12 periods at 1.25%. The accumulated interest is .16075, or 16.8%. Answer (B) is incorrect because it has both amounts wrong. Answer (C) is incorrect because the compounded rate will be greater than the 15% APR. Answer (D) is incorrect because it substitutes the APR for the compounded rate.

138 . REQUIRED: The effective interest rate on the borrowing.DISCUSSION: (B) The effective interest rate on the 180-day borrowing is equal to the net interest cost divided by the net available proceeds of $1,800,000 ($2,000,000 loan – $200,000 increase in the compensation balance). The net interest cost is equal to the gross interest cost minus the incremental interest revenue. The gross interest cost is $120,000 [$2,000,000 x 12% x (6 12)]. Because the incremental interest revenue is $6,000 [$200,000 x 6% x (6 12)], the net interest cost if $114,000 ($120,000 – $6,000). The six-month effective interest rate is therefore 6.33% ($114,000 $1,800,000). The annual effective interest rate is 12.67% (6.33% x 2).Answer (A) is incorrect because 12% is the annual rate. Answers (C) and (D) are incorrect because the interest revenue from the checking account must be included in the calculations.

139 . Answer (B) is correct. The total cost to the company will be $21,200 ($20,000 discount + $1,200 of transaction costs), and the net amount available will be $978,800. The annualized amount of the costs is $84,800 (4 x $21,200). Accordingly, the annual interest cost will be 8.66% ($84,800/$978,800). Answer (A) is incorrect because the annual interest cost will be 8.66%. Answer (C) is incorrect because the annual interest cost will be 8.66%. Answer (D) is incorrect because the annual interest cost will be 8.66%.

140 . Answer (C) is correct. The total cost to the company will be $51,500 (50,000 discount + 1,500 of transaction costs), and the net amount available will be $1,448,500. The annualized amount of the costs is $206,000 (4 x $51,500). Accordingly, the annual interest cost will be 14.22% ($206,000 ÷ $1,448,500). Answer (A) is incorrect because 3.45% ignores the transaction costs and fails to annualize the percentage. Answer (B) is incorrect because 3.56% fails to annualize the result. Answer (D) is incorrect because 13.79% ignores the transaction costs.

141

?. Interest rate on borrowed funds = 0.09 + 0.015 = 10.5%.Cost of unused portion: $4,000,000 0.005 = $ 20,000Cost of used portion: $6,000,000 0.105 = 630,000

CMA EXAMINATION QUESTIONS Page 53 of 61

Page 54: P03 - Working Capital Finance

MANAGEMENT ADVISORY SERVICES WORKING CAPITAL FINANCE

142 . Answer (C) is correct. At 10% interest, the annual expense would be $120,000 on a loan of $1.2 million. However, because of the 15% compensating-balance requirement, $180,000 of the funds cannot be used. Therefore, usable funds are only $1,020,000. Dividing the $120,000 of interest by the $1,020,000 results in an effective rate of 11.76%. Answer (A) is incorrect because 10% is the contract rate, not the effective rate. Answer (B) is incorrect because 11.11% results from assuming a 10% compensating balance. Answer (D) is incorrect because 13.33% results from assuming a $300,000 compensating balance instead of $180,000.

143 . REQUIRED: The interest expense for six months.DISCUSSION: The sum of the beginning balance and inflows exceeds the outflows for the first 2 months. At the end of March, however, Garth must use $2,000,000 of its line of credit ($2,000,000 beginning balance + $6,000,000 inflows - $100,000,000 outflows). Thus, interest for April is $20,000 (1% x $2,000,000). The net cash outflow for April (ignoring short-term borrowings) is $1,000,000 of an additional $1,000,000 of the line of credit). However, the $20,000 of interest for April must also be paid, so the amount of the line of credit used in May is $3,020,000 ($2,000,000 + $1,000,000 + $20,000). Interest for May is therefore $30,200 (1% x $3,020,000). Given the net cash inflow for May of $2,000,000 (again ignoring short-term borrowings) and the borrowing of $30,200 to pay the interest for May, the amount of the line of credit used in June is $1,050,200. Interest in June is $10,502 (1% x $1,050,200), and total interest is $60,702 ($20,000 + $30,200 + $10,502). Consequently, the closest answer is $61,000.Answer (A) is incorrect because interest must be paid monthly when the credit line is used in April, May, and June. Answers (C) and (D) are incorrect because the company would repay the credit line at the end of months with a positive cash flow.

144 . Answer (D) is correct. The first step is to determine the actual annual percentage interest rate for each of the four options. Assuming a $100 invoice, the Fort Company discount represents interest of $1 on a loan of $99 for 20 days (30-day credit period - 10-day discount period). The annual interest rate is 18.1818% [(360/20) periods x ($1/$99)]. The Riley Company discount represents an interest charge of $2 on a loan of $98; i.e., by not paying on the 15th day, the company will have the use of $98 for 45 days (60-day credit period - 15-day discount period). The number of periods in a year would be 8 (360/45). The interest would be 16.326% ($2/$98 x 8 periods). The Shad loan would be for $97 at a cost of $3. The loan would be for 75 days (90 - 15). Given 4.8 interest periods in a year (360/75), the annual interest rate would be 14.845% ($3/$97 x 4.8). The bank loan was quoted at 14.75% on a discount basis. On a $100 note, the borrower would only receive $85.25, giving an interest rate of 17.302% ($14.75/$85.25). Thus, not paying Shad, Inc.'s invoices on time would be the lowest cost source of capital, at a cost of 14.845%. Answer (A) is incorrect because the actual annual percentage rate based on forgoing Shad's discount is 14.845%. This is lower than the rate on the bank loan (17.302% because it is a discount loan), or the cost of forgoing the discounts allowed by Fort Co. (18.182%) and Riley Co. (16.326%). Answer (B) is incorrect because the actual annual percentage rate based on forgoing Shad's discount is 14.845%. This is lower than the rate on the bank loan (17.302% because it is a discount loan), or the cost of forgoing the discounts allowed by Fort Co. (18.182%) and Riley Co. (16.326%). Answer (C) is incorrect because the actual annual percentage rate based on forgoing Shad's discount is 14.845%. This is lower than the rate on the bank loan (17.302% because it is a discount loan), or the cost of forgoing the discounts allowed by Fort Co. (18.182%) and Riley Co. (16.326%).

145 . Answer (B) is correct. The interest cost of borrowing $4,900,000 (98% x $5,000,000) to take advantage of the discount is $24,500 [$4,900,000 x 12% x (15 ÷ 360)], and the total cost will be $4,924,500. The total cost if the discount is not taken will be $5,000,000, a difference of $75,500. Answer (A) is incorrect because $51,000 less is based on a 30-day borrowing period. Answer (C) is incorrect because $100,000 less does not consider the interest paid. Answer (D) is incorrect because $24,500 more reflects interest paid but ignores the discounted price.

CMA EXAMINATION QUESTIONS Page 54 of 61

Page 55: P03 - Working Capital Finance

MANAGEMENT ADVISORY SERVICES WORKING CAPITAL FINANCE

Total cost of loan agreement $650,000146 . Answer (B) is correct. The corporation can obtain trade credit for 20 additional days by not

paying within the discount period. Instead of paying $99,000 to satisfy its obligation within 10 days, it can pay $100,000 at the end of 30 days. The corporation will thus incur $1,000 in interest to hold the $99,000 for the 20 days. Because a 360-day year has 18 such periods, the interest rate is approximately 18.18% [($1,000 ÷ $99,000) x 18]. However, if compounding effects are considered, the rate is higher. The effective rate, taking compounding into consideration, is found using the following formula:

Answer (A) is incorrect because the trade credit rate is 18.18% or 19.83% depending on the method of calculation. Answers (C) and (D) are incorrect because the note has an effective rate, including compounding effects, of 21.94%. The following is the calculation:

147 . Answer (A) is correct. The company will save $200 (2% x $10,000) every 15 days. There are 24 15-day periods in a 360-day year. Thus, the gross savings will be $4,800. The interest expense on a $9,800 loan at 12% is $1,176. Deducting the $1,176 interest expense from the $4,800 of discounts results in a net savings of $3,624. Answer (B) is incorrect because $1,176 is the amount of the interest expense. Answer (C) is incorrect because $4,800 is the gross savings before deduction of interest. Answer (D) is incorrect because $1,224 is based on only $2,400 of discounts, instead of $4,800.

148 . Calculate A/P with and without taking discounts:A/PNo discount = $11,760 x 30 days = $352,800.A/PDiscount = $11,760 x 10 days = $117,600.

Calculate financing amount in notes payable and interest cost. The firm will need to borrow the difference in notes payable.$352,800 - $117,600 = $235,200.The additional interest cost is $235,200 x 0.10 = $23,520.

Calculate total purchases and discounts lost:Total purchases = 360 days x 12,000 gross purchases = $4,320,000.Discounts lost = $4,320,000 x 0.02 = $86,400.

Construct comparative financial statements:I. Partial balance sheet:Take DiscountsDon’t Take Discounts (Borrow N/P) (Use Max. Trade Cdt) DifferenceAccounts payable$117,600 $352,800 -$235,200Notes payable (10%) 235,200

- +235,200 Total current liab.$352,800 $352,800 $ 0 II. Partial income statementEBIT & discounts lost $140,000$140,000 $ 0Less: Interest -23,5200 +23,520 Discounts lost 0 -86,400 -86,400EBT116,480 53,600 +62,880Less: Taxes (at 40%) 46,592 21,440 +25,152Net income $ 69,888 $ 32,160

$ 37,728CMA EXAMINATION QUESTIONS Page 55 of 61

Page 56: P03 - Working Capital Finance

MANAGEMENT ADVISORY SERVICES WORKING CAPITAL FINANCE

149 . Answer (D) is correct. The company will need $49,000 (98% x $50,000) to pay off the invoice. In addition, it will need a compensating balance equal to 10% of the loan. This can be written in equation form as: Loan = $49,000 + (.1)Loan. Thus, the loan amount needed is $54,444 ($49,000/.9). Answer (A) is incorrect because $55,000 is 110% of the invoice. Answer (B) is incorrect because $55,056 is a nonsense number. Answer (C) is incorrect because $55,556 assumes no cash discount.

150 . Answer (B) is correct. The company will need $49,000 (98% x $50,000) to pay off the invoice. In addition, it will need a compensating balance equal to 10% of the loan. Thus, the loan amount needed is $54,444 ($49,000/.9).The interest at 12% annually on a 30-day loan of $54,444 is $544.44 ($54,444 x 12% x 30/360). However, the company has access to only $49,000. Thus, the interest expense on usable funds is at an annual rate of 13.33% (12 months x $544.44/$49,000). Answer (A) is incorrect because 12.00% assumes that the company has access to loan funds of $54,444. Answer (C) is incorrect because $54,444, not $55,000, is the amount of the loan, and the annual interest expense should be divided by $49,000 of usable funds, not $50,000. Answer (D) is incorrect because $54,444, not $55,056, is the amount of the loan used to determine annual interest expense.

151 . Answer (D) is correct. If the company chooses the line of credit, it will pay 17% interest on $80,000 ($100,000 - $20,000 discount) and 1% on the $20,000 unused portion, a total of $13,800. The effective interest rate would thus be 17.25% ($13,800 ÷ $80,000).Answer (A) is incorrect because if the company forgoes the cash discount, its effective rate is 25% ($20,000 ÷ $80,000 immediate cash price). Answer (B) is incorrect because an 18% effective rate exceeds that on the line of credit. Answer (C) is incorrect because the effective rate would be 18.75% [(15% x $100,000) ÷ $80,000 available funds].

CMA EXAMINATION QUESTIONS Page 56 of 61

Page 57: P03 - Working Capital Finance

MANAGEMENT ADVISORY SERVICES WORKING CAPITAL FINANCE

152 . Answer (C) is correct. The company will need $58,800 (98% x $60,000) to pay the invoice. In addition, it will need a compensating balance equal to 9% of the loan. The equation is Loan = $58,800 + .09 Loan. Thus, the loan amount needed is $64,615 ($58,800 ÷ .91). Answer (A) is incorrect because $60,000 is the invoice amount. Answer (B) is incorrect because $65,934 assumes the amount paid to the supplier is $60,000. Answer (D) is incorrect because $58,800 is the amount to be paid to the supplier.

153 . Answer (C) is correct. The interest at 11% annually on a 30-day loan of $64,615 is $592.30 [$64,615 x 11% x (30 ÷ 360)]. However, the company has access to only $58,800. The interest expense on usable funds is therefore at an annual rate of 12.09% [12 months x ($592.30 ÷ $58,800)]. Answer (A) is incorrect because 11% is the contract rate of interest. Answer (B) is incorrect because the effective rate is greater than the contract rate. The usable funds are less than the face amount of the note. Answer (D) is incorrect because the effective rate is greater than the contract rate. The usable funds are less than the face amount of the note.

154 . Answer (C) is correct. By failing to take the discount, the company is essentially borrowing $58,800 for 30 days. Thus, at a cost of $1,200, the company acquires the use of $58,800, resulting in a rate of 2.04081% ($1,200 ÷ $58,800) for 30 days. Assuming a 360-day year, the effective annual rate is 24.489% [2.04081% x (360 days ÷ 30 days)]. Answer (A) is incorrect because 2% is the discount rate for a 30-day period. Answer (B) is incorrect because 24% assumes that the available funds equal $60,000. Answer (D) is incorrect because 36.73% assumes a 20-day discount period.

155 . Answer (D) is correct. If the loan is processed in the form of a discounted note, the interest will be deducted from the proceeds of the loan. Thus, on the 10% loan, the $10,000 of interest will be deducted from the $100,000 note, resulting in loan proceeds of $90,000. The borrower is paying $10,000 for a loan of $90,000, resulting in an effective interest rate of 11.11%. On the 10.5% loan, the $10,500 of interest will be deducted from the $100,000 note, resulting in proceeds of $89,500. The borrower is paying $10,500 for a loan of $89,500, resulting in an effective interest rate of 11.73%. Answer (A) is incorrect because Bank 2 has an effective interest rate of 10.49%. Answer (B) is incorrect because Bank 4 also has an effective interest rate between 11% and 12%. Answer (C) is incorrect because Bank 1 has an effective interest rate of 9.89%.

CMA EXAMINATION QUESTIONS Page 57 of 61

Page 58: P03 - Working Capital Finance

MANAGEMENT ADVISORY SERVICES WORKING CAPITAL FINANCE

156 . Answer (A) is correct. The effective cost of a loan is found by dividing the interest on the loan by the amount available for use. Therefore, the interest charged by Bank 1 is $7,000 (7% x $100,000). Given that the borrower has to maintain a 20% compensating balance, only $80,000 [$100,000 - (20% x $100,000)] is available for use. Thus, the company is paying $7,000 for the use of $80,000 in funds at an effective cost of 8.75% ($7,000 ÷ $80,000). Answer (B) is incorrect because Bank 2 offers an effective loan cost of 9.41%. Answer (C) is incorrect because Bank 3 offers an effective loan cost of 9.15%. Answer (D) is incorrect because Bank 4 offers an effective loan cost of 9.00%.

157 . Answer (A) is correct. Since Bank 1 has a 25% compensating balance requirement, the amount of available funds is $3,750,000 (75% x $5 million). Interest at 12% is $600,000. Dividing $600,000 by $3,750,000 results in an effective rate of 16.0%. For Bank 2, the interest is $750,000. Dividing $750,000 in interest by $4.5 million of available funds results in an effective rate of 16.7%. Thus, the rate on the loan from Bank 1 is 0.7% less than the rate on the loan from Bank 2. Answer (B) is incorrect because 3% is the difference in the two contract rates, not the effective rates. Answer (C) is incorrect because the effective rate for Bank 1 is lower than for Bank 2. Answer (D) is incorrect because the effective rate for Bank 1 is lower than for Bank 2.

CMA EXAMINATION QUESTIONS Page 58 of 61

Page 59: P03 - Working Capital Finance

MANAGEMENT ADVISORY SERVICES WORKING CAPITAL FINANCE

158 . Answer (A) is correct. If the firm plans to have $500,000 on deposit in any case, the Bank 1 loan will require only $750,000 more to maintain the compensating balance, meaning that $4,250,000 of the loan will be available for use. Dividing $600,000 by $4,250,000 results in an effective rate of 14.1%. All of the loan from Bank 2 would be available, resulting in an effective rate equal to the 15% contract rate. Answer (B) is incorrect because 3% is the difference in contract rates, not the effective rates. Answer (C) is incorrect because Bank 1 has a lower effective rate. Answer (D) is incorrect because Bank 1 has a lower effective rate.

159 . Answer (A) is correct. To make a capital budget decision, the present values of the inflows must be determined and compared to see which is the greater value. The lump sum payment is a one-time payment at a set date at a specified interest rate. The formula to determine the present value of the lump sum is

Where i = interest rate, .005, and n = number of periods, 26 bi-weekly periods. The computation of the present value of the lump sum is as follows:

The present value of the bi-weekly payments can be calculated using the formula for the present value of an ordinary annuity, as the interest compounds bi-weekly. The formula is:

Where i = interest rate, .005, and n = number of periods, 26 bi-weekly periods. The computation of the present value of the bi-weekly payments is as follows: CMA EXAMINATION QUESTIONS Page 59 of 61

Page 60: P03 - Working Capital Finance

MANAGEMENT ADVISORY SERVICES WORKING CAPITAL FINANCE

160 . REQUIRED: The annual percentage cost of factoring receivables.DISCUSSION: (D) The factor will advance $100,000 (80% of $125,000). This amount is the average balance outstanding throughout the year. Thus, annual interest will be $10,000 (10% x $100,000). In addition, the company will pay an annual fee of $30,000 (2%x $125,000 per month x 12 months), so the total annual net cost is $16,000 ($10,000 + $30,000 - $24,000 savings). Hence, the annual cost is 16% ($16,000 ÷ $100,000).Answer (A) is incorrect because 10% is the interest rate on the amount advanced. Answer (B) is incorrect because 12% is the sum of the interest rate and the fee percentage. Answer (C) is incorrect because 13.2% is the cost of Option B.

161 . REQUIRED: The annual percentage cost of borrowing with a compensating balance requirement.DISCUSSION: (C) Even though the company will borrow $110,000, it will have use of only $100,100 because a 9% compensating balance, or $9,900, must be maintained at all times. Consequently, the effective annual interest rate is 13.2% [(12% x $110,000) ÷ $100,000].Answer (A) is incorrect because 9% is the compensating balance requirement. Answer (B) is incorrect because 12% is the contract rate. Answer (D) is incorrect because 21% is the sum of the contract rate and the compensating balance requirement.

162 . REQUIRED: The annual percentage cost of issuing commercial paper.DISCUSSION: (D) By issuing commercial paper, the company will receive $100,000 and repay $110,000 every six months. Thus, for the use of $100,000 in funds, the company pays $10,000 in interest each six-month period or a total of $20,000 per year. The annual percentage rate is therefore 20% ($20,000 ÷ $100,000).Answer (A) is incorrect because 9.1% is the 6-month rate based on the face amount of the paper. Answer (B) is incorrect because 10% is the rate for six months. Answer (C) is incorrect because 18.2% is based on the face amount of the commercial paper.

163 . REQUIRED: The annual percentage cost of a discounted note.DISCUSSION: (B) The company will receive $100,000 (80% x $125,000) at an annual cost of $25,000), so the annual interest rate is 25% ($25,000 ÷ $100,000).Answer (A) is incorrect because the effective rate must exceed the contract rate of 20%. Answer (C) is incorrect because 40% assumes no discount and a 6-month loan term. Answer (D) is incorrect because 50% assumes a 6-month loan term.

CMA EXAMINATION QUESTIONS Page 60 of 61

Page 61: P03 - Working Capital Finance

MANAGEMENT ADVISORY SERVICES WORKING CAPITAL FINANCE

By comparing the present values of the options ($17,568; $18,243), Roger should choose the second option of bi-weekly payments, as the present value is higher by $675. Answer (B) is incorrect because the two options will have different present values. Answer (C) is incorrect because the biweekly payments have a higher present value. Answer (D) is incorrect because the biweekly payments have a higher present value.

CMA EXAMINATION QUESTIONS Page 61 of 61